Torts BarPrep

Ace your homework & exams now with Quizwiz!

4) Effect of Compliance with Statute Even though the violation of an applicable criminal statute may be negligence, compliance with it will not necessarily establish ...

due care. If there are unusual circumstances or increased danger beyond the minimum that the statute was designed to meet, it may be found that there is negligence in not doing more.

Res Ipsa Loquitur relates to which elements most?

duty and breach; include in your breach discussion, because it relates to a duty and breach.

If there is not just one D in negligence, don't use "but for" actual cause, use the...

alternative causes approach.

*(1) Definition of Licensee*

A licensee is one who enters on the land with the landowner's permission, express or implied, for her own purpose or business rather than for the landowner's benefit.

A four-year-old child sustained serious injuries when a playmate pushed him from between two parked cars into the street, where he was struck by a car. The child, by his representative, sued the driver of the car, the playmate's parents, and his own parents. At trial, the child's total damages were determined to be $100,000. The playmate's parents were determined to be 20% at fault because they had failed to adequately supervise her. The driver was found to be 50% at fault. The child's own parents were determined to be 30% at fault for failure to adequately supervise him. The court has adopted the pure comparative negligence doctrine, with joint and several liability, in place of the common law rules relating to plaintiff's fault. In addition, the common law doctrines relating to intra-family liability have been abrogated. How much, if anything, is the child's representative entitled to recover from the driver?

$100,000 The issue in this question is the proper application of joint and several liability. Under that doctrine, multiple individuals who were at fault in causing the plaintiff's harm can be liable for not just their own percentage of fault but the entire amount of plaintiff's damages. However, joint and several liability is available only in certain circumstances. For example, joint and several liability applies when defendants act in a manner such that their separate acts combine to harm the plaintiff in one indivisible injury. That is what occurred here: one defendant, the playmate, pushed the child into the street because her parents failed to adequately supervise her, and a second individual, the driver, was also at fault in striking the child with driver's car. Those failings combined to cause the child's harm - being struck by a car in the street - because the actions of no one defendant alone would have produced the harm. Thus, the child (through his parents as representative) can recover his total damages from the driver pursuant to joint and several liability, .

A homeowner who regularly borrowed garden tools from his neighbor went to the neighbor's house to borrow the neighbor's leaf blower. The neighbor was not at home, but the leaf blower was in his unlocked garage with his other garden tools, and so the homeowner took it. Unbeknownst to the homeowner, the neighbor had drained the oil from the leaf blower's motor. The homeowner ran the leaf blower for an hour; the motor was totally destroyed because it had no oil. The value of the leaf blower at the time that the homeowner took it was $300. An identical new leaf blower costs $500. The cost of repairing the motor is $150. A new motor will cost $250. If the neighbor sues the homeowner on a theory of conversion and is successful, what damages can he recover?

$300, but the homeowner will keep the leaf blower. If the plaintiff is successful in a conversion action, the measure of damages is the fair market value of the chattel converted. This value is generally computed as of the time and place of the conversion (NOT the replacement value)

A pedestrian crossed the street at a crosswalk without looking for oncoming traffic. He was struck first by a car and then by a truck. The pedestrian sued both the driver of the car and the driver of the truck for negligence. The jury determined that the pedestrian was 60% at fault, the driver of the car 30%, and the truck driver 10%. The jury also determined that the pedestrian suffered damages of $100,000. The driver of the car is insolvent. In a pure comparative negligence jurisdiction retaining traditional joint liability rules, how much can the pedestrian collect from the driver of the truck?

$40,000. The pedestrian can collect $40,000 from the driver. In a pure comparative negligence jurisdiction, the plaintiff can recover even if he was over 50% at fault. The recovery will be limited to the percentage of damage attributed to the defendant(s), in this case 40%. The jurisdiction retains the rule on joint and several liability. Therefore, each defendant is responsible for the combined liability of all defendants.

A pedestrian brought a negligence action against a motorist who struck him with her car in a shopping mall parking lot. The trier of fact determined that the pedestrian suffered $100,000 in damages and that he was 25% at fault for crossing in front of the motorist's car. The shopping mall was determined to be 30% at fault for allowing bushes to obscure the view of drivers, and the motorist was found to be 45% at fault for striking the pedestrian. The jurisdiction uses a comparative contribution system. How much can the pedestrian recover from the motorist?

$75,000, but the motorist can recover $30,000 contribution from the shopping mall. The pedestrian can recover $75,000 from the motorist, but the motorist can recover $30,000 from the shopping mall in contribution. Under pure comparative negligence, an injured plaintiff may recover damages from a negligent defendant no matter how great his own negligence was. Hence, the pedestrian is entitled to recover $75,000 of his $100,000 in damages because he was 25% at fault. The pedestrian may recover the entire $75,000 from the motorist under joint and several liability rules. Under joint and several liability, when two or more tortious acts combine to proximately cause an indivisible injury to the plaintiff, the plaintiff can recover the entire judgment from any defendant. However, the motorist can recover $30,000 from the shopping mall because the jurisdiction uses a comparative contribution system. Under the rule of contribution, any defendant required to pay more than her share of damages has a claim against the other jointly liable parties for the excess. In jurisdictions that have adopted a comparative contribution system, nonpaying tortfeasors are required to contribute only in proportion to their relative fault. Thus, the motorist can recover $30,000 from the shopping mall because it was found to be 30% at fault. (A) is incorrect because the jurisdiction retains joint and several liability, so the motorist can be required to pay all but 25% of the pedestrian's damages.

Malicious Prosecution

(1) Initiating a criminal proceeding against the plaintiff (2) ending in plaintiff's favor, (3) absence of probable cause for the prosecution, (4) improper purpose (malice), and (5) damages

2) INTENTIONAL TORTS: Assault -

(1) Intentional placement of another (P) in (2) reasonable *apprehension* (anticipation/awareness) of an (3) *Imminent Harmful or Offensive contact* (immediate battery) ---words alone lack immediacy. Has to be menacing conduct. ---Words CAN negate the menacing conduct. EX: D draws hand back as if to hit P and says "if you weren't my best friend I'd smack you." *Apprehension* = must have awareness/anticipation; do NOT have to be afraid. ---EX: tiny 100 pound weakling threatens to hit Francis Ngannou. Ngannou is not afraid, but he apprehends an imminent battery.

IIED: *Causation in Bystander Cases* when D intentionally causes physical harm to a third person and P suffers severe emotional distress because of it, P may recover by showing EITHER IIED elements OR that

(1) P was present when the injury occurred, (2) P is a close relative of the injured person; and (3) D knew facts 1 and 2

*Slander Per Se* - gets the same presumed damages as libel. 4 statements:

(1) statement relating to P's business or profession (2) statement that P committed a serious crime (involves honesty or violence) (3) statement that imputes unchastity to a woman (4) statement that the P suffers from a loathsome disease (leprosy, sexually-transmitted disease)

Two exceptions to negligence per se (class of person, class of risk)

1) more dangerous to follow statute (EX: kid runs into street; better to violate traffic law than hit kid) 2) compliance with statute is impossible or beyond the defendant's control (EX: driver has unexpected heart attack/stroke)

Accountants may be held liable for negligent misrepresentation to noncontracting parties who rely to their detriment on inaccurate financial reports if (something approaching privity):

(1) the accountants were aware of the purposes the statements would be used for; (2) a known third party was intended to rely on the statements; and (3) there was some conduct on the part of the accountants linking them to the third party.

The owner of a self-propelled riding mower started the engine to mow his front lawn when the clutch of the mower suddenly engaged, causing it to lurch forward rapidly and throw him off. By the time the owner caught up with the mower, it had started into the street. A motorist swerved to avoid the mower and struck a tree on the opposite side of the street. An investigation revealed that the sudden shift of the clutch was caused by a defective gear in the transmission. If the motorist brings a negligence action for personal injuries and property damage against the owner, will she prevail?

(A) No, because the owner was so startled by the mower's sudden movement that he was unable to react swiftly enough to prevent the harm. The motorist will not prevail under these facts because the owner does not appear to have breached his duty of care to her. While the owner owed a duty of care to the motorist because she was passing by his property while he was operating his riding mower, the facts do not suggest a breach of duty. He was thrown off by the sudden forward movement of the mower and could not catch up to it before it entered the street. Absent the owner's breach of duty, the motorist cannot recover. [(B) No, because the sudden movement was caused by a defective gear in the mower] is incorrect because the owner may be found liable for negligence in his operation of the mower regardless of what caused the sudden movement. Here, however, nothing suggests that he was negligent in starting the mower or failing to discover or guard against the defect

What type of actual cause describes the guest's food poisoning? The two dishes were kept separate, and the guest took portions from both dishes. Both were found to contain the bacteria at a dangerous level.

*Merged causes* Both parties were negligent, and the harm was caused from the combined contents of both dishes in the guest's stomach after she consumed them. The negligence of both parties was a *substantial factor* in causing the guest's illness

4) INTENTIONAL TORTS: Trespass/Chattel -

1) act that interferes with P's right of possession in a chattel 2) intent 3) causation, and 4) damages - not necessarily to chattel, but at least to a possessory right *Two types of interference* - may be intermeddling (damaging the chattel) or dispossession *Intent required* - Intent to trespass NOT required, just intent to do the act of interference. Mistaken belief that he owns the chattel is no defense.

INTENTIONAL MISREPRESENTATION: A misrepresentation is fraudulent if the maker

(a) knows or believes that the matter is not as he represents it to be, (b) does not have the confidence in the accuracy of his representation that he states or implies, or (c) knows that he does not have the basis for his representation that he states or implies.

Damages - 3 general elements of a personal injury award.

1) past and future medical expenses 2) past and future lost income 3) P's pain and suffering. P has a DUTY TO MITIGATE: - EX: P might have a duty to get a new job to mitigate lost income

PRIMA FACIE CASE FOR PRODUCTS LIABILITY BASED ON STRICT LIABILITY

(i) a *commercial supplier*, (ii) production or sale of a *defective product unreasonably dangerous* to users, (iii) the *defect existed when the product left the defendant's control* ---will be *inferred* if the product move through normal channels of distribution (iv) actual and proximate cause, and ---*Actual cause*: P must show defect existed when it left D's control. If difficult to trace, P may be able to rely on an inference that this type of product failure occurs only as a result of product defect ---> P claiming product defect based on *inadequate warning is entitled to presumption that adequate warning would have been heeded*. ---*Proximate cause*: same as negligence (*must have used product in reasonably foreseeable manner*) (v) damages. ---requires physical injury or property damage. Recovery will be denied if claim is solely for economic loss (the product does not work as well as expected or requires repairs)

Usual NIED requirements

(i) plaintiff must be within the "zone of danger" of physical risk (near miss); and (ii) plaintiff must suffer physical symptoms from the distress.

Two women, Debbie and Jane, both parents of young children, were talking outside of a school one afternoon while they were waiting for their children. Jane asked Debbie for the phone number of the woman she employed in her home to take care of her children. Debbie, knowing that a reliable childcare provider like the woman she has is hard to find, did not want to share the information with Jane because she was afraid Jane would hire the woman away. She instead said, "You don't want her number, we discovered that she was drunk last week when we came home from work." This accusation was false, but it quickly spread throughout the neighborhood. Pamela, the woman employed by Debbie, learns of the accusation. She sues Debbie for defamation and establishes the above facts. Which of the following additional facts must Pamela show in order to succeed? (A) That she suffered special damages by reason of Debbie's statement. (B) That she lost other childcare jobs because of the statement. (C) That the statement was extreme and outrageous because any drinking of alcohol is against Pamela's personal moral beliefs. (D) No additional facts.

(D) No additional facts. SLANDER PER SE: related to her business/profession as a child worker.

Duty Regarding Negligent Infliction of Emotional Distress A duty to avoid negligent infliction of emotional distress may be breached when the defendant creates a foreseeable risk of physical injury to the plaintiff. The plaintiff usually must satisfy two requirements to prevail:

(i) plaintiff must be within the "zone of danger"; and (ii) plaintiff must suffer physical symptoms from the distress.

Neg Infliction of Emotional Distress - TWO TYPES

*1) NEAR MISS*: (a) Negligent conduct leading to near miss because P is *zone of danger* that (b) causes *severe emotional distress resulting in physical injury/problems* (EX: miscarriage/heart attack) *2) CLOSE RELATIVE*: (a) negligent conduct that harms close relative, (b) plaintiff is present and (c) witnesses/perceives the harm

"Attractive Nuisance" Doctrine Most courts impose upon a landowner the duty to exercise ordinary care to avoid a reasonably foreseeable risk of harm to children caused by artificial conditions on his property. Under the general rule, to assess this special duty upon the owner or occupier of land with regard to children on his property, the plaintiff must show the following: (4)

(i) There is a *dangerous artificial condition* present on the land of which the *owner is or should be aware*; (ii) The owner *knows or should know that young persons frequent the vicinity* of this dangerous condition; (iii) The condition is *likely to cause injury because of the child's inability to appreciate the risk*; and (iv) The *expense of remedying the situation is slight compared with the magnitude of the risk*.

"Attractive Nuisance" Doctrine Most courts impose upon a landowner the duty to exercise ordinary care to avoid a reasonably foreseeable risk of harm to children caused by artificial conditions on his property. Under the general rule, to assess this special duty upon the owner or occupier of land with regard to children on his property, the plaintiff must show the following: (4)

(i) There is a dangerous condition present on the land of which the owner is or should be aware; (ii) The owner knows or should know that young persons frequent the vicinity of this dangerous condition; (iii) The condition is likely to cause injury because of the child's inability to appreciate the risk; and (iv) The expense of remedying the situation is slight compared with the magnitude of the risk.

[GOOD RULE PARAGRAPH FOR RES IPSA] This is the doctrine of res ipsa loquitur. However, for the doctrine to apply, plaintiff must show that: (3)

(i) the accident causing his injury is the type that would not normally occur unless someone was negligent; (ii) the negligence was attributable to defendant; and (iii) the injury was not attributable to plaintiff. For the second requirement, plaintiff must establish that this type of accident ordinarily happens because of the negligence of someone in defendant's position. This can often be done by showing that the instrumentality causing the injury was in the exclusive control of the defendant. Res Ipsa is NOT about proving elements of negligence. Instead, it is about what a jury could infer/find. -A jury could infer...that a surgeon was negligent if you wake up from surgery with a scalpel in your gut. -If no one else was there but one surgeon, it raises the inference that the surgeon was responsible. Exclusive control -If there were 8 surgeons, can't infer that the one person was responsible because there's no exclusive control. EX: P walking by, flower pot falls on head from 2nd story apartment. Only one person home, and there's only one balcony. Strong inference. Res Ipsa. EX: If P is walking by a huge hotel with a hundred balconies with dozens of people on balconies, there's no exclusive control, so no res ipsa.

3) Special Situations Where Requirements Not Always Necessary a) Bystander Not in Zone of Danger Seeing Injury to Another Traditionally, a bystander outside the "zone of danger" of physical injury who sees the defendant negligently injuring another could not recover damages for her own distress. A majority of states now allow recovery in these cases as long as:

(i) the plaintiff and the person injured by the defendant are closely related, (ii) the plaintiff was present at the scene of the injury, and (iii) the plaintiff personally observed or perceived the event. Many of these states also drop the requirement of physical symptoms in this situation. Example: Mother sees her child struck by negligently driven automobile on the other side of the street and goes into shock. Most courts would allow recovery.

6) INTENTIONAL TORTS: Intentional Infliction Emotional Distress -

*(1) Intentional or RECKLESS* *(2) Extreme and Outrageous conduct* - exceeds all bounds of decency tolerated in a civilized society. ---mere insults are not considered outrageous conduct *(3) Causing* *(4) Severe Emotional Distress (Damages)* (No actual physical harm required) ---P has to offer some evidence; but there is no particular kind required.

7) INTENTIONAL TORTS: False Imprisonment -

*(1) Intentional* *(2) Act of Confinement (restraint)* ---plausible threats are sufficient: "I will shoot you if you leave" [but not mere moral pressure] ---*omission can be act of restraint if there was a pre-existing duty between the parties*: EX: flight attendants leave disabled person who needs wheelchair on the plane. *(3) in a bounded area with no reasonable means of escape* ---Way out can't be dangerous, disgusting, humiliating, or hidden. EX: P is taking a shower at the gym, D takes his clothes as a prank. *(4) person knows/aware at the time they are being confined OR be harmed by the confinement* Exception - Shopkeeper Priv.

Duties Owed by Bailor Sole benefit of Bailee

*(1) Sole Benefit of Bailee Bailments* If the bailment is for the sole benefit of the bailee (e.g., the bailor gratuitously loans her lawnmower to the bailee), the bailor *need only inform the bailee of known dangerous defects* in the chattel. There is no duty with regard to unknown defects.

3 intervening forces that will not limit D's liability IF FORESEEABLE. [If unforeseeable, then liability is cut off.]

*1) Negligence of a 3P* - sometimes it is foreseeable others will be negligent. D is to blame because it exposes P to risk of 3P negligent. - EX: D negligently blocks sidewalk, forcing P to walk into street who is hit by negligent driver. D is liable. *2) 3P criminal conduct* - EX: D negligently leaves parking garage unlocked. P is attacked in the garage. D is liable for injuries. *3) Act of God* - EX: D worker is on the roof of a building, heard weather reports about high winds coming. Leaves tools on the roof which are blown off and hit P. D is liable.

If exam essay question features person injured by a product, you almost always can argue multiple theories:

*1) Negligence* - failed RPP standard in ---negligent assembly, etc. ---negligent failure to include warnings ---If P can't show exactly what D did wrong, can probably use *res ipsa loquitur* to win negligence claim. *2) UCC Claims* ---implied warranty of merchantability. Could be a breach of warranty if product causes injury. *3) Misrepresentation* ---If there were representations in the package or by the sales person that are connected to the injury *4) Strict Liability* ---products liability is a possibility; but don't assume so. Depends on circumstances.

Common Foreseeable Intervening Forces Intervening forces that are normal responses or reactions to the situation created by defendant's negligent act are almost always foreseeable. The following are common intervening forces that are almost always foreseeable:

*(1) Subsequent Medical Malpractice* The original tortfeasor is usually liable for the aggravation of plaintiff's condition caused by the malpractice of plaintiff's treating physician. *(2) Negligence of Rescuers* Generally, rescuers are viewed as foreseeable intervening forces, and so the original tortfeasor usually is liable for their negligence. *(3) Efforts to Protect Person or Property* Defendant usually is liable for negligent efforts on the part of persons to protect life or property of themselves or third persons endangered by defendant's negligence. *(4) "Reaction" Forces* Where defendant's actions cause another to "react" (e.g., negligently firing a gun at another's feet), liability generally attaches for any harm inflicted by the "reacting" person on another. *(5) Subsequent Disease* The original tortfeasor usually is liable for diseases caused in part by the weakened condition in which defendant has placed the plaintiff by negligently injuring her; e.g., injury caused by defendant weakens plaintiff, making her susceptible to pneumonia. *(6) Subsequent Accident* Where the plaintiff suffers a subsequent injury following her original injury, and the original injury was a substantial factor in causing the second accident, the original tortfeasor is usually liable for damages arising from the second accident. For example, as a result of defendant's negligence, plaintiff's leg is broken. Walking on crutches, plaintiff falls and breaks her other leg.

Duty of Possessor to Those Off the Premises b) Artificial Conditions As a *general rule*, there is also *no duty* owing for artificial conditions. Two major exceptions exist, however.

*(1) Unreasonably Dangerous Conditions* A landowner is liable for damage caused by unreasonably dangerous artificial conditions or structures abutting adjacent land. Example: While one would not be liable for natural collections of ice on the sidewalk, he might be liable for negligently permitting water to drain off his roof and form ice on the sidewalk. *(2) Duty to Protect Passersby* A landowner also has a duty to take due precautions to protect persons passing by from dangerous conditions, e.g., by erecting a barricade to keep people from falling into an excavation at the edge of the property.

Common "always foreseeable" (4) scenario that don't cut off liability.

*(1) subsequent medical malpractice.* Errors by health care providers are foreseeable. - D still liable (as well as the doctor) *(2) negligent rescue*. "danger invite rescues" - law recognizes a certain percentage of rescues will be done negligently *(3) reaction forces* - what happens when people react to negligent situation - EX: you negligently pull alarm, people get trampled. *(4) subsequent accident or disease* - if you negligently made someone more vulnerable to harm and they get harmed, you're liable. EX: you negligently injure someone; have to walk with crutches. They aren't used to crutches and get hurt again. You are liable.

Duties Owed by Bailor Bailments for Hire

*(2) Bailments for Hire* If the bailment is for hire (e.g., the bailor loans her lawnmower to the bailee for a fee), the bailor *owes a duty to inform the bailee of defects known to him, or of which he would have known by the exercise of reasonable diligence*. [akin to duty to invitee on land]

INTENTIONAL TORTS: Trespass to chattels

*1) An intentional act by the defendant that causes* ---The defendant need not have intended to commit a trespass to the chattels, only to do the act that causes interference with chattel *2) an interference with the plaintiff's right of possession in a chattel,* ---smaller harm than conversion. If the damage to the chattel is serious, conversion may be more appropriate *3) resulting in damages* ---The tort typically involves damage to or dispossession of the plaintiff's chattel

INTENTIONAL TORTS: Trespass to land

*1) An intentional act by the defendant that causes* ---Do NOT have to be aware that you are on someone else's land. ---The defendant need not have intended to commit a trespass, only to do the act of entering onto land *2) a physical invasion of* ---"Intangible" invasions do NOT count. Annoying sounds/lights/etc. cannot "trespass" physically. (these are actionable under nuisance) *3) the plaintiff's real property* (including air above and soil below out to a reasonable distance) *The trespass does NOT have to cause harm.* ---However, you might not get anything other than nominal damages without harm. EX: Your horse goes nuts after being scared by a snake, etc. and carries you against your will onto someone's land. NOT a trespass since you didn't have intent. EX: Pick up a rock and throw it onto someone's land. TRESPASS.

INTENTIONAL TORTS: Conversion

*1) An intentional act by the defendant that causes* ---The defendant need not have intended a conversion, only to do the act that constitutes a conversion *2) a serious interference with the plaintiff's right of possession in a chattel* ---The interference with the chattel is so serious as to require the *defendant to pay the full market value of the chattel* ---*in effect, a forced sale of the chattel* EX: take a brief case for a month. ---haven't damaged it, but it is a "serious interference with possessory rights"

Products Liability Based on Strict Liability *ELEMENTS*

*1) D must be a merchant* (routinely deals in goods of this type) ---DOES include commercial lessors (EX: rental car company strictly liable) ---includes EVERYONE in distribution chain. Can sue manufacturer, wholesaler, retailer, etc., privity of contract NOT required. *2) Product is defective* (sometimes stipulated in question) THREE TYPES OF DEFECT: Manufacturing Defect, Design Defect, Information/Warning Defect. ---Go thru elements of particular defect. *3) The defect must have existed when the product left the defendant's control* ---product not altered since leaving D's hands. ---presumption in P's favor: if product moved in ordinary channels of distribution, it is assumed it has not been altered. *4) P was making a foreseeable use of the product at the time of the injury.* ---NOT the same as the intended product use. ---EX: P stands on chair to reach top shelf, chair breaks. D can't argue that P was misusing chair because standing on a chair is foreseeable.

*Implied Assumption of Risk* Implied assumption of risk situations are harder to resolve as, of course, the fact issues are difficult to prove.

*1) Knowledge of Risk* Plaintiff must have known of the risk. Knowledge may be implied where the risk is one that the average person would clearly appreciate, e.g., risk of being hit by a foul ball in a baseball game. *2) Voluntary Assumption* The plaintiff must voluntarily go ahead in the face of the risk. However, plaintiff may not be said to have assumed the risk where there is no available alternative to proceeding in the face of the risk, e.g., the only exit from a building is unsafe.

Effect of Res Ipsa Loquitur

*1) No Directed Verdict for Defendant* The doctrine, where applicable, does not change the burden of proof, nor does it create a presumption of negligence. Where the res ipsa element has been proved, the plaintiff has made a prima facie case and no directed verdict may be given for the defendant. *2) Effect of Defendant's Evidence of Due Care* However, the effect of defendant's evidence that due care was exercised has the same effect in a res ipsa case as in all other cases. If the jury rejects the defendant's evidence and draws the permissible inference of negligence, it will find for the plaintiff. If defendant's evidence overcomes the permissible inference that may be drawn from the res ipsa proof, the jury may find for the defendant. Such a finding for the defendant may result even where defendant rests without offering evidence on the issue if the jury elects not to infer negligence.

Defense of Property: Entry onto Land to Remove Chattel

*1) On Wrongdoer's Land* the owner is privileged to enter onto land and reclaim the chattel at a reasonable time and in a reasonable manner after first making a demand for their return *2) On Land of Innocent Party* Owner may enter and reclaim at a reasonable time in a peaceful manner when landowner has been given notice of the presence of the chattel and refuses to return it. ---Chattel owner's right of recapture supersedes landowner's right to defend property; but chattel owner is liable for any actual property damage *3) On Land Through Owner's Fault* ---NO privilege to enter land. Only can be recovered thru legal process.

This reasonable person has the following characteristics, measured by an objective standard: Physical Characteristics

*1) Physical Characteristics*—Same as Defendant's Notwithstanding application of the objective standard, the "reasonable person" is considered to have the same physical characteristics as the defendant. However, a person is expected to know his physical handicaps and is under a duty to exercise the care of a person with such knowledge; e.g., it may be negligent for an epileptic to drive a car.

PRODUCTS LIABILITY- Bought something, it's broken. 3 theories to sue:

*1) Strict Product Liability* 1) Product defective when it left factory, 2) sold by seller engaged in business of selling product, 3) sold to foreseeable user (includes family of purchaser) who 4) used it in manner it was intended (can sue anyone in chain) OR - Inadequate Warning Defense: Assumption of Risk *2) NEGLIGENCE* - Someone in chain of selling (ie. Retailer, manufacturer, etc) failed to do something they were supposed to do *3) WARRANTY* - There was a Label, Sticker, something in writing promising how product would work.

Defenses to Intentional Torts: Self-defense, defense of others, defense of property

*1) The defendant must reasonably believe that a tort is being or about to be committed against himself, a third person, or his property.* ---even if D is wrong, won't be liable if D had a reasonable belief the threat was genuine. *2) Timing must be reasonable - the threat has to be in progress or imminent.* ---NO PREEMPTIVE ACTION (too soon) ---NO retaliation/revenge (too late) *3) Only reasonable force may be used; must be proportionate to circumstances* ---Deadly force is permitted if reasonably believed to be necessary to prevent serious bodily injury ---Deadly force is never permitted to defend only property. EX: no "spring gun" traps to protect property ---*Majority rule for SELF-DEFENSE*: No requirement of retreat

STRICT LIABILITY - Def liable if engaged in 2 activities:

*1) Wild animals* - Injury is from Animal itself or Because it's a Wild animal! (Fear) - non-domesticated animal. *Can domestic animal become wild?* o Yes, if it has "known dangerous propensities" (EX: devil dog) *2) Abnormally Dangerous Activity* - Blasting, Explosives, Dangerous, Hazardous, etc. Chemicals ---ELEMENTS: (1) foreseeable risk of serious harm even when reasonable care exercised, and (2) not common in the community Defense : only ASSUMPTION OF RISK

Defenses to Defamation

*1) consent*; express or implied *2) truth*: D can show that the "defamatory" statement is true which defeats P's claim. ---BOP on D since it is an affirmative defense; ---when the constitutional requirement of proof of falsity does not apply because BOP on P to prove falsity *3) privilege* *a. Absolute privilege* for statements in judicial, legislative, or executive proceedings; OR to spouse in marriage. *b. Qualified privilege* for matters in the interest of the publisher and/or the recipient (may be lost if the statement is outside the scope of the privilege or made with actual malice) ---arises ONLY when there is a public interest in encouraging candor. ---decided on a case-by-case basis. EXAMPLES OF QUALIFIED PRIVILEGES: *References and recommendations* - EX: former employer, previous landlord, etc. ---POLICY: to encourage candor in recommendations, communication has a qualified privilege QUALIFICATIONS OF PRIVILEGE: *1) must have reasonable/good faith basis for statement*. ---can't tell lies about person in recommendation; but honest mistakes are okay. *2) must confine yourself to relevant matters*

*TWO TYPES OF CONSENT:*

*1) express* - *2) implied (apparent or implied by law)* - ---*social custom and usage* - if a P goes to a place or chooses to engage in conduct where certain invasions are routine, then the law treats the P as having consented. EX: team sports, mosh pit, crowded subway. ---*individualized (body language consent)* - a D's reasonable interpretation of the P's objective conduct and surrounding circumstances. ---EX: dating situations where body language & all circumstances indicate openness to a kiss, etc. is reasonable Consent implied by law - when action is necessary to save a person's life or some other important interest in person or property Exception: fraud or duress eliminates consent.

4 exam favorite examples of "outrageous" conduct

*1) repetitive behavior/perpetual harassment.* ---EX: constant debt collection calls at all hours/harassment every night for a month *2) Defendant is common carrier (transportation company) OR innkeeper (hotel)* ---have a duty to treat customers with respect. So if they deliberately do something with the goal of distressing you, it is treated as outrageous. ---EX: hotel clerk says "I can't check you into your room because you are hideously ugly" (usually insults alone are not enough for IIED, but because common carrier/innkeeper, it is enough) *3) P is a member of a fragile class: young children, elderly, pregnant women (and D knows she is pregnant)* *4) P knows of an unique D sensitivity and intentionally targets it* ---EX: knows of a phobia and intentionally exploits it

Group Defamation

1) if the defamatory statement refers to ALL members of a SMALL group, each member may establish that the statement is "of and concerning" him by alleging that he is a group member (i.e. everyone wins!) 2) if a LARGE group, NO member can prove the statement is "of and concerning" him (i.e. no one wins) 3) if the statement only refers to SOME members of a SMALL group, P can recover if a reasonable person would view the statement as referring to the P

4 scenarios the law deems almost always foreseeable. Rarely if ever limit the liability of D.

*1) subsequent medical malpractice* - EX: D negligently breaks P's leg. Doctor does bad job, P gets infection. D liable for infection damages also. *2) negligent rescue* - EX: D negligently breaks P's leg. Rescuer makes leg worse. D liable for additional damages from rescuer. *3) reaction forces* - other person react to D's negligence, causing harm to P. - EX: D shoots gun in the air, causing crowd to panic knocking down P who is injured. D is liable. *4) subsequent diseases/accidents* - EX: D negligently breaks P's leg. P slips and falls on crutches, breaks arms. D is liable for broken arm.

STRICT LIABILITY: ANIMALS

*1. ANIMALS* - STRICT LIABILITY for: *A) the owner of a wild animal* *B) an abnormally dangerous domestic animal* for injuries caused by the animal. SL if you have knowledge of animal's "vicious propensities" [NOT characteristic of species]. EX: dog that has previously bitten humans. ---does NOT cover trespassers on animal owner's land. ---Injury caused by the normally dangerous characteristics of domestic animals (bulls or honeybees) does NOT create SL

Defamation: Common Law Elements

*1. Defamatory statement by the D* ---Hurts the reputation of the P. Lower the estimation of the P in the community. ---Purports to be factual about the P. Susceptible to T/F analysis. ---General opinions aren't defamatory, unless they imply an underlying fact. EX: "In my opinion, you'd be crazy to let that attorney handle your money." Implies he is a thief. ---Mere name calling is not enough; doesn't have factual content. *2. The defamatory language must be "of and concerning" the plaintiff* (i.e., it must identify the plaintiff to a reasonable reader, listener, or viewer); ---doesn't have to use P's name. ---P must be alive at the time the statement is made. *3. Publication of the defamatory language by the defendant to third person*; and ---After all, reputation is about what 3Ps think about the P. So if D only makes defamatory statement to P, and no one else hears, there's no liability. ---Must share the statement with at least one other person besides the P. ---Negligent publication is sufficient; doesn't have to be intentional. EX: person leaves intercom on, and another person hears defamatory statements. ---*REPUBLISHER*: Each time a person "passes on" the defamatory statement to another, that is another instance of defamation by each republisher. *4. Damage to the reputation of the plaintiff.* ---Not always a required proof element. Sometimes elements are PRESUMED. Depends on whether libel or slander. *Libel & Slander Per Se* - PRESUMED damages. No need to prove. But, the more you prove the more you get. But no directed verdict will be entered against you. *Slander* - NO PRESUMED DAMAGES. Must prove damages. ECONOMIC HARM. Lost a job, lost revenue because of reputational loss. ---Hurt feelings do NOT count.

Tort Immunities

*1. Intra-family tort immunities*—the rule that one family member could not sue another in tort for personal injury is abolished in most states, except that children generally cannot sue their parents for their exercise of parental supervision *2. Governmental immunities*—governments generally have waived their sovereign immunity from suit for ministerial acts (acts performed at the operational level that do not require the exercise of judgment) but have not waived immunity for discretionary acts (acts taking place at the planning or decisionmaking level) LOCAL govts: About half of the states have abolished municipal immunity

Invasion of Privacy—Four Kinds of Wrongs in the Common Law (IPAF - invading privacy ain't fair)

*1. Intrusion on the plaintiff's affairs or seclusion* *2. Public disclosure of private facts* *3. Appropriation* of the plaintiff's picture or name for commercial purposes *4. FALSE LIGHT* *COMMON ASPECTS TO ALL PRIVACY TORTS:* *Causation* - privacy invasion must be proximately caused by D's conduct *Proof of Special Damages Unnecessary* - P need not plead and prove special damages. Emotional distress is sufficient *Defenses* - consent, privilege; --->but NOT truth or inadvertence or good faith *Personal rights* - do NOT extend to family members or survive the death of the P. ---Not assignable. ---Not applicable to corporations

This reasonable person has the following characteristics, measured by an objective standard: Mental Ability?

*2) Average Mental Ability* Defendant must act as would a person with average mental ability. Unlike the rule as to physical characteristics, individual mental handicaps are not considered; i.e., low IQ is no excuse. Likewise, insanity is no defense, and the defendant is held to the standard of a reasonable person under the circumstances.

STRICT LIABILITY: ABNORMALLY DANGEROUS ACTIVITIES

*2. Abnormally dangerous activities*—SL for an activity that *A) creates a foreseeable risk of serious harm* *B) even when reasonable care is exercised by all actors* ---activity cannot be made safe even through the exercise ordinary care. *C) uncommon in the area* *RECURRING ABNORMALLY DANGEROUS ACTIVITIES ON EXAM* 1) explosives/blasting 2) handling/transport of highly dangerous chemical/biological material - such as fumigating/crop dusting 3) radiation/nuclear energy. D's liability extends only to foreseeable plaintiffs ---Harm must result from the kind of danger to be anticipated from the dangerous activity or animal (including harm caused by fleeing from the danger) ---Hence, strict liability does NOT apply when the injury is caused by something other than the dangerous aspect of the activity (EX: dynamite truck blows a tire and hits a pedestrian but doesn't explode)

This reasonable person has the following characteristics, measured by an objective standard: Level of Knowledge?

*3) Same Knowledge as Average Member of Community* Defendant is deemed to have knowledge of things known by the average member of the community, e.g., that fire is hot. Again, the individual shortcomings of the particular defendant are not considered. On the other hand, *a defendant with knowledge superior to that of the average person is required to use that knowledge.*

*A) Manufacturing Defect*

*A) Manufacturing Defect* - product differs from the others on the assembly line in a dangerous way that consumers would not expect. ---Departs from intended design. ---SL: good quality control is NOT a defense.

Defamation: Matter of public concern require 2 additional elements Public concern = Newsworthy Examples of public concern: -judicial integrity, politics, celebrities, etc.

*Matters of public concern include 2 extra elements* in a defamation suit: *5. P must prove statement is false* ---shifts BOP to P. ---proving a negative is difficult. *6. P must prove statement was made with fault/culpability* ---Statement was not made in good faith. ---Degree of fault you have to show depends on who you are (public figure or private) *Public figure* - P must prove D acted with *actual malice*: knew it was false, or with reckless disregard to truth or falsity. *Private figure*- P ONLY has to prove statement was made NEGLIGENTLY. Failure to exercise reasonable care to confirm its truth. ---AND actual damages

*B) Design Defect*

*B) Design Defect* - the risks associated with the product's design outweigh the utility of the design. *Design defect is proved by P showing* ---(i) there is a hypothetical alternative design that was safer. ---(ii) prove that alternative is practical. Won't make harder to use or interfere with primary purpose. ---(iii) cost effective/economically feasible.

What type of actual cause describes the guest's food poisoning? The two dishes were kept separate, and the guest took portions from both. Only one ofthe dishes was found to contain the bacteria, and the volunteer who prepared that dish was identified.

*But for causation* You have one negligent volunteer and a clear case of actual cause.

*C) Information/Warning Defect*

*C) Information/Warning Defect* ---If a product has hidden risks not apparent to users, the product is defective unless it has *adequate* warnings and instructions. Factors in assessing warning adequacy: 1) should be prominent: calculated to get user's attention 2) warning should be comprehensible. EX: might need to be in multiple languages; use icons/pictures for those who can't read. 3) warning may have to provide info about mitigating risk.

INTENTIONAL TORTS: CAUSATION [APPLIES TO ALL]

*Causation* - result must have been legally caused by defendant's act or something set in motion by him. ---Causation is satisfied if D's conduct was a SUBSTANTIAL FACTOR* in bringing about injury ---Most often, causation will not be at issue in analysis of intentional tort because it is usually obvious that the D's conduct was a substantial factor in causing the harm

Vicarious Liability: *Tavernkeepers*

*Common law:* no liability *Dramshop acts:* Liability for injuries to third parties caused by intoxicated vendees ---enacted in many states

STRICT LIABILITY: *Defenses*

*Contributory negligence states*: no defense if P has failed to realize the danger or guard against it. ---It is a defense if P knew of the danger and his unreasonable conduct was the very cause of the harm from wild animal or abnormally dangerous activity ---Assumption of risk is a good defense *Comparative negligence states* - applied to SL cases

99% of the time when the call of the question involves a directed verdict motion, the answer will be....

*DENY!* Typically an issue with *breach*, and the answer will say something about how it should go to the jury who can decide whether a breach occurred. To "Grant" a directed verdict, the fact pattern must very clearly establish every element of the prima facie case.

Vicarious Liability 2. *Employer-employee (respondeat superior)*

*Employer-employee (respondeat superior)*—the employer can be held liable as a second defendant for torts of an employee that occur within the *scope of the employment relationship at the time of the incident*. ---*Detour*: when you are engaged in a minor departure, that is a "mere detour" and you are still within the SOE. EX:UPS driver goes two blocks off route to get lunch. ---*Frolic*: major departure from the work, and OUTSIDE the SOE. EX: UPS driver takes truck 50 miles out of town to see ex. *Intentional torts are OUTSIDE the SOE UNLESS the employee is acting to further employer's purposes*; EX: ---1) force is authorized (bouncer) ---2) friction is generated by employment (bill collector) ---3) employee is furthering business of employer (removing rowdy customer)

Assumption of the Risk - Express

*Express* - words that accept risk. EX: "I'll take my chances" - completely bars recovery.

Intentional Torts: INTENT

*INTENT* - Can be *SPECIFIC* (conscious *purpose* to bring about result) or *GENERAL* (know with *substantial certainty* consequences with result) *1) P's hypersensitivity should be disregarded* ---only reasonable sensitivity *2) no incapacity defenses* ---EX: 10-year-old locks you in a closet, you can sue for false imprisonment. ---EX: if an answer choice is "D lacked legal capacity" eliminate it *3) transferred intent* - if a D desires to commit a legally forbidden consequence against one person, but a different forbidden consequence, or different victim is affected, the D is STILL liable. ---EX: throw a rock trying to hit John, but nearly misses Pat. Liable to Pat for assault thru transferred intent. ---*from tort to tort, and person to person* ---DOES NOT APPLY TO IIED.

MISREPRESENTATION - Fraud and Deceit Intentional vs. Negligent

*Intentional* - intentional statement of material fact that person knew or should have known (scienter) could be false to induce detrimental reliance causing damage. *Negligent* - Look for special relationship

What six elements for the basis of a prima facie case for fraudulent misrepresentation?

*MS. JARID*'s fraudulent misrepresentation Misrepresentation, Scienter, Justifiable and Actual Reliance, Intent to Induce reliance, Damages 1) *Misrepresentation* - material past or present fact 2) *Scienter* - knowledge of falsity or reckless disregard of the truth 3) *Justifiable* reliance - P can justifiably rely if not obviously false and not clearly just an opinion 4) *Actual reliance* - causation, must show in fact relied 5) *Intent* to induce reliance 6) *Damages* flowing from reliance - *must be pecuniary*

Three Torts: Three related tort actions protect the P's interest in not being subjected to unwarranted judicial proceedings:

1) malicious prosecution 2) wrongful institution of civil proceedings 3) abuse of process WAM Wrongful Abuse is Malicious

Affirmative Defenses Against Strict Liability

*Modern Rule: Comparative Fault* ---default rule on exam ---assign percentage fault and reduce the P's recovery accordingly. *Traditional Rule* - if a *P knowingly encountered a dangerous situation* that would normally lead to strict liability that P is barred from recovery. ---EX: you know your neighbor has a tiger, you go over to cage and get mauled, you are barred from recovery because you knowingly got close to tiger. ---EX: you know blasting is taking place and want to watch and get hit with debris *ONLY USE ON EXAM IF QUESTION INDICATES "TRADITIONAL RULE" IS FOLLOWED*

DAMAGES. General rule:

*P can recover ALL damages BUT NOT PUNITIVES UNLESS AN INTENTIONAL TORT (OR RECKLESSNESS)* -economic damages medical expenses & lost earnings -non-economic damages like pain and suffering, emotional distress cause by physical injury

Malicious Prosecution Elements

*PlACID* Plaintiff's favor, Absence of probable cause; Criminal proceedings; Improper purpose; Damages 1) proceedings terminated on the merits in *Plaintiff's favor* 2) *Absence of probable cause* to institute proceedings 3) D instituted *criminal proceedings* against him 4) D was motivated primarily by some *Improper purpose* (malice) 5) *damages* - harm to reputation, humiliation, mental suffering, or any other injury that may give rise to damages in a civil action. NOTE: One can infer improper purpose from lack of probable cause, but not vice versa

Vicarious Liability 3. *Principal-independent contractor*

*Principal-independent contractor*—the *general rule* is that a principal is *NOT vicariously liable* for the torts of an independent contractor *EXCEPTIONS* a. The independent contractor is engaged in *inherently dangerous activities* b. The *principal's duty cannot be delegated because of public policy considerations* (EX: nondelegable duty to keep business premises safe for customers) c. *IC's negligence hurts customer while working on principal's property* ---EX: mall owner hires IC to maintain gardens, they accidentally injure customer. Mall owner liable.

NUISANCE: Private vs. Public

*Private Nuisance* - Unreasonable Interference with the USE and ENJOYMENT of someone's property. (Must be unreasonable to an objective person) *Public Nuisance* - Entire Community ---brought by Public official, look for Special Damages

Depending on who the person is, there could be a more specific level of care.

*Professional* - someone with higher education or training. Duty - to act like other professionals with the same training and background in that community. EX: Houston cardiologists need to act like other Houston cardiologists. *Child* - duty - like other children of the same age and maturity EX: 9 year olds have to act like other 9 year olds

Defenses to Intentional Torts: *Necessity* *property torts* ONLY (trespass to land/chattels, conversion)

*Public necessity* -Defendant commits property tort in the midst of an emergency to protect community as a whole. ---an absolute defense *Private necessity* - defendant is acting to protect his own interests. ---the defense is qualified (the defendant must pay for any damage caused) ---but since privileged to enter, NOT liable for nominal/punitive damages. EX: D takes shelter in garage during hurricane. Doesn't damage anything. Not liable. *This privilege trumps a property owner's right to defend his property* ---Privilege to remain as long as the emergency remains. A "right of sanctuary" ---EX: people fleeing terrible storm enter someone's home. Homeowner can't throw them out. Must tolerate presence on property until end of emergency.

Satisfaction and Release

*Satisfaction*: recovery of a full payment is "satisfaction." Only one satisfaction allowed. Until there is satisfaction, one may proceed against all liable parties. *Release*: A release of a tortfeasor does not discharge other tortfeasors unless it is expressly provided for in the release agreement.

Duty toward others regarding dangerous *conditions* on your land?

*Undiscovered trespasser*: no duty *Discovered trespasser*: man-made death traps *Licensee*: concealed dangerous conditions known to landowner (natural or manmade, any danger not just death traps) - NO liability for obvious danger; must be concealed for liability *Invitee*: duty to inspect for dangerous conditions (reasonable care)

What type of actual cause describes the guest's food poisoning? The two dishes were kept separate, and the guest took portions from only one dish, but the dishes looked identical and the guest can't remember which dish she took portions from. Both dishes were found to contain the bacteria.

*Unknown cause* Both volunteers were negligent, but the guest isn't able to show whose negligence caused her illness; so the burden on the actual cause element shifts to the volunteers. Summers v. Tice

Certain Risks May Not Be Assumed Because of public policy considerations, the courts uniformly hold that some risks may not be assumed. These include:

*a) Common carriers and public utilities* are not permitted to limit their liability for personal injury by a disclaimer on, e.g., a ticket, a posted sign, etc. *b) When a statute is enacted to protect a class*, members of that class will not be deemed to have assumed any risk. Example: When a statute imposes safety regulations on an employer, the employee is held not to have assumed the risk where the statute is violated. *c) Risks will not be assumed in situations involving fraud, force, or an emergency.* Thus, for example, one could take action to save his person or property without assuming a risk unless his actions involve an unreasonable risk out of proportion to the value of those rights.

Common Imputed Contributory Negligence Fact Situations The following situations should be noted for bar examination purposes:

*a) Employer and Employee - IMPUTED if SOE* The contributory negligence of the employee or agent acting within the scope of employment will be imputed to the employer or principal when the latter is a plaintiff suing a third person. *b) Partners and Joint Venturers - IMPUTED* The contributory negligence of one partner or joint venturer will be imputed to the other when the other is a plaintiff suing a third person. *c) Husband and Wife - not imputed* *d) Parent and Child* The contributory negligence of the parent or guardian is *not imputed* to the child, nor is the contributory negligence of the child imputed to the parent in actions against a third party. *e) Automobile Owner and Driver - NOT imputed*

Last Clear Chance 1) "Helpless" vs. "Inattentive" Peril Many cases distinguish between "helpless" and "inattentive" peril situations in applying last clear chance rules.

*a) Helpless Peril* Helpless peril exists where plaintiff, through his contributory negligence, puts himself in a position of actual peril from which he cannot extricate himself. In many states, defendant is liable under these circumstances if she had either actual knowledge of plaintiff's predicament or if she should have known of plaintiff's predicament. Other states require actual knowledge. *b) Inattentive Peril* Inattentive peril exists where plaintiff, through his own negligence, is in a position of actual peril from which he could extricate himself if he were attentive. Almost all courts require actual knowledge of plaintiff's predicament on defendant's part.

Effect of Comparative Negligence on Other Doctrines 2) Assumption of Risk

*a) Implied Assumption of Risk* Most comparative negligence jurisdictions have abolished entirely the defense of implied assumption of risk. In these jurisdictions, traditional assumption of risk situations must be broken down into two categories: (1) When the defendant has only a limited duty to the plaintiff because of plaintiff's knowledge of the risks (e.g., being hit by a foul ball at a baseball game), a court may protect the defendant simply by holding that the defendant did not breach his limited duty of care. The defendant will still be liable for reckless conduct. (2) More common is the situation that is a variant of contributory negligence, in that defendant's initial breach of duty to plaintiff is superseded by plaintiff's assumption of a risk (e.g., builder is negligent in not barricading torn-up sidewalk, but pedestrian chooses to use it despite availability of reasonable alternate route). Here, the reasonableness of plaintiff's conduct is relevant: If the plaintiff has behaved unreasonably, plaintiff is contributorily negligent and damages will be apportioned under the state's comparative negligence statute. *b) Express Assumption of Risk* Most comparative negligence jurisdictions retain the defense of express assumption of risk.

Role of Contract in Creating Duty

*a) Nonfeasance—No Duty* In general, for mere nonfeasance, there is no tort duty of care, regardless of whether the defendant promises to undertake action gratuitously or for consideration. Liability for breach of contract extends only to parties in privity. *b) Misfeasance—Due Care Required* However, for misfeasance, failure to perform with due care contractual obligations owed to one may give rise to violation of a legal duty. Example: Pursuant to a contract with the building owner, Defendant inspected and repaired the elevator, and did so carelessly. The elevator operator is injured as a result. Defendant is liable to the operator.

When Statutory Standard Applicable - Negligence Per Se The precise standard of care in a common law negligence case may be established by proving the applicability to that case of a statute providing for criminal penalties (including fines). If this is done, a clearly stated specific duty imposed by the statute will replace the more general common law duty of due care. In proving the availability of the statutory standard, plaintiff must show the following:

*a) Plaintiff Within Protected Class* - The plaintiff must show that she is in the class intended to be protected by the statute. - Example: A statute requiring a landowner to keep a building in safe condition is meant to protect only those rightfully on the premises and not trespassers. *b) Particular Harm to Be Avoided* The plaintiff must show that the statute was designed to prevent the type of harm that the plaintiff suffered. Example: Violation of a Sunday closing law is not evidence of negligence in the case of an accident in a store on Sunday.

Res Ipsa Loquitur The circumstantial evidence doctrine of res ipsa loquitur ("the thing speaks for itself") deals with those situations where the fact that a particular injury occurred may itself establish or tend to establish a breach of duty owed. Where the facts are such as to strongly indicate that plaintiff's injuries resulted from defendant's negligence, the trier of fact may be permitted to infer defendant's liability. *Res ipsa loquitur requires the plaintiff to show the following:*

*a. Inference of Negligence* Plaintiff must establish that the accident causing his *injury is the type that would not normally occur unless someone was negligent.* Example: A windowpane fell from a second story window in Defendant's building, landing on Plaintiff. Res ipsa loquitur may apply. *b. Negligence Attributable to Defendant* Plaintiff must establish evidence connecting defendant with the negligence in order to support a finding of liability, i.e., evidence that this type of accident ordinarily happens because of the negligence of someone in defendant's position. This requirement often can be satisfied by showing that the instrumentality that caused the injury was in the *exclusive control* of defendant, although actual possession of the instrumentality is not necessary.

Res ipsa loquitur requires the plaintiff to show the following:

*a. Inference of Negligence* Plaintiff must establish that the accident causing his injury is the type that would not normally occur unless someone was negligent. Example: A windowpane fell from a second story window in Defendant's building, landing on Plaintiff. Res ipsa loquitur may apply. *b. Negligence Attributable to Defendant* Plaintiff must establish evidence connecting defendant with the negligence in order to support a finding of liability, i.e., evidence that this type of accident ordinarily happens because of the negligence of someone in defendant's position. This requirement often can be satisfied by showing that the instrumentality that caused the injury was in the *exclusive control* of defendant, although actual possession of the instrumentality is not necessary.

Custom evidence [often relates to whether a party breached the standard of duty]

*always admissible, but never conclusive* (so don't expect a directed verdict). either the P is trying to show: 1) everyone in the world except the D followed a particular precaution, OR 2) D tries to argue that no one follows the precaution the P says D should have observed. Equates reasonableness with typicality. But even if everyone/no one does or doesn't do it, the court could still rule against the supposed "custom" and find D negligent.

A *private nuisance* is a

*substantial, unreasonable interference with another person's use or enjoyment of her property* that he actually possesses or to which he as a right of immediate possession ---Many cases involve "inconsistent land use": neighbors who shouldn't be neighbors. ---EX: polluting factory covers homeowner with soot. "Intent" only means they know it is happening, not that harming homeowner is the object of the action. ---EX: spite actions: super bright lights that shine into neighbor's windows at night ---EX: inconsideration: hosting loud parties every night. MUST BE A SUBSTANTIAL INTERFERENCE - offensive, inconvenient, or annoying to average person in the community. UNREASONABLE INTERFERENCE: severity of the inflicted injury must outweigh the utility of defendant's conduct. Everyone is entitled to use his own land in a reasonable way

One of the effects of res ipsa loquitur is that no directed verdict may be given for the defendant, because

*when the res ipsa element has been proved, the plaintiff has made a prima facie case for negligence.* The doctrine, however, does NOT switch the burden of proof to the defendant and does NOT create a presumption of negligence. Furthermore, it does NOT require the defendant to present evidence of due care in rebuttal. If the jury elects not to infer negligence, it may find for the defendant even if the defendant presents no evidence on that issue.

Invasion of Privacy—Four Kinds of Wrongs in the Common Law *3. FALSE LIGHT*

*widespread dissemination of a material falsehood* about P that would be *highly offensive to the average person*. The publication of facts about the plaintiff putting her in a false light in the public eye in a way that would be highly offensive to a reasonable person *First Amendment limitation: Actual malice* must be shown if the publication is in the public interest Exam Tip: very same statement can be both defamatory and false light invasion of privacy. Difference? ---Defamation: recover for economic damages ---False Light: recover for emotional and dignitary damages relating to feeling shame. However, some statements are false light but not defamatory. ---EX: "Bob's a Catholic" when that isn't true. It's not a defamatory statement, but it puts Bob in a false light Tip: A lot of false light claims are the result of accidents. EX: erroneous caption in newspaper. *Defenses—consent and absolute or qualified privileges*

Invasion of Privacy—Four Kinds of Wrongs in the Common Law *4. Public disclosure of private facts* about the plaintiff

*widespread dissemination of confidential information about the plaintiff that would be highly offensive to a reasonable person* ---Public disclosure requires publicity, NOT just publication to a few people ---Liability may attach even though the statement is TRUE EX: doctor receptionist mails your medical records to everyone at your place of work Tips: ---newsworthiness exception. Investigative journalists job is to publicly disclose private facts. Tabloids, etc. ---facts must be truly confidential *Defenses—consent and absolute or qualified privileges*

Defamation: Common Law Elements in Brief

1) defamatory language 2) of or concerning P 3) publication to a 3P 4) damage to P's reputation (only for regular slander; damages PRESUMED in libel & slander per se) If public concern, Constitution requires 2 more: 5) falsity of the defamatory language (BOP shifted to P) 6) fault on the part of the D ---Private figure: negligence or more; AND actual damages ---Public figure: actual malice: knowledge of falsity or reckless indifference to truth or falsity

Duties of Vendor of Realty The vendor, at the time of transfer of possession to the vendee, has the duty to: (3)

1) disclose concealed, unreasonably dangerous conditions of which the vendor knows or has reason to know, and 2) of which he knows the vendee is ignorant and 3) is not likely to discover on reasonable inspection. The vendor's responsibility continues until the vendee should have, in the exercise of reasonable care in inspection and maintenance, discovered and remedied the defect.

Three types of duties on bar exam:

1) duty to foreseeable plaintiffs to follow reasonable case 2) duty by statute (negligence per se - class of person class of risk) 3) relationship between parties (rescuer/rescuee; landowner/tenant; landowner/invitee; parent/child)

Duty of Lessor to Lessee The lessor is obligated to give warning to the lessee of: (3)

1) existing defects in the premises 2) of which the lessor is aware, or has reason to know, and 3) which he knows the lessee is not likely to discover on reasonable inspection.

Elements of §1983 Civil Rights COA

1) "Every Person" Individuals (so long s they act under color of state law) City, county and other local governmental entities (so long as the deprivation of rights is caused by a custom or policy of the governmental entity) Term does NOT include the state itself (11th amendment) 2) Acting under "color" of law 3) Deprives another of rights, privileges and immunities secured by: US Constitution OR Federal law

Three different scenarios of NIED?

1) *near miss* - something is coming your way, and it barely miss you. (usual scenario) Special Situations Where Requirements Not Always Necessary: 2) Bystander at the sees *witness a close family member injured* 3) *business relationship* - EX: funeral home mishandles the corpse.

Defenses to Intentional Torts

1) Consent - must have capacity; must not exceed consent 2) Self Defense- Reasonable under circumstances,,only use equal amt. of force 3) Defense of Others 4) Defense of Property- Never use deadly force 5) Necessity- Private/Public

STRICT LIABILITY PRIMA FACIE CASE [APPLY TO: ANIMALS & ABNORMALLY DANGEROUS ACTIVITIES]

1) DUTY: the nature of the D's activity imposes an *absolute duty to make safe* 2) CAUSATION: the dangerous aspect of the activity was the *actual and proximate cause of the P's injury* 3) DAMAGES: the P suffered damage to person or property REMEMBER, even if the D used reasonable care, that is NOT a defense. However, strict liability generally is not imposed in favor of undiscovered trespassers against landowners in the absence of negligence, such as when the landowner knows that the trespassers are on the land and fails to warn them of the animal.

Intentional Interference with Business Relations

1) Existence of a valid contractual relationship between the P and a 3P or expectancy 2) D's knowledge of the contractual relationship or expectancy 3) Intentional (usually improper in itself) interference by D that induces a breach or termination of the relationship (or loss of value) or expectancy 4) Damage to P - P must prove actual damage from the interference or expectancy ---may also recover mental distress and punitive in appropriate cases

INVASION OF PRIVACY (4 types)

1) False Light - Not true 2) Appropriation - 3) Public Disclosure of Private Facts - 4) Intrusion Upon Seclusion -

For fraudulent misrepresentation, how do you measure damages?

1) expectation interest (minority: out-of-pocket) 2) consequential damages, AND 3) punitive (if D intended to cause harm) [no nominal damages] MUST prove damages as part of prima facie case. compare: negligent misrepresentation is limited to out-of-pocket and consequentials;

1) INTENTIONAL TORTS: Battery -

1) Intentional 2) Harmful or Offensive Contact 3) to P's person or anything connected to that person ---EX: person grabs umbrella out of your hand. ---EX: P riding horse; D slaps horse on the ass. That's a battery of P because horse is connected to P. Offensive = unpermitted by person of normal sensitivity Contact can be delayed: poisoning someone's food.

Nonrecoverable Items Certain items are not recoverable as damages in negligence actions. These include:

1) Interest from date of damage in personal injury action; and 2) Attorneys' fees.

Elements of Negligent Misrepresentation

1) Misrepresentation of material fact made by D in a business or professional capacity 2) D owed duty to communicate truthful information to particular plaintiff 3) D Breached duty by failing to exercise due care 4) Causation (actual and proximate - actual reliance) 5) P's justifiable reliance on misrepresentation 6) Damages *Negligence elements PLUS misrep and justifiable reliance* *Intentional/Fraudulent Misrepresentation MINUS Scienter and Intent to Induce*

Other intervening forces

1) Negligent conduct by 3Ps 2) intentional torts or crimes by 3Ps 3) acts of God have to make a judgment call on foreseeability; but shouldn't be a close call. Remember, does the D have to be psychic to have foreseen?

Types of Comparative Negligence

1) Partial 2) Pure

A bystander who witnesses the defendant negligently injuring another can recover for *negligent infliction of emotional distress* in most states by showing:

1) a *close relationship* between the bystander and the person injured, 2) the *bystander's presence* at the scene of the injury, and 3) the *bystander's observation or perception of the event*.

What are superseding causes?

1) act of God 2) intentional torts 3) criminal acts If it's not one of these three, it's foreseeable. Law specifically says that all negligent acts are foreseeable and do not cut off liability. Thus, default answer, P is liable for EVERYTHING. Unless Act of God, intentional tort or criminal act. Don't infer or assume a supervening act. It will be obvious.

3) INTENTIONAL TORTS: Trespass to Land

1) physical invasion of P's real property 2) intent - only to enter into land (doesn't have to know it is owned by another) 3) causation *Physical invasion* - may be by a person or object (throwing a baseball onto P's land). ---If intangible matter (vibrations/odors) use NUISANCE *Real Property* - not just the surface, but also airspace and subterranean space for a reasonable distance *Potential plaintiffs* - anyone in actual or constructive possession of the land may maintain an action *No damages required*

NO GENERAL DUTY TO DISCLOSE No general duty to disclose, unless the defendant: (3)

1) stands in a fiduciary relationship to the P 2) is selling real property and knows the P is unaware of, and cannot reasonably discover, material info about the transaction, OR 3) has spoken and her utterance deceives the P. Physical concealment of a material fact may also constitute a misrepresentation

Products Liability: Misrepresentation of Fact Seller is liable for misrepresentations of fact concerning a product where: (2)

1) the statement was of a material fact concerning quality or uses of goods (mere puffery insufficient) and 2) the seller intended to induce reliance by the buyer 3) justifiable reliance 4) causation (actual shown by reliance)s 5) damages 6) defenses: assumption of risk is NOT a defense if plaintiff is entitled to rely on teh representation. Contributory negligence is the same as in SL, unless D committed intentional misrepresentation

Negligence per se requires (5 elements)

1) there was a criminal or other regulatory statute/ordinance/law that imposed a penalty for its breach; 2) the D violated that law 3) the harm caused by the violation was the type of harm that the law was intended to protect against (class of risk) 4) the P as in the class of persons the law was intended to protect; and 5) the violation of the law caused (both prximate and actual) the harm to the P.

Negligence Per Se Can be used to replace set the standard of care (duty element)

1) violation of a statute or ordinance 2) plaintiff is part of protected class of people the statute is trying to protect. 3) the risk or harm suffered is the kind of injury the state is trying to prevent. *CLASS OF PERSON, CLASS OF RISK* By definition, virtually every negligence per se question could also be a common law negligence situation. On essay, do both. But on MC, they will only ask one concept at a time. EX: driving 50 mph in 15mph school zone. Hit kid. Liable under negligence per se? 1) violate law? - yes 2) P part of protected class? - yes 3) injury the type trying to protect? - yes Many of the correct answers on the bar are NOT written like legal elements, but in plain language.

Abuse of Process Elements

1) wrongful use of process 2) for an ulterior purpose 3) some definite act or threat against P to accomplish that purpose

Nuisance Defenses

1. *Legislative Authority* = persuasive, not absolute 2. *Conduct of Others* = others engage in activity too. EX: 10 steel mills polluting river; each is responsible only for the pollution it causes. 3. "*coming to the nuisance*" is not a defense - One who has just moved onto land adjacent to a nuisance may bring a nuisance action; 4. *contributory negligence* NOT a defense unless P's case is based on negligence theory

Products Liability Based on Negligence

1. *Standard elements of negligence prima facie case apply* 2. Breach of duty is shown by negligent conduct by the defendant that leads to supplying a defective product to the plaintiff Any D in supply chain can be held liable; however, it is *difficult to hold retailers and wholesalers liable in negligence because they can usually satisfy their duty through a cursory inspection* Can use *res ipsa* if defect is something that would not usually occur without manufacturer's negligence. *DAMAGES*: requires physical injury or property damage. Recovery will be denied if claim is solely for economic loss (the product does not work as well as expected or requires repairs)

Survival and Wrongful Death

1. *Survival statutes* preserve a *victim's cause of action after his death*, such as personal injury and property ---NOT for torts involving intangible personal interests (e.g., defamation, privacy, malicious prosecution) 2. *Wrongful death statutes* permit a personal representative or surviving spouse to recover damages from a tortfeasor *for loss of the decedent's support and companionship* ---decedent's creditors have no claim against award ---recovery only to extent deceased could have recovered (thus contributory negligence applies)

A. Defamation: Private figure/Private concern

1. Apply the common law *prima facie case:* (1) Defamatory language (2) of and concerning the plaintiff (3) published to a third person (4) that causes damage to the plaintiff's reputation a. *Damages* *presumed* - libel (in writing or other permanent form) OR slander per se (spoken) (business or profession, loathsome disease, crime of moral turpitude, or unchastity of a woman); *ordinary slander* - no presumed damages, special (pecuniary) damages must be shown

Interference with Business Relations

1. Defined: (1) A valid contractual relationship or business expectancy of the plaintiff and a third party, (2) the defendant's knowledge of the relationship/expectancy, (3) intentional interference by the defendant inducing a breach or termination of the relationship, and (4) damages. 2. The defendant's conduct may be privileged if it is a proper attempt to obtain business or protect the defendant's interests

PRIMA FACIE CASE To establish a prima facie case for negligence, the following elements must be proved:

1. The existence of a *duty* on the part of the defendant to conform to a specific standard of conduct for the protection of the plaintiff against an unreasonable risk of injury; 2. *Breach* of that duty by the defendant; 3. That the breach of duty by the defendant was the *actual and proximate cause of the plaintiff's injury*; and 4. *Damage* to the plaintiff's person or property.

STRICT LIABILITY: Animals B. Extent of Liability

1. The harm must result from the kind of danger that makes the animal or activity abnormally dangerous 2. Many states apply their comparative fault rules to strict liability cases

Duty of Care for Children

A majority of courts take the view that a child is required to conform to the standard of care of a *child of like age, education, intelligence, and experience*. This permits a subjective evaluation of these factors.

DUTY OF CARE The "Unforeseeable" Plaintiff Problem 1) The Problem The "unforeseeable" plaintiff problem arises when defendant breaches a duty to one plaintiff (P1) and also causes injury thereby to a second plaintiff (P2) to whom a foreseeable risk of injury might or might not have been created at the time of the original negligent act. Example: An employee of Defendant negligently aided a passenger boarding the train, causing the passenger to drop a package. The package exploded, causing a scale a substantial distance away to fall upon a second passenger. Is the second passenger a foreseeable plaintiff?

2) The Solution(s) Defendant's liability to P2 will depend upon whether the Andrews or Cardozo view in Palsgraf is adopted. [Palsgraf v. Long Island Railroad, 248 N.Y. 339 (1928)] Most courts considering this issue have followed the Cardozo view. a) Andrews View - According to the Andrews view in Palsgraf, the second plaintiff (P2) may establish the existence of a duty extending from the defendant to her by showing that the defendant has breached a duty he owed P1. In short, defendant owes a duty of care to anyone who suffers injuries as a proximate result of his breach of duty to someone. b) Cardozo View - According to the Cardozo view in Palsgraf, the second plaintiff (P2) can recover only if she can establish that a reasonable person would have foreseen a risk of injury to her in the circumstances, i.e., that she was located in a *foreseeable "zone of danger."*

Vicarious Liability 4. *Automobile owner-driver*

4. *Automobile owner-driver*—an automobile owner is NOT vicariously liable for the negligence of the driver UNLESS the state has adopted: a. *A permissive use statute* (imposing liability for the torts of anyone driving with permission), or b. The *family car/purpose doctrine* (imposing liability for the torts of a family member driving with permission) Car owner can be DIRECTLY LIABLE for NEGLIGENT ENTRUSTMENT; or if present at the time of the accident on the theory that she could have prevented the negligent driving. (NOT VL)

PRODUCTS LIABILITY A. General Principles

5 theories of liability: intent, negligence, strict liability, implied warranties of merchantability and fitness for a particular purpose, and representation theories (warranty and misrepresentation) ---strict liability is the easiest to prove 2. Liability arises when a commercial supplier supplies a product in a defective condition unreasonably dangerous to users

a) Duty to Disclose Risks of Treatment

A doctor proposing a course of treatment or a surgical procedure has a duty to provide the patient with enough information about its risks to enable the patient to make an *informed consent* to the treatment. *If an undisclosed risk was serious enough that a reasonable person in the patient's position would have withheld consent to the treatment, the doctor has breached this duty.* Example: Patient consents to an operation not necessary to save his life. Patient is not informed that there is a 40% probability of paralysis in such operations, and paralysis results. Since a reasonable person would not have consented to the operation had the risks been disclosed, Doctor has breached his duty of disclosure.

To show breach of duty, a plaintiff may rely on evidence of any of the following except: A) Res ipsa loquitur B) Violation of a statute C) A "Good Samaritan" law D) Custom or usage

A "Good Samaritan" law has nothing to do with breach of duty. It refers to a statute exempting licensed doctors, nurses, etc., who voluntarily and gratuitously render emergency treatment, from liability for ordinary negligence. Hence, it would not be used to establish breach of duty. To prove breach of duty, it must be shown what in fact happened, and (based on these facts) that the defendant acted unreasonably. Proof of what happened may be established by either direct or circumstantial evidence. Other matters may also be offered into evidence to establish the standard by which defendant's conduct is to be measured, such as: 1. Custom or usage; 2. Violation of an applicable statute; and 3. The circumstantial evidence doctrine of res ipsa loquitur.

Special Relationship Between Parties

A defendant having a special relationship to the plaintiff (e.g., parent-child, employer-employee) may be liable for failure to act if the plaintiff is in peril.

If the plaintiff establishes res ipsa loquitur, it will have the following effect:

A directed verdict will not be given for the defendant.

Which of the following is correct for the defense of contributory negligence?

A failure to mitigate damages is not contributory negligence, rather, it is an avoidable consequence.

General Duty of Care

A general duty of care is imposed on all human activity. When a person engages in an activity, he is under a legal duty to act as an ordinary, prudent, reasonable person. It is presumed that an ordinary, prudent, reasonable person will take precautions against creating unreasonable risks of injury to other persons. Thus, if the defendant's conduct creates an unreasonable risk of injury to persons in the position of the plaintiff, the general duty of care extends from the defendant to the plaintiff. No duty is imposed on a person to take precautions against events that cannot reasonably be foreseen. Therefore, if at the time of the negligent conduct, no foreseeable risk of injury to a person in the position of the plaintiff is created by the defendant's act, the general duty of care does not extend from the defendant to the plaintiff. In addition, certain other factors such as the status of the parties (e.g., owners or occupiers of land) or statutes may limit or extend this general duty.

Watch out for those who CANNOT be held SL for Products:

A) nonmerchant causal sellers B) service providers with products that are incidentally provided (NOT considered merchants). ---EX: dentist chair breaks, injures D. Not strict liability since dentist is not a merchant of chairs.

*"Wrongful Life" Action* -

A lawsuit by or on behalf of a child with birth defects, alleging that, but for the doctor-defendant's negligent advice, the parents would not have conceived the child or would have terminated the pregnancy so as to avoid the pain and suffering resulting from the child's defects. Not Recognized In most states, the failure to diagnose a congenital defect of the fetus or to properly perform a contraceptive procedure does not permit the unwanted child to recover damages for "wrongful life," even if the child is born handicapped.

Transferred Intent

A legal principle under which a person who intends to harm one individual, but unintentionally harms a second person, can be liable to the second victim for an intentional tort. 1. Intent will transfer from the intended tort to the committed tort, or from the intended victim to the actual victim 2. Both the tort intended and the tort committed must be battery, assault, false imprisonment, trespass to land, or trespass to chattels ---in other words, CANNOT USE TRANSFERRED INTENT FOR IIED

Duty Owed Undiscovered Trespassers

A landowner owes *no duty* to an undiscovered trespasser. He has no duty to inspect in order to ascertain whether persons are coming onto his property.

"Good Samaritan" Statutes

A number of states have enacted statutes exempting licensed doctors, nurses, etc., who voluntarily and gratuitously render emergency treatment, from liability for ordinary negligence. Liability still exists, however, for gross negligence.

Scope of Invitation for Invitee

A person loses her status as an invitee if she exceeds the scope of the invitation—if she goes into a portion of the premises where her invitation cannot reasonably be said to extend. (Note that the invitation normally does extend to the entrance and steps of a building.) Example: Gas station customer, buying gas, loses status as invitee when she leaves pumps and falls into grease pit inside station. (Reversion to licensee, perhaps even trespasser, status.)

Recapture of chattels as defense to battery:

A person may use reasonable force, including harmful or offensive contact, to recapture chattels when in hot pursuit of one who has obtained the chattels wrongfully, such as through theft.

Particular Standards of Conduct Some persons are held to a standard of conduct different from that of the ordinary person. 1) Professionals

A person who is a professional or has special skills (e.g., doctor, lawyer, airplane mechanic, etc.) is required to possess and exercise the knowledge and skill of a *member of the profession or occupation in good standing.* For *doctors, most courts will apply a national standard* of care to evaluate their conduct. The professional must also use such superior judgment, skill, and knowledge as he actually possesses. Thus, a *specialist might be held liable where a general practitioner would not.*

Standard of Care for Contributory Negligence: Rescuers

A plaintiff may take extraordinary risks when attempting a rescue without being considered contributorily negligent. The emergency situation is one of the factors taken into account when evaluating the plaintiff's conduct.

*Alternative Causes Approach* *a) Burden of Proof Shifts to Defendants*

A problem of causation exists where two or more persons have been negligent, but uncertainty exists as to which one caused plaintiff's injury. Under the alternative causes approach, plaintiff must prove that harm has been caused to him by one of them (with uncertainty as to which one). The burden of proof then shifts to defendants, and each must show that his negligence is not the actual cause. Example: Alex and Basil both negligently fire shotguns in Clara's direction. Clara is hit by one pellet, but she cannot tell which gun fired the shot. Under the alternative causes approach, Alex and Basil will have to prove that the pellet was not theirs. If unable to do this, they may both be liable. [Summers v. Tice, 33 Cal. 2d 80 (1948)]

PRODUCTS LIABILITY: Govt safety standards

A product's noncompliance with govt safety standards establishes that it is defective, while compliance with safety standards (including labelling requirements) is evidence, but NOT conclusive, that the product is not defective

NUISANCE

Nuisance is not a separate tort in itself, but a type of harm that may be based on intent, negligence, or strict liability Two types: public and private

DUTY OF CARE Intended Beneficiaries of Economic Transactions

A third party for whose economic benefit a legal or business transaction is made (e.g., the beneficiary of a will) is *owed a duty of care if the defendant could reasonably foresee harm to that party if the transaction is done negligently*. Note that this is an exception to the general rule that one who suffers only economic loss as a result of another's negligence cannot recover damages in a tort action

When Is a Trespasser "Discovered"?

A trespasser is discovered, of course, when she is actually noticed on the property by the owner or occupier. But in addition, a trespasser is viewed as discovered if the owner or occupier is *notified by information sufficient for a reasonable person to conclude that someone is on the property*.

Implied Warranty of Merchantability

A warranty that goods being sold or leased are reasonably *fit for the ordinary purpose for which they are sold or leased*, are properly packaged and labeled, and are of fair quality. The warranty automatically arises in every sale or lease of goods made by a merchant who deals in goods of the kind sold or leased. Who can be sued: *merchant* dealing in the kinds of goods sold

Which of the following statements of law does NOT relate to proving actual cause in a strict products liability case? A) The defendant cannot avoid liability by showing negligent failure of an intermediary to discover the defect. B) If the defect is difficult to trace, the plaintiff may rely on an inference that such a product failure ordinarily would occur only as a result of a defect. C) If the defect has inadequate warnings, the plaintiff is entitled to a presumption that an adequate warning would have been read and heeded. D) The defect in the product must have existed when the product left the defendant's control.

A) The defendant cannot avoid liability by showing negligent failure of an intermediary to discover the defect. The rule that the defendant cannot avoid liability by showing negligent failure of an intermediary to discover the defect does not relate to actual cause; *rather, it relates to the proximate cause principle that negligence of an intermediary is foreseeable and not a superseding cause*. Under this principle, the intermediary's negligence does not cut off the defendant's liability for supplying a defective product. The *basic requirement to show ACTUAL cause is that the defect in the product must have existed when the product left the defendant's control*. When a defect is difficult to establish (such as if the product is destroyed), the plaintiff may rely on an inference that such a product failure ordinarily would occur only as a result of a defect (similar to res ipsa loquitur). To show that inadequate warnings were an actual cause of the injury, the plaintiff is entitled to a presumption that an adequate warning would have been read and heeded (i.e., but for the lack of an adequate warning, the plaintiff would not have been injured).

Partial Comparative Negligence Jurisdiction—Single Defendant Plaintiff is 30% negligent and Defendant is 70% negligent in causing the accident. Each party suffers $100,000 in damages.

Plaintiff will recover $70,000 from Defendant: $100,000 minus 30% ($30,000). Defendant will recover nothing from Plaintiff because Defendant was more than 50% at fault.

The negligence of a hospital is...

ALWAYS FORESEEABLE. [relevant usually to an intervening forces]

PRIVILEGE as defense to defamation

Absolute- official proceedings Qualified - matter of public interest... Stated honestly and reasonable belief that was true

Duty to Undiscovered Trespassers

Activity - NONE Static Condition - NONE

Duty to Invitee Invitee: Anyone who comes onto premises held open to the public at large. Often a business visitor. [Does NOT include police/firefighters, who are treated as licensees]

Activity - REASONABLE CARE (always reasonable care unless undiscovered trespasser) Static Conditions - duty to protect from 1) concealed 2) known to owner/occupier of land 3) could have discovered through by reasonable inspection - DUTY TO INSPECT

Duty to Licensee (social guest, police/firefighter) You LIKE licensees.

Activity - REASONABLE CARE (always reasonable care unless undiscovered trespasser) Static Conditions - duty to protect from 1) concealed condition 2) known to owner/occupier of land natural and artificial, highly and moderately dangerous

Duty to Discovered Trespassers

Activity - REASONABLE CARE (always reasonable care unless undiscovered trespasser) Static Conditions - only a duty regarding *man-made death traps* 1) highly dangerous, 2) concealed 3) artificial 4) known to owner/occupier of land

In most states, when a medical provider was negligent before or during a patient's pregnancy, damages for __________ may be recovered by the __________ in a wrongful _________ case.

Additional medical expenses; parents; birth

Indirect Cause (proximate cause) cases

After D's negligent conduct another intervening force combines with it to cause injury All about foreseeability. If intervening force was foreseeable, causal chain is intact. If not foreseeable, chain broken by a *superseding cause* If you have to be psychic to have foreseen it, it's not foreseeable.

Invasion of Privacy—Four Kinds of Wrongs in the Common Law *2. Intrusion on the plaintiff's affairs or seclusion*

An act of prying or intruding on the plaintiff's private affairs or seclusion that would be *highly offensive to a reasonable person* EX: eavesdropping, planting secret cameras, peeping Tom, intercepting mail, listening/looking at the keyhole. Exam Tip: you *must be in a place where you have a reasonable expectation of privacy*. *Defenses—consent*

"But For" Test

An act or omission to act is the cause in fact of an injury when the injury would not have occurred but for the act. Example: Failure to provide a fire escape is a cause of death of one who is thereby unable to flee a fire, but it is not a cause of death of one who suffocated in bed.

Indirect Cause Cases

An indirect cause case is one where the facts indicate that a force came into motion after the time of defendant's negligent act and combined with the negligent act to cause injury to plaintiff. In short, indirect cause cases are *those where intervening forces are present*. Whether an intervening force will cut off defendant's liability for plaintiff's injury is determined by foreseeability.

Definition of Invitee

An invitee is a person who enters onto the premises in response to an express or implied invitation of the landowner. Basically, there are two classes of invitees: (a) Those who enter as members of the public for a purpose for which the land is held *open to the public*, e.g., museums, churches, airports; and (b) Those who enter for a *business purpose* or other interests of the landowner or occupier, e.g., store customers and persons accompanying them, employees, persons making deliveries, etc.

Conduct of Persons on Property

An owner of land has a *duty to exercise reasonable care with respect to his own activities on the land* and to *control the conduct of others on his property* so as to avoid unreasonable risk of harm to others outside the property.

Defamation: Public figure or public concern

Apply the constitutional rules if the plaintiff is a public official or figure, or if the defamation involves a matter of public concern: a. The *plaintiff must prove that the statement was FALSE* Also must prove *FAULT*, BUT LEVEL DEPENDS ON TYPE OF PLAINTIFF: b. *Public officials or figures must prove "actual malice,"* i.e., that the statement was made with knowledge of its falsity or reckless disregard of its truth or falsity c. *Private figures suing on a matter of public concern must show* (i) *at least negligence* as to truth or falsity, and (ii) *actual injury* (no presumed damages) - economic, reputational, as long as P presents evidence of such damages

1) General Rule—No Affirmative Duty to Act

As a general matter, no legal duty is imposed on any person to affirmatively act for the benefit of others. This general rule is, however, subject to exception, as indicated below.

Collateral Source Rule

As a general rule, damages are not reduced or mitigated by reason of benefits received by plaintiff from other sources, e.g., health insurance, sick pay from employer. Hence, at trial, defendants may not introduce evidence relating to any such financial aid from other sources. A growing number of states have made exceptions to this rule in certain types of actions (e.g., medical malpractice actions), allowing defendants to introduce evidence of insurance awards or disability benefits. Note: These damages rules also are generally applicable to actions based on intentional torts.

Duty to Mitigate Damages

As in all cases, the plaintiff has a duty to take reasonable steps to mitigate damages—in property damage cases to preserve and safeguard the property, and in personal injury cases to seek appropriate treatment to effect a cure or healing and to prevent aggravation. Failure to mitigate precludes recovery of any additional damages caused by aggravation of the injury.

Effect of Contributory Negligence

At common law, plaintiff's contributory negligence completely barred his right to recover. This was so even though the degree of defendant's negligence was much greater than that of plaintiff. The severe consequences of strict application of contributory negligence rules initially caused courts to develop "escape" doctrines, such as last clear chance (below). More recently, however, most jurisdictions have rejected entirely the "all or nothing" approach of contributory negligence in favor of a comparative negligence system (discussed infra).

Standard of Care for Contributory Negligence: Violation of Statute by Plaintiff

Plaintiff's contributory negligence may be established by his violation of a statute under the same rules that govern whether a statute can establish defendant's negligence

1) Foreseeable Results Caused by Foreseeable Intervening Forces—

Defendant Liable Where defendant's negligence caused a foreseeable harmful response or reaction from an intervening force or created a foreseeable risk that an intervening force would harm plaintiff, defendant is liable for the harm caused.

D. CAUSATION 1. Actual Cause (Causation in Fact)

Before the defendant's conduct can be considered a proximate cause of plaintiff's injury, it must first be a cause in fact of the injury. Several tests exist: But for, substantial factor.

Implied Warranties (additional info) [covers merchantability & fitness for a particular purpose]

Breach - product fails to live up to the standard; no proof of fault by D necessary Causation - same as Negligence Damages - personal injury and property; AND pure economic loss recoverable Defenses (assumption of risk, contributory negligence, failure to give notice of breach) Disclaimers: ineffective (but upheld for purely economic loss)

Which of the following is NOT required for a statutory standard of care to apply in a negligence case? A) The statute must have been designed to prevent the type of harm that occurred B) The statute must state clearly what standard of conduct is expected C) The statute must provide for a civil penalty D) The plaintiff must be in the class intended to be protected by the statute

C The standard of care in a common law negligence case may be established by proving the applicability of a statute that provides for a *criminal penalty, NOT a civil penalty*. If the statute applies, the statute's specific duty will replace the more general common law duty of due care. If the statute in question provides for a civil remedy, the plaintiff will sue directly under the statute: i.e., it is not a common law negligence case. For a statute to apply, the specific duty imposed by the statute must be clearly stated. The plaintiff also must show that: (i) She is in the class intended to be protected by the statute; and (ii) The statute was designed to prevent the type of harm that the plaintiff suffered.

Duty of Care: Common Carriers and Innkeepers

Common carriers and innkeepers are required to exercise a very high degree of care toward their passengers and guests; i.e., they are *liable for slight negligence.*

Duty of Common Carriers

Common carriers are under a *duty to use reasonable care to aid or assist passengers.*

Damages

Common law = Actual physical harm is required. Do not discuss punitive, emotional harm, economic harm Common law requires an actual physical injury Facts will tell you if there's an injury

The "shopkeeper's privilege" allows a shopkeeper to avoid liability for false imprisonment when detaining a suspect that he reasonably believes has committed a theft. The shopkeeper also must:

Conduct the detention in a reasonable manner and detain the suspect for only a reasonable time. By statute in some states and case law in others, shopkeepers have been given a privilege to detain someone suspected of shoplifting and thus avoid liability for false imprisonment. The following conditions must be satisfied: 1. There must be a reasonable belief as to the fact of theft; 2. The detention must be conducted in a reasonable manner and only nondeadly force can be used; and 3. The detention must be only for a reasonable period of time and only for the purpose of making an investigation. A shopkeeper is NOT required to notify the police in a reasonable amount of time to avoid liability for false imprisonment when detaining a suspect for shoplifting

Defenses to Intentional Torts: Consent

Consent is a defense to all 7 intentional torts. *Was there valid consent? [no fraud]* "Did the D stay within the boundaries of the consent? [no gun in a boxing match]* The *plaintiff must have capacity* to consent ---EX: drunk person can't consent to boxing match. However, *people with diminished capacity CAN consent to things within their capacity*. ---EX: 11 year old can consent to the "battery" of wrestling with his 12 year old friend. ---EX: mentally disabled person can consent to the "battery" of a haircut, but not to major elective surgery the defendant *must NOT exceed the scope of the consent* ---EX: doctor can't extend operation to a new part of the body that wasn't discussed prior to surgery.

No Defense to Intentional Torts

Contributory negligence is never a defense to an action for an intentional tort or for willful or wanton misconduct.

What effect does comparative negligence have on implied assumption of risk? [continued] But what if the P wasn't acting unreasonably but still voluntarily assuming a known risk? EX: the ski resort didn't have the P sign an express AOR waiver, but the resort argues that the P knew the risks of skiing and participated anyway. So the P shouldn't recover. Result?

Courts go all the way back to the duty element and limit the D's duty. The court will say that the D didn't have a duty to protect the plaintiff against a known risk of an activity like skiing. So unless the D did something egregious, the P can't recover.

Custom or Usage

Custom or usage may be introduced to establish the standard of care in a given case. However, customary methods of conduct do not furnish a test that is conclusive for controlling the question of whether certain conduct amounted to negligence.

Which of the following will *not negate* a claim of strict liability for an abnormally dangerous activity? A) There was an unforeseeable intervening force. B) The type of harm that occurred was not foreseeable. C) The plaintiff was not foreseeable. D) The defendant did not foresee a risk of harm.

D ELEMENTS: (1) foreseeable risk of serious harm even when reasonable care exercised, and (2) not common in the community The fact that the defendant did not foresee a risk of harm will not negate a claim of strict liability for an abnormally dangerous activity. To prevail, a plaintiff need only show that a *reasonable person (objective standard) could have foreseen the risk of harm, regardless of whether the defendant did not (so it is NOT a subjective standard).* If the plaintiff was not foreseeable, the strict liability claim is NOT established. The defendant's liability for an abnormally dangerous activity extends only to foreseeable plaintiffs, who are persons to whom a reasonable person would have foreseen a risk of harm under the circumstances. Note, though, that the nature of the abnormally dangerous activity may create a large class of foreseeable plaintiffs. If the type of harm was not foreseeable, the plaintiff cannot establish a strict liability claim. The harm must result from the kind of danger to be anticipated from the abnormally dangerous activity; i.e., it must flow from the "normally dangerous propensity" of the condition or thing involved. An unforeseeable intervening force may allow a defendant to avoid liability in a strict liability action for an abnormally dangerous activity. The same rules govern causation for strict liability as they do for negligence, and thus a defendant's liability may be cut off by an unforeseeable intervening force that brings about the injury.

No Duty To Rescue; Unless Special Relationships - has heightened duty (4 common examples)

Innkeeper Common Carrier Employer/ee Parent/child

Duty of Places of Public Accommodation

Innkeepers, restaurateurs, shopkeepers, and others who gather the public for profit have a duty to use reasonable care to aid or assist their patrons and to prevent injury to them from third persons.

Invasion of Privacy—Four Kinds of Wrongs in the Common Law *1. Appropriation of the plaintiff's picture or name*

D uses name or image for a *commercial purpose* without authorization. ---Limited to the advertisement or promotion of products or services ---EX: using pro athlete's image on cereal box without permission. Exam Tips: 1) Newsworthiness Exception: can print a story about a newsworthy event involving athlete. 2) Tort applies to everyone, not just celebrities. ---EX: Person takes picture of you secretly then makes into billboard. You can sue for appropriation of image for commercial purposes. *Defenses—consent*

Duties Owed by Bailee *(1) Sole Benefit of Bailor Bailment*

If the bailment is for the sole benefit of the bailor (e.g., the bailor asks his neighbor (the bailee) to take in the bailor's mail while he is on vacation), the *bailee is liable only for gross negligence*.

DAMAGES

Damage is an essential element of plaintiff's prima facie case for negligence. This means actual harm or injury. Unlike the situation for some of the intentional torts, damage will not be presumed. Thus, nominal damages are not available in an action in negligence; some proof of harm must be offered.

Unforeseeable Extent or Severity of Harm—

Defendant Liable In both direct cause and indirect cause cases, the fact that the extent or severity of the harm was not foreseeable does not relieve defendant of liability; i.e., the tortfeasor takes his victim as he finds him. This is also known as the "eggshell-skull plaintiff" rule. Thus, where defendant's negligence causes an aggravation of plaintiff's existing physical or mental illness, defendant is liable for the damages caused by the aggravation. Example: A car negligently driven by D collides with a car driven by P. P suffers a slight concussion, which was foreseeable, and also suffers a relapse of an existing mental illness, which was not foreseeable. D is liable for all of P's damages.

Direct Cause Cases A direct cause case is one where the facts present an *uninterrupted chain of events* from the time of the defendant's negligent act to the time of plaintiff's injury. In short, there is no external intervening force of any kind. *1) Foreseeable Harmful Results*—

Defendant Liable If a particular harmful result was at all foreseeable from defendant's negligent conduct, the unusual manner in which the injury occurred or the unusual timing of cause and effect is irrelevant to defendant's liability. Example: D is driving her sports car down a busy street at a high rate of speed when a pedestrian steps out into the crosswalk in front of her. D has no time to stop, so she swerves to one side. Her car hits a parked truck and bounces to the other side of the street, where it hits another parked vehicle, propelling it into the street and breaking the pedestrian's leg. D is liable despite the unusual way in which she caused the injury to the pedestrian.

4) Unforeseeable Results Caused by Unforeseeable Intervening Forces—

Defendant Not Liable As a general rule, intervening forces that produce unforeseeable results (i.e., results that were not within the increased risk created by defendant's negligence) will be deemed to be unforeseeable and superseding. A superseding force is one that serves to break the causal connection between defendant's initial negligent act and the ultimate injury, and itself becomes a direct, immediate cause of the injury. Thus, defendant will be relieved of liability for the consequences of his antecedent conduct. Example: D negligently blocks a road, forcing P to take an alternate road. Another driver negligently collides with P on this road, injuring him. Even though D is an actual (but for) cause of P's injury, the other driver's conduct is an unforeseeable intervening force because D's negligence did not increase the risk of its occurrence. Thus, the other driver is a superseding force that cuts off D's liability for his original negligent act.

Direct Cause Cases 2) Unforeseeable Harmful Results—

Defendant Not Liable In the rare case where defendant's negligent conduct creates a risk of a harmful result, but an entirely different and totally unforeseeable type of harmful result occurs, most courts hold that defendant is not liable for that harm. Example: D, a cabdriver, is driving too fast on a busy elevated highway, threatening P, his passenger, with injury. Without warning, the section of highway that D is on collapses because its support beams had deteriorated with age. P is seriously injured. Even if D's negligent conduct was an actual cause of P's injury (because the cab would not have been on that section of the highway but for D's speeding), courts would not hold D liable for the injury to P.

PROXIMATE CAUSE: INDIRECT CASE Unforeseeable Results Caused by Foreseeable Intervening Forces—

Defendant Not Liable Most intervening forces that produce unforeseeable results are considered to be unforeseeable intervening forces (see below). Similarly, most results caused by foreseeable intervening forces are treated as foreseeable results. In the rare case where a foreseeable intervening force causes a totally unforeseeable result, most courts would not hold the defendant liable. Example: D, a cabdriver, was driving recklessly during a violent windstorm that was blowing large branches and other debris onto the road, creating a risk to P, his passenger, that D would not be able to stop the cab in time to avoid an accident. D slammed on his brakes to avoid a large branch in the road, causing his cab to swerve sideways onto the shoulder of the road. Before he could proceed, another branch crashed onto the roof of the cab, breaking a window and causing P to be cut by flying glass. D is not liable to P even though his negligent driving was the actual (but for) cause of P's injury and the wind that was blowing the branches down was a foreseeable intervening force.

PROXIMATE CAUSE: INDIRECT CAUSE Foreseeable Results Caused by Unforeseeable Intervening Forces—

Defendant Usually Liable The problem: Defendant is negligent because his conduct threatens a result of a particular kind that will injure plaintiff. This result is ultimately produced by an unforeseeable intervening force. Most courts would generally find liability here because they give greater weight to foreseeability of result than to foreseeability of the intervening force. An exception exists, however, where the intervening force is an unforeseeable crime or intentional tort of a third party; it will be deemed a "superseding force" that cuts off defendant's liability (see discussion below). Examples: 1) Defendant failed to clean residue out of an oil barge, leaving it full of explosive gas. Negligence, of course, exists since an explosion resulting in harm to any person in the vicinity was foreseeable from any one of several possible sources. An unforeseeable bolt of lightning struck the barge, exploding the gas and injuring workers on the premises. Defendant is liable. 2) Same facts as above example, except that an arsonist caused the explosion. Most courts would not hold Defendant liable here. They think it unfair to make him responsible for such malevolent conduct. The important point here is that an unforeseeable intervening force may still relieve the defendant of liability if it is an unforeseeable crime or intentional tort of a third party.

Products Liability Based on INTENT

Defendant will be liable to anyone injured by an unsafe product if D intended the consequences or knew that they were substantially certain to occur. ---unusual. Punitives are available Defenses: same as intentional torts

A pilot was flying her small airplane when she realized that she was rapidly losing fuel and would not make it to the nearest airport. Looking down, she could find no large open space on which to attempt a landing except for a highway off to her left and a nearby lake about a mile to her right. She considered ditching the plane in the water but decided against it under the circumstances. As the pilot maneuvered over the highway and saw a long section free of any overpass or obstruction, her engine sputtered and died. In a barely controlled glide, the pilot descended onto the highway, but her left wing sideswiped the median and her plane veered to the right, crashing into a car. The plane and car catapulted into a fence, severely injuring both the pilot and driver. The driver brought an action for personal injuries against the pilot. At trial, the above facts were established, and the parties stipulated that the sudden loss of fuel was due to a defect in the fuel system that could not have been discovered by the pilot. At the close of the evidence, both parties moved for a directed verdict. How should the court rule?

Deny both motions, because the jury could decide that the pilot's selection of the highway rather than the lake was not a reasonable choice under the circumstances. The court should deny both motions and submit the case to the jury. Through process of elimination, this has to be a negligence action. Clearly the pilot did not commit an intentional tort, and the driver cannot sue the pilot, the plane's owner, in strict liability. The jury could determine that the pilot was negligent in selection of the landing site; hence, the court should deny the pilot's motion. The court should also deny the driver's motion because this is not a strict liability action; negligent conduct needs to be established.

The driver of a tanker truck was transporting radioactive waste from a nuclear power plant to a permanent storage facility in a remote western region of the United States. After driving all night, the driver fell asleep at the wheel and the truck crossed over the center line, off the road, and onto a homeowner's property, coming to rest after crashing into several glass cases containing the homeowner's collection of poisonous snakes, the keeping of which was permitted by local ordinance. When the driver exited the truck, he was bitten on the leg by one of the poisonous snakes and became seriously ill. The driver brought an action against the homeowner for his injuries. The parties stipulated to the above facts, and that the driver violated a state statute by driving off of the road. Both parties moved for judgment as a matter of law on the liability issue. How should the court rule?

Deny the driver's motion and grant the homeowner's motion, because the driver was a trespasser on the homeowner's property. The court should grant the homeowner's motion for judgment as a matter of law because the driver has not established a prima facie case against the homeowner. An owner of wild (dangerous) animals is strictly liable for injuries caused by those animals as long as the person injured did nothing, voluntarily or consciously, to bring about the injury. However, *strict liability generally is NOT imposed in favor of undiscovered trespassers against landowners in the absence of negligence*, such as when the landowner knows that the trespassers are on the land and fails to warn them of the animal. Here, despite the fact that the driver did not intend to enter the homeowner's land (and thus would not be liable for the intentional tort of trespass), his status on the homeowner's land is that of a trespasser rather than a licensee or invitee. The driver has presented no evidence of negligence on the homeowner's part and therefore has not established a prima facie case against the homeowner.

A fire broke out in a home that had been recently remodeled, destroying the house and injuring the homeowner. An investigation by the fire marshal established that the fire started from a short in some wiring behind a wall. A small section of wiring that ran to an outlet through a narrow gap between a furnace chimney and a hot water pipe had had part of its outer sheath cut off. The homeowner filed suit against the electrical company that did the rough wiring. The parties stipulated for trial that the company had installed the wiring in compliance with the blueprints, and that the wiring had been inspected and approved by the building inspector before the chimney and the water pipe had been installed and the walls put up, all by different contractors. At trial, the homeowner introduced the report of the fire marshal establishing how the fire started, and evidence of his medical expenses and other damages. At the end of the homeowner's case, the electrical company's attorney rested her case and moved for a directed verdict. The homeowner's attorney also moved for a directed verdict. How should the court rule on the directed verdict motions?

Deny the homeowner's motion and grant the electrical company's motion for a directed verdict, because the wire could have been damaged by another contractor. The court should grant the electrical company's motion for a directed verdict because the homeowner has not established a prima facie case of negligence on the company's part. The homeowner has established that the electrical company owed a duty to him and that he has suffered harm from the fire caused by the short in the wiring. However, he has not established that the company breached any duty to him. While breach of duty is ordinarily a question for the trier of fact, a plaintiff's failure to offer any evidence on that element of the prima facie case will permit a directed verdict for the defendant. Under certain circumstances, the fact that a particular injury occurred may itself establish or tend to establish a breach of duty owed, permitting the trier of fact to infer the defendant's liability. This is the doctrine of res ipsa loquitur ("the thing speaks for itself"). However, for the doctrine to apply, the plaintiff must show that (i) the accident causing his injury is the type that would not normally occur unless someone was negligent; (ii) the negligence was attributable to the defendant; and (iii) the injury was not attributable to the plaintiff. The second requirement can often be satisfied by showing that the instrumentality causing the injury was in the exclusive control of the defendant. Here, however, the wiring was exposed to work done by other contractors in installing a chimney and a hot water pipe nearby and putting up the walls, and the homeowner has offered no evidence that the cut in the outer sheath of the wiring was present when the electrical company finished its work. Instead, the fact that the wiring had been approved by the building inspector suggests that the wiring was intact when the electrical company finished. Given these facts, the homeowner has not presented evidence that the negligence was attributable to the defendant. Since res ipsa loquitur does not apply and no other evidence of breach of duty was established, the electrical company's motion for a directed verdict should be granted.

Duty of a Landowner to those who come onto property

Depends on who you are: *Unknown trespasser* - no duty of care. *Known trespasser or licensee* Licensee - social guests, friends, personal relationship. Duty: duty to warn of known dangers *Invitee* - business/commercial relationship. EX: law school, grocery store, etc. property. Duty: to warn, clean up, and make safe. All three are required. EX: banana peel on aisle 4.

Imputed Contributory Negligence

EX: Driver and Passenger are involved in an automobile accident with Cyclist. Driver is negligent; Cyclist is also negligent. Passenger, who is injured, brings an action against Cyclist. Cyclist argues that liability should be denied because of Driver's negligence to the same extent as if Passenger had been negligent himself. This is the concept of "imputed contributory negligence." *1) General Rule—Plaintiff May Proceed Against Both Negligent Parties* -As a general rule, a plaintiff's action for his damages is not barred by imputed contributory negligence. He may proceed against both negligent parties as joint tortfeasors to the extent that each is a legal cause of the harm. *2) When Contributory Negligence Is Imputed* Contributory negligence will be *imputed only where the plaintiff and the negligent person stand in such a relationship to each other that the courts find it proper* to charge plaintiff with that person's negligence, i.e., *where plaintiff would be found vicariously liable for the negligent person's conduct* if a third party had brought the action.

Abuse of Process vs. Malicious Prosecution

Even if a criminal or civil proceeding is brought with probable cause, for allowable motives, a person involved in it may use various litigation devices available to him during the course of the litigation for improper purposes. This is abuse of process. AoP is not wrongful institution of the action or proceeding, but rather the improper use of process in connection therewith. Thus, the merits of the action itself are of no relevance. *In contrast to malicious prosecution, therefore, neither want of probable cause nor favorable termination are elements*

As Defense to Violation of Statute by Defendant Contributory negligence is ordinarily a defense to negligence proved by defendant's violation of an applicable statute. But where the defendant's negligence arose from violation of a statute designed to protect this particular class of plaintiffs from their own incapacity and lack of judgment, then plaintiff's contributory negligence is not a defense.

Example: D is exceeding the speed limit in a school zone when a child on his way to school darts into the street without looking. Because of her speed, D is unable to stop and hits the child. Any contributory negligence on the child's part is not a defense to D's violation of the statute, because the statute was designed to protect children on their way to school.

Breach

Failure to comply with the level of care owed. Involves an analysis of specific conduct

Last Clear Chance: Prior Negligence Cases

For last clear chance to operate, defendant must have been able to avoid harming plaintiff at the time of the accident. In short, defendant must have had the "last clear chance" to avoid the accident. Hence, if defendant's only negligence had occurred earlier, e.g., she negligently failed to have the steering wheel fixed, the courts will not apply last clear chance

Difference between foreseeability in duty and foreseeability in proximate cause?

Foreseeability in duty looks at WHO are foreseeable persons in the zone of danger to which D owes a duty. Proximate cause looks at WHAT HARMS are foreseeable. Ask: why was the D's conduct deemed to be negligent in the first place? D is liable for the type of harm, the risk of which made D's conduct negligent in the first place

To Whom Is the Duty of Care Owed?

Foreseeable Plaintiffs

A camper at a state park built a campfire within a fire ring on a calm day according to approved procedures. Just as a sudden strong wind arrived and blew some embers onto the grass, a large bear came out of the woods and charged at the camper. The camper ran to his car, which was some distance away, with the bear in close pursuit. By the time the bear left and the camper was able to exit the car and summon assistance, the embers in the grass had started a brush fire. The fire destroyed another camper's equipment and automobile at a nearby campsite before it could be extinguished. The other camper sued the camper who started the fire. At trial, the parties stipulated to the above facts. The plaintiff introduced into evidence a state statute that prohibited leaving any campfires unattended and required them to be extinguished immediately if any embers were blown out of the fire ring. At the conclusion of the proofs, both parties moved for a directed verdict. How should the court rule on the motions?

Grant the defendant's motion, because the plaintiff has not established a prima facie case of negligence. The court should grant the defendant's motion because the plaintiff has not offered sufficient evidence of negligence on the defendant's part to go to the jury. The standard of care in a negligence case may be established by proving that a statute imposing a specific duty applies instead of the more general common law duty of care. However, violation of the statute may be excused where compliance would cause more danger than violation or where compliance would be beyond the defendant's control. Here, the statute regulating campfires is applicable because (i) the plaintiff, a fellow camper, is in the class intended to be protected by the statute, (ii) the statute was designed to prevent the escape of a campfire, which is what occurred here, and (iii) the statute clearly specifies what is required. However, even though the statute would apply to the defendant's conduct and the defendant violated the statute, the violation will be excused here because he was fleeing for his life from a bear and had to take refuge in his car. Hence, the defendant will not be held to the statutory standard of care here. Because the plaintiff has offered no other evidence that the defendant was negligent, the defendant's motion should be granted.

If a condition on a landowner's property creates a danger to those off the premises:

He owes a duty to passersby to take due precautions Essnetially an application of the regular duty of care standard. For example, he should erect a barricade to keep people from falling into an excavation at the edge of his property.

If a statement attacking the P is true, the P has no COA for defamation. However, consider whether the P may have these two torts:

IIED Invasion of privacy (unless P is a public figure or matter of public concern is involved)

Assault - Unloaded Gun Problem D threatens P with unloaded gun. Assault?

If P knows gun is unloaded, then P "apprehends" there can't be an imminent battery. If P does NOT know, then P reasonably "apprehends" that there COULD be an imminent battery. Don't have to know to a certainty. ---It is REASONABLE to assume gun is loaded.

Duties Owed by Bailee *(3) Mutual Benefit Bailments*

If the bailment is for the mutual benefit of the bailor and bailee (typically a bailment for hire such as in the computer repair example above), the *bailee must exercise ordinary due care.*

Duties Owed by Bailee *(2) Sole Benefit of Bailee Bailment*

If the bailment is for the sole benefit of the bailee (e.g., the bailor gratuitously loans her lawnmower to the bailee), the *bailee is liable even for slight negligence*.

Effect of Lessor's Covenant to Repair

If the lessor has covenanted to make repairs and reserves the right to enter the leased premises for the purpose of inspecting for defects and repairing them, he is subject to liability for unreasonably dangerous conditions.

Effect of Admission of the Public on Lessor's Duties

If the lessor leases the premises knowing that the lessee intends to admit the public, the lessor is subject to liability for unreasonably dangerous conditions existing at the time he transfers possession where the nature of the defect and length and nature of the lease indicate that the tenant will not repair (e.g., lessor rents convention hall to tenant for three-day period). This liability continues until the defect is actually remedied. A mere warning to the lessee concerning the defect is not sufficient. (The duty of care of tenants and lessors is also covered in the Real Property outline.)

Effect of Voluntary Repairs by Lessor

If the lessor, though under no obligation to make repairs, does so, he is subject to liability if he does so negligently, failing to cure the defect; it is not necessary that his negligent repairs make the condition worse.

How can you tell a res ipsa questions from CL negligence from negligence per se?

If they are asking for "recovery" "liable" etc. = CL negligence If they say, "motion for directed verdict" or MSJ, motion to dismiss, motion, etc. and question is how will the court rule? = then it's res ipsa. Statutory violation = Negligence Per Se [class of person, class of risk]

Bailment Duties

In a bailment relationship, the bailor transfers physical possession of an item of personal property to the bailee without a transfer of title. The bailee acquires the right to possess the property in accordance with the terms of the bailment. A bailment obligates the bailee to return the item of personal property to the bailor or otherwise dispose of it according to the bailment terms. Example: When the owner of a computer delivers it to a technician to be repaired, the technician becomes a bailee of the computer and the owner is the bailor.

Proximate Cause (Legal Causation)

In addition to being a cause in fact, the defendant's conduct must also be a proximate cause of the injury. Not all injuries "actually" caused by defendant will be deemed to have been proximately caused by his acts. Thus, the doctrine of proximate causation is a limitation of liability and deals with liability or nonliability for unforeseeable or unusual consequences of one's acts.

A *public nuisance* is

an act that unreasonably interferes with the health, safety, or property rights of the community EX: using a building for criminal activities such as prostitution Recovery by a private party is available for a public nuisance ONLY if the private party suffered unique damage not suffered by the public at large.

Effect of Comparative Negligence on Other Doctrines 3) Wanton and Willful Conduct

In most comparative negligence jurisdictions, plaintiff's negligence will be taken into account even though the defendant's conduct was wanton and willful" or "reckless." However, plaintiff's negligence is still not a defense to intentional tortious conduct by the defendant.

Duty of Care: Automobile Driver to Guest

In most jurisdictions today, the duty owed by the driver of an automobile to a rider is one of *ordinary care*. *Guest Statutes* A few states have guest statutes. Under these statutes, the driver's only duty to a nonpaying rider is to refrain from gross or wanton and willful misconduct. Note that guest statutes do not apply to "passengers," i.e., riders who contribute toward the expense of the ride; they are owed a duty of ordinary care.

Standard of Care Owed by Owners and/or Occupiers of Land

In this section, duty problems are resolved by application of special rules that have been developed imposing duties on individuals because of their relationship to property. In some cases, the duty of the owner or occupier depends on whether the injury occurred on or off his premises; in others it depends on the legal status of the plaintiff with regard to the property, i.e., trespasser, licensee, or invitee.

PRODUCTS LIABILITY: Privity NOT required

In virtually all products liability actions, the fact that there was no contractual privity between P and D will NOT prevent P from recovering. It is a favorite WRONG choice. ANY FORESEEABLE PLAINTIFF, including a bystander, can sue any commercial supplier in the chain of distribution regardless of the absence of a contractual relationship

Indemnity involves shifting the entire loss between or among tortfeasors, and is available where:

Indemnity = 100% contribution for liability (i) there is a contractual promise to indemnify; (ii) there is a special relationship between the defendants that would allow for vicarious liability; or (iii) the defendant is a supplier in a strict products liability case who is liable to an injured customer, thus giving the supplier a right of indemnification against previous suppliers in the distribution chain.

Indirect cause (proximate cause fact pattern)

Intervening cause. [order of events: D acts, intervening act, then P injured] *Law asks two questions:* 1) Was the the intervening act foreseeable? 2) Was the result foreseeable? *Leads to 4 possible scenarios:* 1) foreseeable intervention/foreseeable outcome - P wins 2) unforeseeable intervention/unforeseeable outcome - P loses 3) foreseeable intervention/unforeseeable outcome - depends 4) unforeseeable intervention/foreseeable outcome - depends

Privilege of Arrest

Invasion of land - privilege of arrest carries with it the privilege to enter another's land to arrest Subsequent misconduct - arrestor may be liable for failing to bring before a magistrate; extended detention Misdemeanor - privileged to arrest only for breach of the peace that takes place in front of D ---reasonable force allowed, but NEVER deadly Felony - a POLICE OFFICER may make a reasonable mistake. ---CITIZENS may make a reasonable mistake regarding the identity of the felon, but NOT regarding whether a felony occurred ---Degree of Force: only that which is reasonably necessary; deadly force only when threat of serious harm

Which of the following is an effect of res ipsa loquitur?

It establishes a prima facie case for the plaintiff

A homeowner purchased a ladder from a home supply retailer. While he was using the ladder, an improperly installed bolt fastening one of the rungs gave way, causing him to fall and break his leg. The homeowner sued the manufacturer of the ladder to recover damages for his injury. If it is established at trial that the home supply retailer could have discovered the defectively installed bolt if it had conducted a reasonable inspection of the ladder, what is the effect of the retailer's failure to inspect?

It has no legal effect on the manufacturer's liability. The failure of the home supply retailer to inspect the ladder has no legal effect on the manufacturer's liability, regardless of whether the plaintiff is suing in negligence OR strict liability. *Under either negligence OR strict liability, an intermediary's negligent failure to discover a defect is not a superseding cause*, so the defendant who supplied the defective product will still be liable. Thus, even if the home supply retailer were negligent in not discovering the defect, it would not relieve the manufacturer of liability.

Abuse of Process

It is a tort to use any form of process, civil or criminal, to bring about a result other than that for which the process was intended EX: D garnished an account to force P to sign a lease

Section 1 of the Vehicle Code of a state makes it illegal to cross a street in a central business district other than at a designated crosswalk. Section 2 of the Code prohibits parking any motor vehicle so that it blocks any part of a designated crosswalk. A pedestrian wanted to cross Main Street in the central business district of a city, located in the state, but a truck parked by a trucker was blocking the designated crosswalk. The pedestrian stepped out into Main Street and carefully walked around the back of the truck. The pedestrian was struck by a motor vehicle negligently operated by a driver. If the pedestrian asserts a claim against the driver, the pedestrian's failure to be in the crosswalk will have which of the following effects?

It may be considered by the trier of fact on the issue of the driver's liability. The Examiners' instructions state that pure comparative negligence is to be used unless there are specific instructions in the facts to the contrary. A comparative negligence analysis determines responsibility and damages based on the negligence of every party directly involved in the accident. The statute that the pedestrian violated was designed to protect pedestrians, and the harm the pedestrian suffered was the type that the statute was designed to protect. Since this violation is not excused, the violation would be negligence per se by the pedestrian. Because the pedestrian was negligent and this is a comparative negligence jurisdiction, the pedestrian's failure to use the crosswalk can be considered by the trier of fact to determine the driver's liability.

Standard of Care for Contributory Negligence: Remaining in Danger

It may be contributorily negligent to fail to remove oneself from danger, e.g., remaining in a car with a drunk driver.

A man working at a clothing store discovered that his girlfriend, a coworker, had been taking money from the cash register. Not wanting to be a party to the situation, he ended the relationship and found another job. Not long after this, the man's new boss, who knew why the man had quit, came into the clothing store. He asked the girlfriend if she missed her boyfriend working with her at the store. She replied, "Yes, but when we found that he was stealing from the cash register, we had no choice but to let him go." If the man sues his former girlfriend for defamation, the fact that the new boss knew the truth of why the man had left his job at the store will have what result?

It may diminish the damages that the man would be entitled to recover. The new boss's knowledge of the true circumstances behind the man's departure from the store may diminish the man's recovery. The girlfriend is liable for defamation because she made a defamatory statement about the man to a third person. As long as it is understood in its defamatory sense, an accusation need not be believed to be actionable. Because the statement that he was stealing at his job constituted slander per se, damages are presumed, completing the prima facie case. Nevertheless, the stated fact will diminish the amount of damages that the man will recover because his reputation was not likely harmed in the eyes of his boss. actual injury encompasses not only damage to reputation but also humiliation and mental distress, for which the man could recover even if his reputation was not damaged.

A construction company that was putting in a swimming pool for a homeowner left a couple of large pieces of equipment in the backyard overnight. The equipment was not owned by the construction company but was leased from an equipment company, which was responsible for its repair and maintenance. After the workers had left, a seven-year-old boy came onto the homeowner's property to play. The homeowner was aware that the boy often came onto his property to play with his dog. The boy climbed up on one of the pieces of equipment and began pushing buttons and moving levers. The engine started and the equipment began to move because the equipment company had not replaced a defective safety locking device on the ignition. The boy became frightened and jumped off, falling into the hole that had been dug that day, and was injured. The boy's parents brought suit against the homeowner and the construction company. If the construction company is held liable for the boy's injuries, may it recover anything from other tortfeasors?

It may obtain contribution from the equipment company because the equipment was negligently maintained in an unsafe condition. When two or more tortious acts combine to proximately cause an indivisible injury to a plaintiff, each tortious actor will be jointly and severally liable for that injury. Joint and several liability permits a plaintiff to recover the entire judgment amount from any defendant. Contribution allows a defendant required to pay more than his share of damages to recover from the other jointly liable parties for the excess. In other words, contribution apportions responsibility among those who are at fault. Here, if the construction company is held liable for the boy's injuries, it will be because of its negligence in leaving unattended a piece of equipment without a working safety locking device. However, because the equipment company, which was responsible for repair and maintenance of the equipment, negligently performed such maintenance, resulting in the absence of a working safety locking device, then the equipment company's negligence would have combined with that of the construction company to proximately cause the boy's injuries. This would render the companies jointly and severally liable to the boy for the entire damage incurred. Thus, if the construction company is held liable for the injuries, it has a claim against the equipment company, as a jointly liable party, for the amount it pays in excess of its share of damages. Note that, in the usual case, the equipment company would have been included in the lawsuit. However, the fact that it was not included does not preclude the construction company from recovering contribution in a separate action. ( Indemnity is not available here. Indemnity involves shifting the entire loss between or among tortfeasors, and is available where: (i) there is a contractual promise to indemnify; (ii) there is a special relationship between the defendants that would allow for vicarious liability; or (iii) the defendant is a supplier in a strict products liability case who is liable to an injured customer, thus giving the supplier a right of indemnification against previous suppliers in the distribution chain. In addition, some states allow a joint tortfeasor to recover indemnification from a co-joint tortfeasor where there is a considerable difference in degree of fault. Here, there is no evidence of a contractual right to indemnity between the construction company and the equipment company, there is no relationship between them that causes the construction company to be held vicariously liable for the equipment company's negligence, and this is not a strict products liability case. Also, there is no indication of a considerable difference in degree of fault between the two companies. Therefore, none of the circumstances in which indemnity is available is present.

Assumption of Risk - Implied Two elements

KNOWING AND VOLUNTARY 1) P must have knowledge of the risk 2) P must voluntarily encountered risk. - HOWEVER, if acting to help others in an emergency situation (jump in front of car to save child), it will NOT be considered voluntary and will be able to maintain action against D. CONSEQUENCE - Traditionally: a complete bar to recover - Today, most jx have moved to comparative negligence, which changes this.

Tenant Remains Liable to Invitees and Licensees

Keep in mind that the potential liability of the lessor for dangerous conditions on the premises does not relieve the tenant, as occupier of the land, of liability for injuries to third persons from the dangerous conditions within the tenant's control.

Defamatory Language

Language tending to adversely affect one's reputation. Actionable only if it appears to be based on specific facts, and an express allegation of those facts would be defamatory. Name calling insufficient *Inducement and innuendo* ---If the statement is not defamatory on its face, P may plead additional facts as "inducement" to establish defamatory meaning by "innuendo" *Living Person Requirement* ---Defamation is not actionable for deceased person ---In a limited sense, a corporation, unincorporated association, or partnership may be defamed

Effect of Comparative Negligence on Other Doctrines 1) Last Clear Chance

Last clear chance is not used in most comparative negligence jurisdictions.

PRODUCTS LIABILITY: Unavoidably Unsafe Products

Manufacturers will not be held liable for some dangerous products (knives) if the danger is apparent and there is no safer way to make the product

1) "Partial" Comparative Negligence

Most comparative negligence jurisdictions will still bar the plaintiff's recovery if his negligence passes a threshold level. In some of these states, a plaintiff will be barred if his negligence was more serious than that of the defendant (i.e., the plaintiff will recover nothing if he was more than 50% at fault). In the other states, a plaintiff will be barred from recovering if his negligence was at least as serious as that of the defendant (i.e., the plaintiff will recover nothing if he was 50% or more at fault). *a) Multiple Defendants* If several defendants have contributed to plaintiff's injury, most of these states use a "combined comparison" approach to determine the threshold level (i.e., plaintiff's negligence is compared with the total negligence of all the defendants combined).

If a statute providing for a criminal penalty is applicable to a common law negligence case, a clearly stated specific duty imposed by the statute will replace the more general common law duty of care. Most courts hold that violation of an applicable statute __________.

Most courts still adhere to the rule that violation of an applicable statute is "*negligence per se*." This means that the plaintiff will have established a *conclusive presumption of duty and breach of duty by showing a violation of the statute*. (The plaintiff still must establish causation and damages to complete the prima facie case for negligence.) For a statute to apply in a negligence case: (i) The plaintiff must show that she is in the class intended to be protected by the statute; and (ii) The plaintiff must show that the statute was designed to prevent the type of harm that the plaintiff suffered. The *minority view* holds either that: (i) a rebuttable presumption as to duty and breach arises, or (ii) the statutory violation is only prima facie evidence of negligence.

P's contributory negligence is

NEVER an intervening force for proximate cause. Just treat as a defense, not as part of the main negligence analysis.

Vicarious Liability: *Bailor for Bailee*

NOT vicariously liable for tortious conduct of bailee. Can be DIRECTLY LIABLE for NEGLIGENT ENTRUSTMENT in entrusting the bailed object (NOT VL)

DEFAMATION-

Negative Statement concerning Plaintiff which hurts their reputation and it is published (3rd party hears and understands) causing damage If statement is Defamatory, look at who is being defamed: *Private person* - Def must act with NEGLIGENCE as to Truth Vs. *Public Person* -Def must act w/MALICE as to truth (i.e RECKLESS DISREGARD)

If a statute providing for a criminal penalty is applicable to a common law negligence case, a clearly stated specific duty imposed by the statute will replace the more general common law duty of care. Most courts hold that violation of an applicable statute __________.

Negligence Per Se This means that the P will have established a *conclusive presumption of duty and breach* of duty by showing a violation of the statute. The P must still establish causation and damages to complete the prima facie case for negligence. For a statute to apply: (1) class of person (2) class of risk.

What type of actual cause describes the guest's food poisoning? Two dishes were combined into one larger dish, from which the guest served herself. It is not known which of the dishes contained the bacteria, or whether both did.

No actual cause can be established. We don't know which party caused the harm, and we also don't know whether both parties were negligent.

A husband and wife took their 12-year-old son to a political rally in an auditorium to hear a controversial United States senator speak. The speaker was late, and the wife stepped outside the auditorium to smoke a cigarette. While there, she saw a man placing what she believed to be a bomb against the back wall of the auditorium. She went back inside and told her husband what she had seen. Without alerting anyone, they took their son and left. Some 20 minutes later, the bomb exploded, killing 8 persons and injuring 50. In the jurisdiction, murder in the first degree is defined as an intentional homicide committed with premeditation and deliberation; murder in the second degree is defined as all other murder at common law; and manslaughter is defined as either a homicide in the heat of passion arising from adequate provocation or a homicide caused by gross negligence or reckless indifference to consequence. As to the deaths of the eight persons, what crime, if any, did the wife commit?

No crime. The wife is not guilty of any crime because she did not have a legal duty, enforceable by the criminal laws, to warn the others about the bomb.

No affirmative duty to rescue (Exceptions)

No duty to rescue, but if you do, must take reasonable care. Exceptions: 1) if the D put the person in peril, then have a duty to act. 2) strong relationship between parties (close family; common carrier/innkeeper relationship; invitee/invitor) 3) individual has obligation to prevent 3P from causing harm

Duty toward others regarding dangerous activities on your land?

No duty to undiscovered trespassers; everyone else: reasonable care

Direct cause (proximate cause fact pattern)

No intervening cause. Nothing happens between D's bad conduct and the P's injury. Just focus on result that occurred. If it's foreseeable, we've got proximate cause and probably liability. Direct cause fact patterns usually means that the P will prevail.

An infant was injured in an automobile accident when the vehicle, driven by the infant's mother, left the roadway and rolled over down an embankment. At the time of the accident, the infant was buckled into an infant carrier car seat. The carrier was designed to snap into a base that was secured in the back seat by the rear center seat belt. Prior to driving, the mother had snapped the car carrier onto the base and pulled up on the car carrier's handle to ensure that the carrier was indeed secured in the base. When the rollover occurred, however, the carrier came loose from the base and was thrown about the inside of the vehicle, causing injuries to the infant's neck and face. The mother brought a products liability action on behalf of the child against the manufacturer of the car carrier, alleging that the manufacturer was negligent in the design of the base and seat combination. If the mother establishes at trial that the force of the rollover was enough to knock the seat loose, and that a reasonable, economically feasible alternative design existed, which of the following, if true, would be most helpful to the manufacturer's defense?

No one had reported a car carrier coming loose in a rollover prior to this accident. The most helpful fact is that no one had reported this type of problem previously. The mother is alleging that the manufacturer's negligence led to the supplying of a defective product. *To establish this, the plaintiff must show that those designing the product knew or should have known of enough facts to put a reasonable manufacturer on notice about the dangers of marketing the product as designed. Negligent design is not shown, however, if the danger of the product becomes apparent only after the product reaches the public*. Hence, the absence of any previous complaints about this problem would be most helpful to the manufacturer.

SLANDER PER SE- 4 types-

No special damages required 1)profession or trade 2)loathsome disease 3)lack of chastity of a woman 4)serious crime DEFENSES: TRUTH, PRIVILEGE

duty to prevent 3P from harming P?

No unless: 1) authority and 2) ability to control 3P. Classic example: mom has authority and ability to control toddler from knocking over shopper at grocery store.

Do you have a duty of care to everybody?

No. Only owe of duty of care to foreseeable plaintiffs within the zone of danger. EX: close eyes, throw bowling ball into crowd. Everyone in crowd is foreseeable plaintiff. But if the bowling ball flew into a vent and hit someone on the other side of the building? Not foreseeable

The personnel director of an investment company told a job applicant during an interview that the company was worth millions of dollars and that the company's portfolio would triple in the next several months. The applicant was very excited about the company's prospects and accepted an offer to work for the company. Two days later, the applicant read in the newspaper that the investment company had filed for bankruptcy reorganization. As a result of reading this news, the applicant suffered severe emotional distress but he immediately found another comparable position. Is the applicant likely to prevail in his action for negligent misrepresentation?

No, because the applicant did not suffer any physical injury or pecuniary loss. Negligent misrepresentation is similar to a standard negligence claim and also has similarities to the tort of intentional misrepresentation. To prove negligent misrepresentation, the plaintiff must show: (i) the defendant made a misrepresentation in a professional capacity; (ii) the defendant breached a duty owed to the plaintiff; (iii) the plaintiff justifiably relied on the misrepresentation; (iv) the misrepresentation caused harm to the plaintiff, and (v) damages. The applicant's claim fails on the damages element. In negligent misrepresentation claims, if the applicant suffered emotional distress and no other harm, damages for that emotional distress are not recoverable. One exception to that rule is negligent infliction of emotional distress, but most jurisdictions require a physical injury that came from the emotional distress. Here, the facts state the applicant suffered emotional distress but did not state he suffered any physical harm; the question also states that he found another position "immediately," suggesting he did not suffer pecuniary harm either.

A developer constructed several small stores in a commercial district. She received a bid from a contractor to install awnings on the front windows of the stores. The developer had heard that the contractor did shoddy work, but the price was right and the contractor expressly assumed all of the risk of any liability. The developer subsequently sold one of the stores to a barber. A few months later, an awning collapsed without warning, injuring a customer who was about to enter the barbershop. An investigation by the building inspector revealed that the awning collapsed because the brackets used by the contractor were cheaper and weaker than the required brackets, although they looked the same. The developer and the contractor are now both bankrupt. If the customer sues the barber for his injuries, is the customer likely to prevail?

No, because the barber had no opportunity to oversee the contractor's actions. The customer is not likely to prevail because the barber had no opportunity to oversee the contractor's work. A property owner owes a duty to those adjacent to the premises to take due precautions to protect them from dangerous conditions, and a business owes its customers a duty to make reasonable inspections to discover and make safe any dangerous conditions. Further, that duty cannot be delegated to an independent contractor; the owner remains vicariously liable. However, nothing in the facts establishes that the barber breached his duty to the customer. There was nothing wrong with the brackets evident from a reasonable inspection, and the awning collapsed without warning. Further, the barber was not involved in hiring or supervising the contractor and would not be responsible for the contractor's negligence. Hence, because no facts point to negligence by the barber, the customer is not likely to prevail. Although it is true that a business owner would be vicariously liable to customers and passersby injured by the negligent work of an independent contractor that he hired. Here, however, the barber did not engage the contractor and is not responsible for the contractor's conduct.

A columnist for a major metropolitan newspaper had a very antagonistic relationship with the city's mayor. When a restaurant owned by the columnist's family was shut down by city health inspectors, the columnist responded with a column publicizing the shutdown and asserting that it was in retaliation for his prior columns in which he had criticized the mayor. In fact, the mayor had nothing to do with the action by the city health inspectors. While the columnist had no evidence of the mayor's involvement, he believed that there was a connection because "that's how the city works." Can the mayor recover against the columnist for defamation?

No, because the columnist did not act with actual malice. A public official, such as a mayor, may not recover for defamatory words relating to his official conduct unless there is *clear and convincing proof that the statement was made with actual malice*, which is defined as knowledge that the statement was false or reckless disregard as to truth or falsity. Reckless conduct is not measured by whether a reasonable person would have investigated before publishing; rather, there *must be a showing that the defendant in fact (subjectively) entertained serious doubts as to the truthfulness of his publication*. Here, while the columnist had no evidence of the mayor's involvement with the action of the health inspectors, he believed that there was a connection based on his belief as to how the city operates. Hence, he has not acted with actual malice and is not liable to the mayor for defamation.

The defendant operates a residential rehabilitation center for emotionally disturbed and ungovernable children who have been committed to his custody by their parents or by juvenile authorities. Though the children are not permitted to leave the center without his permission, there are no bars or guards to prevent them from doing so. It has been held in the state where the center is located that persons having custody of children have the same duties and responsibilities that they would have if they were the parents of the children. A child, aged 12, who had been in the defendant's custody for six months, left the center without permission. The defendant became aware of the child's absence almost immediately, but made no attempt to locate him or secure his return, though reports reached him that the child had been seen in the vicinity. 36 hours after the child left the center, the child committed a brutal assault upon the plaintiff, a five-year-old child, causing the plaintiff to suffer extensive permanent injury. The defendant had no reason to suspect that the child had a propensity to attack younger children. If an action is brought against the defendant on behalf of the plaintiff to recover damages for the plaintiff's injuries, will the plaintiff prevail?

No, because the defendant did not know or have reason to know that the child had a propensity to attack younger children. A parent or legal custodian has a duty to exercise reasonable care to supervise the conduct of his or her minor child. However, this principle does not make the parent vicariously liable for the child's torts. Instead, it constitutes direct negligence by the parent not to use reasonable care in controlling the child where the parent has the ability to control the child and knows or should have known of the risk being posed by the child's conduct. Liability is generally limited to specific types of actions that were foreseeable by the custodian. Here, the fact pattern states that the defendant had no reason to suspect that the child had a propensity to attack younger children, and therefore the defendant will prevail. Just because the child was in a home for children with emotional difficulties does not mean his attack on another child was foreseeable.

A state statute prohibits leaving a child under the age of five years unattended in an automobile. A mother parked her car at a supermarket parking lot. She left her four-year-old son in the car with his seatbelt fastened while she did her grocery shopping. While the mother was shopping, the son undid his seatbelt, left the car, and started riding on the grocery carts that customers had left in the parking lot. The son crashed one of the carts into another shopper's car, causing damage. The shopper brought a negligence action against the mother to recover for the damage caused by the son. At trial, the shopper presented evidence of the statute and the facts stated above. At the conclusion of the shopper's case, the mother moved for a directed verdict in her favor. Should the court grant the mother's motion?

No, because the jury could find that it was foreseeable that the son would cause damage to cars in the parking lot if the mother left him unattended. The court should not grant the directed verdict because the jury could find that it was foreseeable that the son might cause damage to cars in the parking lot if he was left unattended. The common law rule is that a parent is not vicariously liable for the tortious conduct of her child. However, the parent may be held liable for her own negligence in allowing the child to do something that injures another's person or property. Under ordinary negligence principles, the mother owed a duty to the owners of other cars in the parking lot if it was foreseeable that her son might cause damage to them if left unattended. It is a question of fact for the jury whether she breached her duty to the other car owners by leaving her son unattended, and whether her conduct was an actual and proximate cause of the damage to the shopper's car. it is not necessary for the statute to apply to find negligence on the part of the mother. The jury could find that the mother should have known that her son might cause damage to cars if left unattended, making her liable under ordinary negligence principles without resort to the statute.

On the way home from a nightclub, a passenger began yelling at the designated driver claiming that he was not taking the best route back to her house. The driver disagreed and contended that his route was the quickest. The passenger impulsively grabbed the steering wheel, causing the car to swerve and strike a pedestrian, injuring him. At trial, the pedestrian established that the driver's license had expired the day before the accident. The driver's traffic record qualified him for an automatic renewal of his driver's license, but he had forgotten to submit it in time. A statute in the jurisdiction makes it an offense to drive a vehicle on any public road in the state without a valid driver's license. Will the pedestrian prevail?

No, because the passenger's action was the proximate cause of the injury. The pedestrian will not prevail because the passenger's grabbing of the wheel is the negligent conduct that caused the pedestrian's injuries. To establish a prima facie case for negligence, the following elements must be proved: (i) the existence of a duty on the part of the defendant to conform to a specific standard of conduct for the protection of the plaintiff against an unreasonable risk of injury; (ii) breach of that duty by the defendant; (iii) the breach of the duty by defendant was the actual and proximate cause of plaintiff's injury; and (iv) damage to plaintiff's person or property. Here, the driver's actions were an actual cause of the pedestrian's injury because, but for the driver's driving and the passenger's grabbing the steering wheel, the injury to the pedestrian would not have happened. However, the driver's actions were not a proximate cause of the injury because *the passenger's grabbing of the steering wheel was a superseding intervening force.* A superseding force is one that serves to break the causal connection between the initial wrongful act and the ultimate injury, and itself becomes a direct immediate cause of such injury. Thus, the first actor would be relieved of liability from the consequences of his antecedent conduct. The passenger's conduct in suddenly grabbing the steering wheel was an unforeseeable intervening force creating an unforeseeable harmful result, and thus constituted a superseding force. Consequently, the driver would be relieved of any negligence liability since the passenger's actions were the proximate cause of the accident.

An associate professor in the pediatrics department of a local medical school was denied tenure. He asked a national education lobbying organization to represent him in his efforts to have the tenure decision reversed. In response to a letter from the organization on the professor's behalf, the dean of the medical school wrote to the organization explaining truthfully that the professor had been denied tenure because of reports that he had abused two of his former patients. Several months later, after a thorough investigation, the allegations were proven false and the professor was granted tenure. He had remained working at the medical school at full pay during the tenure decision review process and thus suffered no pecuniary harm. In a suit for libel by the professor against the dean of the medical school, will the professor prevail?

No, because the professor invited the libel A person's interest in his reputation is protected by the tort actions for libel and slander, collectively called "defamation." To establish a case for libel or slander, the plaintiff must offer sufficient evidence of the following: (i) false and defamatory language by the defendant (i.e., reputation-damaging); (ii) that the defamatory language is "of or concerning" the plaintiff; (iii) publication of the defamatory language by the defendant to a third party; (iv) fault by the defendant that amounts to at least negligence if the defendant is of the media, and either knowledge of falsity or reckless disregard for the truth if the plaintiff is a public official or figure; and (v) in certain types of slander cases, special harm, i.e., harm of a pecuniary nature. Libel consists of all written or printed matter, meaning any communication embodied in "physical form." This may include a photograph or computer recording as well. Libel does not require the plaintiff to prove that any special harm occurred as a result of the defendant's defamatory statement. The traditional common law rule was that if the defamatory nature of the communication was apparent from the statement alone, actual harm did not need to be proved, and "presumed" damages could be awarded. Thus, a plaintiff could recover damages that would normally flow from a defamatory statement such as the one at issue, even in the absence of proof of actual or pecuniary harm. Regarding defamatory statements that involve matters of purely private concern, damages may be presumed (in contrast to matters of public concern). Even if a plaintiff establishes all the elements of a defamation action, if the defendant was protected by either an absolute or qualified privilege while making the statement, it will bar the plaintiff from recovery. For example, where the plaintiff consented to the publication of the statement, the defendant will be protected by absolute privilege. Although the professor would be able to establish a prima facie case for defamation based on libel, he will not prevail because the dean could invoke an absolute privilege of consent. The dean of the medical school wrote a false and reputation-damaging statement that the professor had reportedly abused two patients, which was published to a third party (the organization and any other individuals who read it during the subsequent investigation). However, the dean's letter was in response to the initial letter from the lobbying organization on behalf of the professor and followed by an authorization of his agents to investigate the allegation. Based on these facts, the dean could reasonably invoke a consent privilege in that he believed the professor had invited to limited publication of the statement.

A sporting goods shop was burglarized by an escaped inmate from a nearby prison. The inmate stole a rifle and bullets from a locked cabinet. The burglar alarm at the shop did not go off because the shop's owner had negligently forgotten to activate the alarm's motion detector. Shortly thereafter, the inmate used the rifle ammunition stolen from the shop in a shooting spree that caused injury to several people, including the plaintiff. If the plaintiff sues the shop's owner for the injury she suffered, will the plaintiff prevail?

No, because there is no evidence of circumstances suggesting a high risk of theft and criminal use of firearms stocked by the shop's owner The commission of a criminal act often supersedes the liability of the original negligent actor. An exception occurs if the negligent act creates a condition such that a criminal act is the foreseeable consequence of that action. Here, the shop owner didn't forget to lock his door; he just forgot to set the alarm on his gun shop. The negligence of not turning on the alarm does not contribute to the likelihood of the crime happening. In this case, the inmate broke in not knowing if the alarm was on or off. *The break-in was a superseding force that would cut off the defendant's liability.* It is not foreseeable that the breaking would happen just because there is a prison nearby unless it is a notably high crime area already

A customer fell and injured himself when he slipped on a banana peel while shopping at a grocery store. The banana peel was fresh and unblemished except for a mark made by the heel of the customer's shoe. In an action brought by the customer against the store, these are the only facts in evidence. Should the trial judge permit the case to go to the jury?

No, because there is not a reasonable basis for inferring that the store knew or should have known of the banana peel. Res ipsa loquitur is a doctrine that permits the factfinder to infer negligence when there is not direct evidence that the defendant acted negligently. To establish res ipsa loquitur, the plaintiff must show (i) the event that happened is one that usually does not occur absent the negligence of a party; (ii) the harm was caused by something in the defendant's exclusive control; and (iii) the plaintiff is not the one who caused the event to occur. Courts generally apply the "exclusive control" element liberally. If the above elements are met, res ipsa loquitur creates an inference of negligence. Here, however, these elements are not met. First, the facts do not state the plaintiff presented evidence establishing his fall was not attributable to him (i.e. that he did not drop the peel) or that the store aisles are in the store's exclusive control (customers, from minute-to-minute, also affect the state of the aisles significantly). He has therefore not met the requirements for using res ipsa loquitur. Without the res ipsa loquitur doctrine, the plaintiff is left with no evidence that the defendant store failed to comply with its standard negligence duty owed to customers: to inspect and make safe the premises as to any nonobvious, dangerous conditions. Further, note that this question states that the banana peel was "fresh and unblemished" to convey that there is no evidence the store had time to comply with its duty. The store has a duty to keep its aisles clean and safe, but it does not have a duty to do so instantly upon any debris or hazard forming. The customer's failure to watch where he stepped is contributory negligence, but the MBE instructs test-takers to apply pure comparative negligence. Under that doctrine, a negligent plaintiff may still recover against a negligent defendant. Thus, this answer choice does not provide a valid basis for preventing the case from going to the jury.

Duty to Aid (Rescue) Is there a duty to rescue?

No. EX: Can you watch someone drown? Yes. But, if you get involved and start to render aid, you owe reasonable care under the circumstances. Don't lose common sense. They will tell you what is reasonable.

Duty Owed Discovered Trespassers

Once a landowner discovers the presence of a trespasser, he is under a duty to exercise ordinary care to warn the trespasser of, or to make safe, artificial conditions known to the landowner that involve a risk of death or serious bodily harm and that the trespasser is unlikely to discover. *warn of man-made death traps* There is no duty owed for natural conditions and less dangerous artificial conditions. The owner or occupier also has a duty to *exercise reasonable care in the exercise of "active operations" on the property*.

Defense of Others

One may use force to defend another when the actor reasonably believes that the other person could have used force to defend himself Reasonable mistake allowed Only reasonable force; the same as he could have used in self-defense

Assumption of Duty to Act by Acting

One who gratuitously acts for the benefit of another, although under no duty to do so in the first instance, is then under a duty to act like an ordinary, prudent, reasonable person and continue the assistance. Example: Defendant, under no duty to aid Plaintiff who has been injured, picks her up and carries her into a room. He then leaves her there unattended for seven hours and Plaintiff's condition is worsened. Defendant, having acted, may be considered to have breached his duty to act reasonably.

Peril Due to Defendant's Conduct

One whose conduct (whether negligent or innocent) places another in a position of peril is under a duty to use reasonable care to aid or assist that person.

Duties of a Lessor of Realty General Duty Rule

Ordinarily, tort liability in regard to conditions on the property is an incident of occupation and control. Thus, when the owner leases the entire premises to another, the lessee, coming into occupation and control, becomes burdened with the duty to maintain the premises in such a way as to avoid unreasonable risk of harm to others. Similarly, where the owner leases portions of the premises to tenants, the *owner continues to be subject to liability for common areas* as a landowner for unreasonably dangerous conditions in those portions of the premises such as corridors, entry lobby, elevators, etc., used in common by all tenants, or by third persons, and over which the owner has retained occupation and control. THERE ARE EXCEPTIONS!

Ameliorating Doctrine for Contributory Negligence Last Clear Chance -

P can recover despite contributory negligence if D had last clear chance, but failed to do so

contributory negligence

P fails to use relevant standard of care to protect self from foreseeable risks of harm use same analysis as D's negligence. old rule: complete bar to recovery. Exam will say "traditional contributory negligence rules"

"Of and Concerning" Plaintiff

P must establish that a reasonable listener, reader, viewer would under stand that the defamatory statement referred to the P. *Colloquium* - if the statement does not refer to P on its face, extrinsic evidence may be offered to establish that the statement refers to the P. This is known as pleading "colloquium"

Avoidable Consequences Distinguished from Contributory Negligence

P's duty to mitigate damages to person or property after the damage is inflicted is an avoidable consequence, not contributory negligence

Contributory Negligence

P's failure to use the relevant degree of care for his/her own safety. [P's standard of care might vary if a child, or violated a statute, etc.] Have to determine standard of care, then if P failed to live up to it. Traditional: absolute bar to recovery. Very harsh. - EX: Jaywalker hit by drunk driver. Jaywalker gets nothing even if injured for life.

2) Partial Comparative Negligence Jurisdiction—Multiple Defendants Plaintiff is 40% negligent in causing the accident and suffers $100,000 in damages. D1 is 35% negligent and D2 is 25% negligent.

Plaintiff can recover $60,000 from either D1 or D2 under joint and several liability rules. Note that if D1 or D2 also suffered damages, each of them would have a claim against the other two negligent parties because each one's negligence is less than the total negligence of the other two.

3) Pure Comparative Negligence Jurisdiction Plaintiff is 30% negligent and Defendant is 70% negligent in causing the accident. Each party suffers $100,000 in damages.

Plaintiff has a right to recover $70,000 from Defendant, and Defendant has a right to recover $30,000 from Plaintiff. Defendant's damages will be *offset* against Plaintiff's damages, and Plaintiff will have a net recovery of $40,000.

Damages Recoverable in the Action Personal Injury

Plaintiff is to be compensated for all his damages (past, present, and prospective), both special and general. This includes fair and adequate compensation for economic damages, such as medical expenses and lost earnings, and noneconomic damages, such as pain and suffering. Plaintiff is also entitled to compensation for impaired future earning capacity, discounted to *present value* so as to avoid an excess award; i.e., plaintiff receives an amount that, if securely invested, would produce the income that the jury wishes him to have. *1) Foreseeability Irrelevant* As noted above in the proximate cause section, it is generally not necessary to foresee the extent of the harm. In other words, a tortfeasor takes the victim as he finds him. *2) Emotional Distress Damages* Plaintiff's noneconomic damages include damages for any emotional distress suffered as a result of the physical injury. Example: Plaintiff was struck by a piece of metal when the engine blew on a defectively manufactured lawnmower. The piece of metal lodged in his spine at an inoperable location, significantly increasing his risk of future paralysis. In plaintiff's products liability action against the manufacturer of the lawnmower, plaintiff can recover damages not only for his physical injury but also for the emotional distress he suffers from his knowledge of the risk of paralysis, because it arises out of the physical injury caused by the defective product.

Plaintiff's Freedom from Negligence in Res Ipsa

Plaintiff must also establish that the injury was not attributable to him, but may do so by his own testimony.

DUTY OF CARE Prenatal Injuries - actionable?

Prenatal injuries are actionable; i.e., a duty of care is owed toward a fetus. The *fetus must have been viable* at the time of injury. (Most states also permit a wrongful death action if the fetus dies from the injuries.)

Punitive Damages

Punitive damages generally are not available in negligence cases. However, if the defendant's conduct was "wanton and willful," reckless, or malicious, most jurisdictions permit recovery of punitive damages

If the person is a foreseeable plaintiff, what is the general duty of care?

Reasonably prudent person.

A consumer purchased a grass trimmer from a hardware store. He took it out of the box and assembled it according to the instructions. He noticed that there were bolts and screws left over and some joints that could have accepted additional fasteners, but he just discarded the extra hardware. As he was using the trimmer, the housing came apart and a hard piece of plastic flew off. His neighbor, who was standing nearby, was struck in the eye by the piece of plastic and suffered permanent injuries. The neighbor sued the hardware store and the manufacturer of the trimmer in a strict liability action. Through discovery, it was determined that the instructions omitted a critical step in the assembly process that would have used the extra hardware, which is why the housing came apart, and that the manufacturer had received some complaints about the instructions previously. The hardware store had no knowledge of any complaints regarding any of the manufacturer's products. As to the hardware store, the neighbor will:

Recover, because the consumer's failure to recognize the improper assembly does not cut off the store's liability. The neighbor will recover against the hardware store. In a products liability action based on strict liability, the plaintiff need show only (i) the defendant is a commercial supplier, (ii) the defendant produced or sold a product that was defective when it left the defendant's control, (iii) the defective product was an actual and proximate cause of the plaintiff's injury, and (iv) the plaintiff suffered damages to person or property. Here, the hardware store is in the chain of supply of the product. The product was defective because the instructions omitted a critical part of the assembly process. Finally, the omission was an actual and proximate cause of the neighbor's injury, allowing him to recover against the hardware store.

Duty of Care: Children Engaged in Adult Activities

Where a child engages in a potentially dangerous activity that is normally one that only adults engage in, most cases hold that he will be required to conform to the same standard of care as an adult in such an activity, e.g., driving an automobile, flying an airplane, driving a motorboat.

A court likely will not permit a defendant to raise an assumption of risk defense when:

Risk has been assumed by a disclaimer from a common carrier. Remember: AOR can be used EVEN when P's injury is caused by D's wonton and reckless conduct.

5) INTENTIONAL TORTS: Conversion-

SAME AS TRESPASS TO CHATTELS, BUT DAMAGE IS ENTIRE VALUE 1) act that interferes with P's right of possession in a chattel 2) intent 3) causation, and 4) damages - an interference *serious enough in nature or consequences to warrant that defendant pay the chattel's full value at the time/place of conversion (NOT replacement value)* *Acts of Conversion* - theft, wrongful transfer, wrongful detention, substantially changing, severely damaging, or misusing *Intent required* - mistake as to ownership is no defense; the only intent required is to do the act that interferes *Seriousness of interference* - the longer the withholding period and more extensive the use, the more likely it is to be conversion. Less serious interference is trespass to chattels. *Subject Matter* - only physical property and intangibles reduced to physical form (promissory note) *Damages* - full market value or possession (replevin)

What is the difference between malicious prosecution and wrongful civil proceedings?

SAME ELEMENTS. *Lack of probable cause harder to prove* in civil actions because reasonable people would more readily file a doubtful case where the only consequences to the person sued are civil. Only needs to believe there is a respectable chance of winning In order to have probable cause for bringing a civil action, an attorney must 1) make a reasonable investigation of legal authority and 2) must thereafter retain an honest belief that the claim is tenable. *Some courts add the requirement of "special injury"* Special injury = injury caused by the seizure of the π's person or property

Difference between fraudulent & negligent misrepresentation

STATE OF MIND - D negligent in believing representation was true vs. deceit CONSEQUENCES - negligent misrep: only liable to those she intended to reach; -fraudulent: all those likely to rely on statement DAMAGES - negligent: reliance (out-of-pocket); consequential - if proven - fraudulent: expectation

Products Liability Based on Strict Liability

STRICT PRODUCTS LIABILITY: Liability arises from supplying a defective product *even if the defendant exercised due care and was not negligent* ---*Even a retailer who had no opportunity to inspect* the product may be liable as a commercial supplier Privity NOT required *Does NOT extend to services* (even where a product is provided incident to a service) *DISCLAIMERS INEFFECTIVE* - in negligence or SL cases if personal injury or property damages occur

Actual Cause: 1) multiple defendants; 2) commingled cause

Substantial Factor test for actual causation Two or more Ds do something negligent that creates destructive forces (EX: fires); those forces merge into a single destructive force that harms the P. *Substantial Factor TEST*: whether the D's conduct was a "substantial factor" in the P's harm. Common examples: - pollution cases: two Ds both pollute water/air that harms P.

A housecleaning agency was given a key to a customer's house so that the agency could have its employees clean while the homeowner was away. After a maid sent by the agency had finished and left the homeowner's house, she went back because she had forgotten her cigarettes. She neglected to lock the door when she left the second time because she was already late for the next job. When the homeowner returned after a few days away, she discovered that her house had been ransacked and several items of jewelry stolen. The front door was open, and there were no signs of forced entry. If the homeowner brings an action against the agency that employed the maid, what is the likely result?

She will prevail, because the maid's failure to lock the door created the risk that someone might enter and take the homeowner's valuables. The homeowner will prevail because the maid's negligence increased the risk of criminal conduct by a third party. Criminal acts and intentional torts of third persons are foreseeable independent intervening forces if the defendant's negligence created a foreseeable risk that they would occur. Here, the maid's failure to lock the door was negligent because it created a risk of burglary; hence, the burglary does not cut off the agency's liability for the maid's negligence. As the maid's employer, the agency is vicariously liable under respondeat superior.

Actual Cause: Multiple Defendants, UNKNOWN Cause (AKA "Alternative Causes" Appraoch)

Shift the burden of proof to the Ds. Summers v. Tice.

Shopkeeper's privilege as defense to battery

Similarly, a shopkeeper and his employees are privileged to use reasonable force to detain a person whom they reasonably believe has stolen the shopkeeper's merchandise.

What effect does comparative negligence have on implied assumption of risk?

States that have adopted comparative negligence, usually apply it to all types of fault by the P, including implied assumption of risk. [Express AOR stays the same because it's a contractual thing] But they replace implied assumption of risk with comparative negligence. Have to look at whether the P's conduct was unreasonable. If the P proceeded in the face of a known danger, not only voluntarily but unreasonably, you analyze it just like comparative negligence by calculating the respective fault of the parties.

While practicing their target shooting at the firing range, a man and woman got into an argument that almost erupted into physical combat, except that they were restrained and separated by bystanders. Later, in the parking lot of the range, the man shot the woman in the shoulder. Bystanders who rushed to the scene immediately after hearing the man's shot found the woman on the pavement with a black flashlight in her hand. The woman's pistol was in her locker at the firing range. At the trial of the woman's civil action for battery against the man, the woman established that the man intentionally shot her. In defense, the man testified that the woman approached him, saying, "We'll settle this once and for all, right now," and raised an object toward the man. He testified that he feared that the woman was about to shoot him with a pistol, so he fired in self-defense. Assuming that the jury decides that the man is telling the truth, what else must the jury find for him to prevail?

That a reasonable person in the same circumstances would have believed that the woman was about to shoot. *OBJECTIVE STANDARD!* If the man prevails, it will be because the jury determined that he acted reasonably under the circumstances. One may act in self-defense not only where there is real danger but also where there is a reasonable appearance of danger. An honest but mistaken belief that the woman was about to shoot would justify the use of deadly force by the man if a reasonable person would have acted similarly under those circumstances. The test is an objective one-an honest belief alone is not sufficient.

A plaintiff was injured when the steering mechanism of a snowmobile failed. He brought a negligence action against the snowmobile manufacturer. The steering mechanism was designed and manufactured by a component manufacturer; the snowmobile manufacturer merely assembled the snowmobile, branded it, and distributed it directly to retailers. To prevail against the snowmobile manufacturer, what will the plaintiff need to prove?

That the steering mechanism was in a defective condition unreasonably dangerous to users, and the defect could have been discovered and corrected if the component manufacturer had exercised reasonable care in its quality control process. The plaintiff will prevail against the snowmobile manufacturer if the component manufacturer was negligent in not discovering and correcting the defect. To establish a prima facie case of negligence in a products liability case, the plaintiff must show: (i) the existence of a legal duty owed by the defendant to that particular plaintiff; (ii) breach of that duty; (iii) actual and proximate cause; and (iv) damages. The duty of care arises when the defendant acts as a commercial supplier of products. A commercial supplier who assembles a product from components manufactured by others is subject to the same liability as the manufacturer of the defective component. To prove breach of duty, the plaintiff must show (i) negligent conduct by the defendant that leads to (ii) the supplying of a defective product. Here, because the snowmobile manufacturer assembled the snowmobile from component parts, including the steering mechanism manufactured by the component manufacturer, and sold the snowmobile as its own product, it will be liable for the negligence of the component manufacturer. Hence, if the steering mechanism was defective and the component manufacturer could have discovered and corrected the defect in the exercise of reasonable care, as (C) states, the snowmobile manufacturer has breached its duty to the plaintiff, and this breach of duty caused the plaintiff's injuries.

Concurrent Causes

The "but for" test applies where several acts combine to cause the injury, but none of the acts standing alone would have been sufficient (e.g., two negligently driven cars collide, injuring a passenger). But for any of the acts, the injury would not have occurred.

2) "Pure" Comparative Negligence

The "pure" variety of comparative negligence, adopted in a third of the comparative negligence states, allows recovery no matter how great plaintiff's negligence is (e.g., if plaintiff is 90% at fault and defendant 10%, plaintiff may still recover 10% of his damages). *On the MBE, pure comparative negligence is the applicable rule unless the question specifies otherwise.*

*Compare — "Wrongful Birth" and "Wrongful Pregnancy"* -

The *child's parents, however, DO have an action*: either for *failure to diagnose the defect ("wrongful birth")* or for *failure to properly perform a contraceptive procedure ("wrongful pregnancy")*. The mother can recover damages for the unwanted labor (medical expenses and pain and suffering). If the child has a defect, parents may recover the additional medical expenses to care for the child and, in some states, damages for emotional distress. If the child is born healthy in a wrongful pregnancy case, most cases do *not* permit the parents to recover child-rearing expenses, just damages for the unwanted labor.

What Is Applicable Standard of Care? Basic Standard—

The Reasonable Person Defendant's conduct is measured against the reasonable, ordinary, prudent person.

Where is the attractive nuisance doctrine applied?

The attractive nuisance doctrine has been applied to abandoned automobiles, lumber piles, sand bins, and elevators. Bodies of water are generally not dangerous conditions because the dangers are viewed as obvious and well-known. If, however, a body of water contains elements of unusual danger to children, it may be characterized as a dangerous condition, e.g., logs or plants floating in the water, or a thick scum that appears to be a path on the water.

To fight drug abuse, a state enacted a statute forbidding the selling of model airplane glue to anyone under the age of 18 except in small quantities in prepackaged model kits. Violation of the statute was penalized by fines or, in cases of multiple violations, possible imprisonment. The statute also required that all elementary and secondary schools licensed by the state provide comprehensive drug education programs. Neither the legislature nor the courts of the state have abolished the common law tort defense of assumption of the risk. The owner of a hobby shop in the state sold a large tube of airplane glue to a 15-year-old boy who reasonably appeared to be at least 18 years old. The boy had received drug education in his school, as mandated by the statute, including coverage of the dangers of glue sniffing. The boy understood the anti-drug instruction, but he wanted to experience it for himself. The boy sniffed the glue repeatedly and suffered permanent brain damage. If the boy's parents file suit on the boy's behalf against the store owner, for whom is the court likely to rule?

The boy will prevail because the store owner's violation of the statute resulted in the boy's injuries. The applicable standard of care in a cause of action can be established by proving the applicability to that action of a statute providing for a criminal penalty. If this is done, a clearly stated specific duty imposed by the statute will replace the more general common law duty of care. For the statutory standard to be applicable, the plaintiff must show that (i) he is within the class intended to be protected by the statute, and (ii) the statute was designed to prevent the type of harm that the plaintiff suffered. [aka "negligence per se"] Here, the statute clearly prohibited the transaction that took place between the boy and the store owner. It establishes the standard of care because the boy is a minor purchasing a large quantity of model airplane glue, and the serious injury he suffered from sniffing the glue was one of the harms that the statute was designed to prevent. The effect of establishing a violation of the statute is that a conclusive presumption of duty and breach of duty is established. The boy should then be able to establish that the store owner's sale of the glue was the actual cause and proximate cause of the boy's injuries, completing the prima facie case of negligence on the store owner's part. The boy's *assumption of the risk is not a defense under these circumstances*. The facts indicate that the state has retained the common law tort defense of assumption of the risk. Under this defense, a plaintiff will be denied recovery in a negligence action if he either expressly or impliedly knew of the risk of injury and voluntarily proceeded in the face of the risk. However, courts refuse to permit an assumption of risk defense in some situations because of public policy considerations. *When a statute applies and is enacted to protect a class, members of that class will not be deemed to have assumed any risk*. Here, even though the boy was aware of the danger when he voluntarily sniffed the glue, the statute was enacted to protect minors such as the boy from the dangers of glue sniffing. Thus, the store owner cannot rely on assumption of risk as a defense

Under the alternative causes approach to actual causation in negligence:

The burden of proof shifts to the defendants after the plaintiff establishes that one of them caused the harm

Res Ipsa Loquitur

The circumstantial evidence doctrine of res ipsa loquitur ("the thing speaks for itself") deals with those situations where the fact that a particular injury occurred may itself establish or tend to establish a breach of duty owed. Where the facts are such as to strongly indicate that plaintiff's injuries resulted from defendant's negligence, the trier of fact may be permitted to infer defendant's liability.

Which of the following is not a prerequisite for the plaintiff to rely on the doctrine of res ipsa loquitur?

The defendant had actual possession of the instrumentality causing the injury. it is not necessary for application of res ipsa loquitur to show that the D had actual possession of the instrumentality causing the injury, even though this may be one way to connect the D with the negligence that occurred. The circumstantial evidence doctrine of res ipsa loquitur deals with those situations where the fact that a particular injury occurred may itself establish or tend to establish a breach of duty owed. Res ipsa loquitur requires that: the accident of a type that normally does not occur in the absence of someone's negligence; the evidence connects the D to the negligence (i.e. this type of accidental ordinarily happens because of the negligence of someone in the D's position); and the injury was not attributable to the plaintiff or any 3P

A state child safety statute required children under eight years of age to be in a government-approved car seat when riding in a motor vehicle. A father was driving to a ballgame with his seven-year-old child, who was buckled in the back seat with a regular seat belt. The father did not notice when the child unbuckled himself and started climbing into the front seat. The child grabbed the steering wheel "to help daddy steer," causing the car to swerve into the other lane and collide with another motorist's car. The motorist was seriously injured from the collision; the father and his child were unhurt. The motorist sued the father to recover damages for her injuries. At trial, the motorist presented evidence of the statute, her injuries, and the facts stated above. At the conclusion of the proofs, both parties moved for a directed verdict. How should the trial judge proceed?

The court should deny both motions and submit the case to the jury, because the jury could find that the father was negligent in not preventing his child from grabbing the steering wheel. Under ordinary negligence principles, the father owed a duty to other motorists to maintain control of his vehicle. The jury could find that the father breached this duty of care by not noticing when his child unbuckled the seat belt and started climbing into the front seat, and not preventing the child from grabbing the steering wheel. Hence, the case should go to the jury for a determination of whether the father was negligent. The *father is not vicariously liable for his child's negligent conduct at common law*. Any liability of the father in this case would arise from his own potential negligence in failing to control his child while driving.

A homeowner purchased a riding lawn mower from a lawn mower dealer. During his first use of the mower, the homeowner noticed that the mower was vibrating when he turned, but he was able to finish mowing. A few days later, the homeowner lent the mower to his neighbor. The neighbor was driving the mower back to his yard when he made a turn and a wheel broke off, causing the neighbor to be thrown off the lawn mower and onto the sidewalk. The neighbor was injured. The neighbor brought a *negligence action* against the dealer for his injuries. At trial, the neighbor presented evidence that the wheel broke because of a manufacturing defect. The dealer presented evidence that the homeowner could have discovered the defect after the mower began vibrating when he used it for the first time. In this action, who is likely to prevail?

The dealer, because there is no evidence that the dealer had reason to know that the lawn mower was defective. To prove breach of duty in a products liability action based on negligence, the plaintiff must show (i) negligent conduct by the defendant leading to (ii) the supplying of a defective product by the defendant. However, a dealer who buys from a reputable manufacturer with no reason to anticipate that the product is dangerous need make only a cursory inspection of the goods to avoid liability for manufacturing defects. Here, there is no evidence that the dealer should have known that the wheel was defective; hence the dealer will likely prevail.

In a civil action, a plaintiff sued a decedent's estate to recover damages for injuries she suffered in a collision between her car and one driven by the decedent. At trial, the plaintiff introduced undisputed evidence that the decedent's car had swerved across the centerline of the highway into oncoming traffic, where it had collided with the plaintiff's car. The decedent's estate introduced undisputed evidence that, before he swerved across the centerline, the decedent had suffered a fatal heart attack, which he had no reason to foresee, and that, just prior to the heart attack, the decedent had been driving at a reasonable speed and in a reasonable manner. A statute makes it a traffic offense to cross the median of a highway. In this case, which party is likely to prevail?

The decedent's estate, because its rebuttal evidence is undisputed The plaintiff's evidence that the decedent violated the statute and crossed over the centerline establishes a prima facie case of negligence. However, the prima facie case of negligence may be rebutted by showing that compliance with the statute was beyond the defendant's control. Here, the decedent's estate successfully rebutted the plaintiff's evidence by providing an undisputed explanation of how the accident happened that is inconsistent with a finding of negligence (the decedent's unforeseeable heart attack made her unable to comply with the statute or, indeed, with any standard of care).

Misfeasance

The incorrect, improper or wrongful performance of a lawful act.

Which of the following is relevant for the defendant in defending a strict products liability claim? A) It was impossible for the defendant retailer to inspect the product because it was in a sealed container. B) The defendant retailer made a reasonable inspection of the product before selling it but did not discover the defect. C) The retailer of the defendant manufacturer's product discovered the defect during the course of an inspection but failed to warn the buyer. D) The retailer of the defendant manufacturer's product could have discovered the defect during a reasonable inspection but failed to make an inspection.

The defendant manufacturer has a defense if the retailer discovered the defect during the course of an inspection but failed to warn the buyer. The same concepts of proximate cause governing general negligence and strict liability actions are applicable to strict liability actions for defective products. As with products liability cases based on negligence, the negligent failure of an intermediary to discover the defect does NOT cut off the supplier's strict liability. But *when the intermediary's conduct becomes something more than ordinary foreseeable negligence, it becomes a superseding cause*. The manufacturer can argue that the retailer's failure to take action after discovering a defect was not foreseeable and therefore cuts off the manufacturer's liability for the defect. The fact that the retailer could have discovered the defect during a reasonable inspection but failed to make any inspection would generally not be relevant to the manufacturer's defense; that typically would be considered ordinary foreseeable negligence which does not cut off the manufacturer's liability. Whether the retailer made a reasonable inspection of the product or that it was impossible to inspect the product are irrelevant to the retailer's defense. A retailer in a strict liability action may be liable for a manufacturing or design defect simply because it was a commercial supplier of a defective product.

Vicarious Liability

The defendant may be vicariously liable for the tort of another based on the *relationship* between the defendant and the tortfeasor Note that, regardless of whether vicarious liability applies, the defendant can be liable for his own negligence, e.g., negligence in hiring the employee, supervising the child, or entrusting the car to the driver

In a torts essay question on actual cause, what small wording change can make a big difference in showing you understand the concept?

The defendant's *negligent conduct* was *an* actual cause of the P's injury. Because there are many actual causes; this shows you are focusing on the negligent conduct at issue.

Last Clear Chance

The doctrine of last clear chance, sometimes called "the humanitarian doctrine," permits the plaintiff to recover despite his own contributory negligence. Under this rule, the person with the last clear chance to avoid an accident who fails to do so is liable for negligence. (In effect, last clear chance is plaintiff's rebuttal against the defense of contributory negligence.) Example: Bowater negligently parked his car on the railroad tracks. The train engineer saw him in time to stop but failed to do so. The engineer had the last clear chance, and thus the railroad will be liable for the accident.

An impatient driver who was fed up with jaywalking pedestrians drove straight at one of them, leaning on the horn and intending to make her jump. The pedestrian did not hear him or change her pace, however, because her music player was turned to full volume. A bystander on the curb rushed out to pull her to safety. She tripped as she was being pulled to the curb, fracturing her kneecap. If the pedestrian sues the driver for assault, what will be the likely result?

The driver wins, because the pedestrian did not know at the time that she was in danger from the driver. The driver is not liable for assault because he did not cause the pedestrian to reasonably apprehend an immediate harmful contact. The prima facie case for assault requires (i) an act by defendant causing a reasonable apprehension in plaintiff of immediate harmful or offensive contact to plaintiff's person, (ii) intent by defendant to bring about in plaintiff apprehension of that contact, and (iii) causation. For there to be apprehension, plaintiff must be aware of defendant's act at the time that it is occurring. Here, because the pedestrian was oblivious to the driver's attempt to scare her, the driver is not liable for assault.

An engineer licensed by the state was the principal design engineer for a wastewater treatment plant's aeration system. Detailed recommendations for designing aeration systems for this type of plant had been published by a panel of engineers after lengthy study. The engineer fully complied with the recommendations in his design. Nevertheless, the treatment plant's aeration system suffered a major failure, causing the release of bacteria-laden water into a river that damaged a fish hatchery run by the plaintiff. If the plaintiff sues the engineer and prevails, what is the likely explanation?

The engineer knew of a better design that he could have used that would have prevented the failure. If the plaintiff prevails, it will be because the engineer did not use a better design that would have prevented the failure. A person who is a professional is required to exercise such superior judgment, skill, and knowledge as he actually possesses. If the engineer knew of and could have used a better design for the aeration system, he has breached his duty as a professional. If the better design would have prevented the failure of the aeration system, as choice (A) states, this breach was an actual and proximate cause of the damage suffered by the plaintiff.

Standard of Care in Emergency Situations

The existence of an emergency, presenting little time for reflection, may be considered as among the circumstances under which the defendant acted; i.e., he must act as the reasonable person would under the same emergency. The emergency may *not* be considered, however, *if it is of the defendant's own making.*

Independent Intervening Forces Independent intervening forces also operate on the situation created by defendant's negligence but are independent actions rather than natural responses or reactions to the situation. Independent intervening forces may be foreseeable where defendant's negligence increased the risk that these forces would cause harm to the plaintiff.

The following are common fact situations involving independent intervening forces: *(1) Negligent Acts of Third Persons* Defendant is liable for harm caused by the negligence of third persons where such negligence was a foreseeable risk created by defendant's conduct. Example: D negligently blocked a sidewalk, forcing P to walk in the roadway, where he is struck by a negligently driven car. D is liable to P. *(2) Criminal Acts and Intentional Torts of Third Persons* If defendant's negligence created a foreseeable risk that a third person would commit a crime or intentional tort, defendant's liability will not be cut off by the crime or tort. Example: D, a parking lot attendant, negligently left the keys in P's car and the doors unlocked when he parked it, allowing a thief to steal it. D is liable to P. *(3) Acts of God* Acts of God will not cut off defendant's liability if they are foreseeable. Example: D, a roofer, negligently left a hammer on P's roof at the end of the day. P is struck by the hammer when a strong wind blows it off the roof. D is liable to P.

A tenant moving into a new apartment bought a spool of "10-pound test" fishing line, manufactured by a fishing tackle and accessories company, for the purpose of hanging pictures, all of which she knew weighed less than 10 pounds. The spool came with no guidelines or warnings about using it for hanging objects. She attached the fishing line to either end of the pictures and hung them on hooks on the wall. The next week, a friend visiting the tenant was hit and injured while sitting on the couch by a picture that fell when the fishing line broke. It is common knowledge in the sporting goods industry that "10-pound test" indicates that the line will stand a pull of 10 pounds, but is not intended to support a 10-pound weight over a period of time. However, it is also common knowledge in the industry that the public in general uses fishing line to support heavy hanging objects over a period of time. The friend brought a products liability action based on strict liability against the tackle company for damages caused by his injury. What is the most likely result?

The friend will win, because the label did not warn the consumer against relying on the term "10-pound test" for purposes other than fishing The most likely result is that the friend will prevail on grounds of an information defect due to the inadequate warnings. If a product presents an unreasonable risk of injury to users and bystanders, the fact that there is no sufficient warning of the danger may be a dangerous defect that will invoke strict products liability. The facts establish that the fishing tackle company knew that purchasers of its fishing line commonly used that line to support heavy hanging objects, and that the line would not support the weight rating in that type of use. Thus, the fishing tackle company's failure to provide a warning may be a defect that will permit the friend to recover in strict liability.

Duty of Possessor to Those Off the Premises a) Natural Conditions

The general rule is that a landowner owes *no duty* to protect one outside the premises from natural conditions on the land. Example: One is not liable for bugs that live in trees on one's land but that "visit" the neighbors from time to time. Note: An *exception exists for decaying trees next to sidewalks or streets in urban areas.*

A golfer and her instructor were playing golf in a foursome when the golfer became very annoyed with critical comments made by the instructor. To show the other golfers in the group how annoyed she was with her instructor, the golfer stood a few yards behind him while the instructor was teeing off and swung a club at him. The instructor, who was focusing on his shot, was not within range of the club but unfortunately the club slipped out of the golfer's hands and struck the instructor in the head, injuring him. If the instructor brings a battery action against the golfer, will he recover?

The golfer will not be liable because she did not intend to cause harmful or offensive contact. The prima facie case for battery has the following elements: (i) an act by the defendant that brings about harmful or offensive contact to the plaintiff's person; (ii) intent on the part of the defendant to bring about harmful or offensive contact to the plaintiff's person; and (iii) causation. Here, the golfer did not have the intent to cause harmful or offensive contact. Under the transferred intent doctrine, an intent to cause an assault (intent to cause apprehension of imminent harmful or offensive contact) will satisfy the intent requirement for battery when the other elements of battery are present. Here, however, the golfer was standing behind the instructor and was intending only to show the other golfers how annoyed she was. No intent to commit assault is apparent here.

The plaintiff was driving inattentively when she had to swerve to avoid two other negligently driven vehicles at a busy intersection, and her car struck a light pole. The plaintiff, who was the only driver injured, sued one of the other drivers to recover damages in a jurisdiction that has adopted pure comparative negligence. The jury determined that she suffered injuries of $100,000 and was 50% at fault. If the plaintiff is awarded a recovery of only $25,000 from the defendant, what will be the most likely reason?

The jurisdiction has abolished joint and several liability. If the plaintiff recovers only $25,000, it will be because the jurisdiction has abolished joint and several liability. Under joint and several liability, when two or more tortious acts combine to proximately cause an indivisible injury to a plaintiff, each tortfeasor is liable to the plaintiff for the entire damage incurred. Hence, the plaintiff could recover $50,000 from the defendant if joint and several liability applied (her total damages reduced by the amount of her own fault that contributed to her injury). The facts do not state what percent of fault was assigned to the defendant, but given that the other choices are incorrect, the defendant's fault must have been determined to be 25%, so that $25,000 would be the plaintiff's recovery in the absence of joint and several liability.

Duty owed to invitee

The landowner owes an invitee a *general duty to use reasonable and ordinary care in keeping the property reasonably safe* for the benefit of the invitee. This general duty *includes the duties owed to licensees* (to warn of or make safe nonobvious, dangerous conditions known to the landowner and to use ordinary care in active operations on the property) *plus a duty to make reasonable inspections* to discover dangerous conditions and, thereafter, make them safe. *(a) Warning May Suffice* The requirement to "make safe" dangerous conditions usually is satisfied if a reasonable warning has been given. *(b) Obviousness of Danger* A duty to warn usually does not exist where the dangerous condition is so obvious that the invitee should reasonably have been aware of it. "Obviousness" is determined by all of the surrounding circumstances. Example: A banana peel visible on the floor of a supermarket might not be considered obvious if a shopper's attention would likely be diverted by shelf displays.

Tortious Interference with Family Relationships Loss of Consortium: Husband/Wife

The loss by one spouse of the other spouse's companionship, services, or affection. A married person can sue for loss of consortium as an additional claim when spouse is injured. Designed to compensate for: 1) loss of household services 2) loss of society: companionship 3) loss of sex Derivative action, thus any defense that would reduce or bar recovery by the injured family member also reduces/bars recovery for interference with the family relationship

Duty Owed Anticipated Trespassers

The majority of states now treat anticipated trespassers on generally the *same basis as discovered trespassers* in terms of the duty owed them by the landowner. *When Is a Trespasser "Anticipated"?* -An "anticipated trespasser" situation arises where the landowner knows or should reasonably know of the presence of trespassers who constantly cross over a section of his land. (Although note that if the owner has posted "no trespassing" signs, this might serve to convert these "anticipated" trespassers into "undiscovered" trespassers.)

A man purchased a new power boat with an inboard engine from a boating supply store. The boating supply store properly inspected the boat before delivery, but did not detect a virtually invisible manufacturing defect in the boat's steering mechanism. Later that summer, the man was entertaining some friends on his boat on a lake near a dam. There were some warning pylons near the dam, warning boaters to stay clear. The man decided to show off for his friends by weaving his boat in and out of the warning pylons. As he rounded the last of them, the steering mechanism of his boat jammed, and the boat crashed into the dam. The man was severely injured. The man brings an action for damages against the boating supply store on a theory of strict liability in tort in a jurisdiction that does not apply its comparative fault rules to strict liability actions. Who will prevail?

The man, because the steering failed due to a defect present when the boat left the manufacturer. To recover on a theory of strict tort liability, the man must show that his injuries were caused by an unreasonably dangerous defect in the boat that existed when the boat left the boating supply store's control. A prima facie case in products liability based on strict tort liability consists of the following: (i) the defendant is a commercial supplier; (ii) the defendant produced or sold a product that was defective when it left the defendant's control; (iii) the product was the actual and proximate cause of the plaintiff's injury; and (iv) the plaintiff suffered damages to person or property. Examples of commercial suppliers include manufacturers, retailers, wholesalers, and assemblers. The second element is established by proving that the product is in a defective condition unreasonably dangerous to users. A plaintiff need not prove that the defendant was at fault in selling or producing a dangerous product. To prove actual cause, a plaintiff must trace the harm suffered to a defect in the product that existed when the product left the defendant's control. Here, because the steering failed due to a defect present when the boat left the manufacturer, that defect must also have been present when the man bought the boat from the boating supply store, the retailer. This defect rendered the boat unreasonably dangerous to users such as the man. The boating supply store sold the boat in this defective condition, and the defect actually and proximately caused the man to incur severe personal injuries.

Property Damage

The measure of damages for property damage is the reasonable cost of repair, or, if the property has been almost or completely destroyed, its fair market value at the time of the accident (NOT REPLACEMENT VALUE). Courts generally do not permit recovery of emotional distress damages for negligent harm to property.

A tenant's apartment was without hot water for over a week because of a broken water heater, even though the landlord had been notified right away and the lease provided that the landlord would make repairs promptly. The tenant heated a large pot of water on the stove and started to carry it to the bathroom so she could warm up her bath. Her young nephew, who was visiting for a few days, came around the corner suddenly and collided with her. The hot water spilled on the nephew, burning him. Because the nephew had a rare blood disorder, the burns resulted in several of the nephew's toes requiring amputation. The nephew's guardian brought a negligence action against the landlord in a jurisdiction that follows the traditional rules for landowner liability. If the jury finds in favor of the landlord, what is the most likely reason?

The most likely reason to find in favor of the landlord is that proximate cause is not established. To establish proximate cause in indirect cause cases, where an intervening force combines with the defendant's conduct to cause the plaintiff's injury, the plaintiff must show that the defendant's negligence caused a foreseeable harm or caused a foreseeable reaction from a foreseeable intervening force. Intervening forces that produce a harm outside of the scope of what would normally be anticipated from the defendant's negligence are generally deemed unforeseeable and superseding. Such a superseding event will break the chain of causation and relieve the defendant of liability. Here, it is ultimately a question for the jury whether the landlord's failure to fix the water heater was a proximate cause of the nephew's injury. However, the jury could very well find that the landlord's failure to do so, even if negligent, is not a proximate cause of the nephew's burn injuries because the conduct of the nephew and the tenant are superseding forces. Given the invalidity of the other choices, lack of proximate cause is the most likely reason for the landlord to prevail.

A motorist driving his new sports car was stopped at a red light when he was struck from behind by a truck. The truck was driven by a driver who had negligently failed to stop. On impact, the door on the driver's side of the car flew open because of a latent defect in the latch that could not have been detected by the car manufacturer during the ordinary assembly process. The motorist, who was not wearing a seat belt, fell out of the open door and was injured. The jurisdiction retains traditional contributory negligence rules; however, evidence of nonuse of a seat belt is not admissible in a civil action to show contributory negligence. If the motorist asserts a claim against the car manufacturer, will the motorist prevail?

The motorist will prevail because the car was dangerously defective. The motorist's claim against the car manufacturer, as a commercial supplier of the product, likely would be based on strict liability in tort. As such, he would only need to establish that the car was in a defective condition unreasonably dangerous to users in order to recover. A defective door latch would be a dangerous defect.

nonfeasance

The omission to perform a required duty or the failure to act when a duty to act existed. Nonfeasance can more loosely be defined as "not doing something which you ought to do." The term "nonfeasance" commonly appears in the areas of contract and tort law. For example, in contract law, failure to perform the obligations of a contract altogether constitutes "nonfeasance" and could give rise to a suit for breach of contract. In the context of negligence, nonfeasance may be actionable where a landowner failed to warn invitees of concealed and dangerous conditions on their property and an invitee was injured. In contrast, misfeasance and malfeasance refer to acts which are improperly performed or wrongful.

Duty Owed to Licensee

The owner or occupier owes a licensee a *duty to warn of or make safe a dangerous condition known to the owner or occupier that creates an unreasonable risk of harm to the licensee and that the licensee is unlikely to discover*. *No Duty to Inspect* The owner or occupier has no duty to a licensee to inspect for defects nor to repair known defects. *Duty of Care for Active Operations* The owner or occupier also has a duty to exercise reasonable care in the conduct of "active operations" for the protection of the licensee whom he knows to be on the property. *Social Guests Are Licensees* The social guest is a licensee. Performance of minor services for the host does not make the guest an invitee.

A patient sought psychiatric treatment from a psychiatrist. During the treatment, which consisted of hour-long analysis sessions twice a week, the psychiatrist, unbeknownst to the patient, videotaped her. No sound recording was made of the sessions, but the psychiatrist was conducting a study on "body language" and planned to use the videotapes in those experiments. The patient learned that the psychiatrist had been videotaping their analysis sessions and brought an action against him on a theory of invasion of privacy. Which of the following arguments best supports the patient's claims in this action?

The patient's best argument would be that the psychiatrist has intruded into her physical seclusion. One of the four branches of the tort of invasion of privacy is intrusion upon a person's seclusion, which can be proved by showing an act of intrusion upon the seclusion of the plaintiff that would be highly offensive to a reasonable person, where the thing intruded upon is private. A person's body language while revealing inner secrets to a psychiatrist is probably sufficiently private to be the subject of such an action. The fact that the patient permitted the psychiatrist to observe her does not preclude her activity from being "private"; she did not consent to a permanent record being made of it.

Express Assumption of Risk

The risk may be assumed by express agreement. Such exculpatory clauses in a contract, intended to insulate one of the parties from liability resulting from his own negligence, are closely scrutinized but are generally enforceable. (Note that it is more difficult to uphold such an exculpatory clause in an adhesion contract.)

Contributory Negligence Standard of Care for Contributory Negligence: General Rule

The standard of care required is the same as that for ordinary negligence.

Nuisance Remedies

The usual remedy is damages, but injunctive relief may be available for a continuing nuisance or damages are unavailable/inadequate Abatement by self-help: for private nuisance, self-help abatement is available after notice to the D and his refusal to act. ---Only necessary force may be used. ---In PUBLIC nuisance cases, ONLY a public authority or private party who has suffered some unique damage can seek an injunction or abatement.

Comparative Negligence

The vast majority of states now permit a contributorily negligent plaintiff to recover a percentage of his damages under some type of comparative negligence system. In every case where contributory negligence is shown, the trier of fact weighs plaintiff's negligence against that of defendant and reduces plaintiff's damages accordingly. Example: Defendant negligently drove through a stop sign and collided with Plaintiff, who was contributorily negligent by driving inattentively. Plaintiff suffers damages of $100,000. If a jury finds that Plaintiff was 30% negligent and Defendant was 70% negligent, Plaintiff will recover $70,000.

What if the P only suffered emotional damage?

Then apply the rules of Negligent infliction of emotional distress

Duty of Care: Minimum Age for Capacity To Be Negligent

There is a minimum age for which it is meaningful to speak of a child being capable of conforming his conduct to a standard of care. *Most courts, however, do not fix this age at any arbitrary figure*. Each case is dealt with in terms of whether there is evidence that the individual child—plaintiff or defendant—has the experience, intelligence, maturity, training, or capacity to conform his conduct to a standard of care. It is unlikely, nonetheless, that a court would view a child below the *age of five* as having the capacity to be negligent. Or, to put it another way, it is unlikely that a court would impose a legal duty to avoid injuries to others or himself upon a child who is under five.

Characterization of Privileged Entrants

There may be a problem of characterization regarding persons entering the premises in exercise of a privilege, e.g., police, firefighters, census takers, etc. In some situations, they are characterized as licensees, in others as invitees. The following rules should be noted: (a) An entrant *serving some purpose of the possessor generally is treated as an invitee*, e.g., garbage collectors, mail carriers, etc. (b) One who comes *under normal circumstances during working hours generally is treated as an invitee*, e.g., census takers, health inspectors, etc. (c) Under the "*firefighter's rule*," police officers and firefighters are generally *treated like licensees* rather than invitees, based on public policy or assumption of risk grounds. They cannot recover for a landowner's failure to inspect or repair dangerous conditions that are an inherent risk of their law enforcement or firefighting activity.

*Applied in Enterprise Liability Cases*

This concept has been extended in some cases to encompass industry groups. Example: Daughters of women who took the anti-miscarriage drug diethylstilbestrol ("DES") contracted cancer as a result of the drug manufacturer's negligence. However, because the cancer appeared many years after the DES was ingested, it was usually impossible to determine which manufacturer of DES had supplied the drug taken by any particular plaintiff. Several courts have required all producers of DES unable to prove their noninvolvement to pay in proportion to their percentage of the market share. [See Sindell v. Abbott Laboratories, 26 Cal. 3d 588, cert. denied, 449 U.S. 912 (1980)]

Foreseeability of Harm Is True Basis of Liability Under the traditional "attractive nuisance" doctrine, it was necessary for the child/plaintiff to establish that she was lured onto the property by the attractive nuisance/dangerous condition.

This no longer is the case. Most jurisdictions have substantially revised their attractive nuisance doctrines to bring them within general negligence concepts. Foreseeability of harm to a child is the true basis of liability and the element of attraction is important only insofar as it indicates that the presence of children should have been anticipated by the landowner.

If a statute providing for a criminal penalty is applicable to a common law negligence case, the statute's specific duty will replace the more general common law duty of care. Which does a plaintiff need to show to prove the availability of the statutory standard?

To prove the availability of the statutory standard, a P must show that 1) the standards are clearly defined. 2) the P must be in the class intended to be protected by the statute and 3) the statute must have been designed to prevent the type of injury that he suffered. *CLASS OF PERSON, CLASS OF RISK* The plaintiff suffered physical injury because of the defendant's violation of the statute. The P need not suffer physical injury from the D's violation of the statute. While damages is an element of the prima facie case for negligence, any type of damages, including property damages, will suffice.

Duties Owed by Bailee *(4) Modern Trend*

Today the trend is away from such classifications and toward a rule that considers *whether the bailee exercised ordinary care under all the circumstances*. These circumstances include, e.g., value of the goods, type of bailment, custom of a trade, etc.

Causation

Two elements: actual & proximation. Always need both 1) Actual cause But for P's actions, no injury would have occurred. 2) Proximate cause - foreseeability

Defamation: Damage to P's Reputation

Type of damages depends on the type of defamation (libel, slander per se, or slander) *Libel* (printed or permanent form)- does NOT need to prove special damages; DAMAGES PRESUMED. ---Minority distinguishes between libel per se, and libel per quod (not defamatory on face) ---Majority: TV and radio broadcasts are LIBEL *Slander Per Se* - does NOT need to prove special damages; DAMAGES PRESUMED. ---4 categories: (1) relating to business/profession; (2) loathsome disease; (3) moral turpitude crime; (4) woman's chastity *Slander* - MUST PROVE SPECIAL DAMAGES; no presumed

Additional Tests for Actual Cause

Under certain circumstances, the "but for" test is inadequate to determine causation in fact. The courts must rely upon other tests. Joint Causes-Substantial Factor Test Alternative Causes

Duties of Possessor to Those on the Premises

Under the traditional rule followed in many jurisdictions, the nature of a duty owed by an owner or occupier of land to those on the premises for dangerous conditions on the land depends on the legal status of the plaintiff with regard to the property, i.e., trespasser, licensee, or invitee. Note, though, that close to half the states have abolished the distinction between licensees and invitees and simply apply a reasonable person standard to dangerous conditions on the land. A few of these states have gone even further and abolished the trespasser distinction as well.

A defendant cannot assert the defense of property if she uses force against one with a privilege to enter the property.

Whenever an actor has a privilege to enter upon the land of another because of necessity, right of reentry, right to enter upon another's land to recapture chattels, etc., that privilege supersedes the privilege of the land possessor to defend her property. A request to desist must usually precede the use of force, but if the circumstances make it clear that the request would be futile or dangerous, then a request to desist is not required. A reasonable mistake is allowed as to the property owner's right to use force in defense of property, where the mistake involves whether an intrusion has occurred or whether a request to desist is required.

SELF DEFENSE

When a person reasonably believes that she is being or is about to be attacked, she may use such force as is reasonably necessary to protect against injury. ---*Majority rule for SELF-DEFENSE*: No requirement of retreat Not available to *initial aggressor* unless the other party responds to aggressor's non-deadly force with deadly force Self-defense may extend to third party injuries caused while actor was defending herself. A reasonable mistake as to the existence of the danger is allowed Only may use force that appears reasonably necessary (including deadly force)

For comparative negligence jurisdictions, which situation requires consideration of the reasonableness of the plaintiff's conduct?

When the defendant's initial breach of duty to the plaintiff is superseded by the plaintiff's assumption of a risk

Duty of Parent to Control Child to Prevent Harm When does a parent have a duty to control child to prevent harm?

When the parent knew or should have known the child is likely to cause harm. Q: good kid has a accident Q: devil kid - mom knows or should know kid is likely to cause harm.

*Multiple Defendants Problem*

Where more than one person may have been in control of the instrumentality, *res ipsa loquitur generally may not be used* to establish a prima facie case of negligence against any individual party. Example: Plaintiff left the operating room with an injury to part of her body that was healthy prior to entering the operating room. The injury was not in the zone of the original operation. Under the traditional rule, res ipsa loquitur may not be available to establish that any individual in that room was negligent. This is so despite the fact that, clearly, someone was negligent. However, many courts today take a different approach in such cases where defendants have control of the evidence, requiring each defendant to establish that his negligence did not cause the injury. [See, e.g., Ybarra v. Spangard, 25 Cal. 2d 486 (1944)] Compare: The doctrine would be available in all states where a particular defendant had the power of control over the site of the injury. For example, Plaintiff sues Surgeon after a sponge was left in his body at the site of the surgery. Even though Surgeon left it to her assistants to remove the sponges and close up the wound, her responsibility and power of control over the surgery itself allows Plaintiff to use res ipsa loquitur against her.

*1) Joint Causes—Substantial Factor Test*

Where several causes commingle and bring about an injury—and any one alone would have been sufficient to cause the injury—it is sufficient if defendant's conduct was a "substantial factor" in causing the injury. Example: Two fires meet and burn a farm. Either fire alone would have done the damage without the other. Under the "but for" test, neither was the "cause," since, looking at either fire alone, the loss would have occurred without it. Rather than reach this result, the courts consider as causes all those things that were a "substantial factor" in causing injury.

BREACH OF DUTY

Where the defendant's conduct falls short of that level required by the applicable standard of care owed to the plaintiff, she has breached her duty. Whether the duty of care is breached in an individual case is a question for the trier of fact. Evidence may be offered to establish the standard by which defendant's conduct is to be measured, e.g., custom or usage, applicability of a statute, etc.

5) Violation of a Civil Remedy Statute

Where the statute in question provides for a civil remedy, plaintiff will sue directly under the statute; i.e., it is not a common law negligence case.

Duty of Easement and License Holders to Trespassers

While employees and independent contractors acting on behalf of the landowner have the status of the landowner, persons with an easement or license to use the land do not; they *must exercise reasonable care to protect the trespasser.* Example: Power Company obtains an easement from Leonard to run high-tension wires across Leonard's land. Because of Power Company's negligent failure to maintain the wires, one of them falls and injures Plaintiff, an undiscovered trespasser on Leonard's land. Power Company is liable to Plaintiff.

A nonunion carpenter went to work on a construction project that was involved in a labor dispute. Every morning when he arrived at work, he would be accosted by the picketers who would try to persuade him not to continue to work. One morning while the carpenter was trying to get to work, one of the union workers stopped him at the gate and told him that he should not go to work. When the carpenter insisted that the striker get out of the way, the striker said, "Try to make me, scab!" The carpenter, intending to frighten the striker, swung his hammer at him. The head on the hammer, however, was defective and it flew off, hitting the striker in the face. If the striker sues the carpenter for battery, is he likely to prevail?

Yes, because he was struck by the hammer head. The facts state that the carpenter swung the hammer at the striker "intending to frighten" him. Thus, the carpenter did an act with the intent of causing the apprehension of immediate harmful or offensive contact, i.e., an "assault." Because the striker was actually hit by the hammer head, the trial court would utilize the transferred intent doctrine to supply the necessary intent for battery. It makes no difference that the carpenter may not have known that the hammer was defective, because he set in motion the force that injured the striker.

While returning from transporting a group of children to summer camp, a bus driver and his assistant were caught in the leading edge of a forest fire raging down the high mountains. Hurrying ahead of the flames and smoke, the driver reached the last half-mile of a dirt road that ran to the main highway and safety, but he discovered that the road ahead was already blocked by fallen, burning foliage. Separating the driver's bus from the main highway, which angled off to the right, was the fenced property of a rancher. The bus driver drove across the property to reach the main highway, damaging some turf and a fence, and proceeded to the city. If the rancher asserts a claim against the bus driver to recover for the damage to his property, is the rancher likely to win?

Yes, because the bus driver damaged the rancher's property when he drove through the fence to get to the main highway. The rancher will probably win. A person who intentionally intrudes upon land in the possession of another is guilty of the tort of trespass to land. The bus driver has clearly met all the elements of this tort. However, the bus driver has the defense of private necessity, because it was necessary to drive onto the rancher's land to avoid the forest fire. Although private necessity is a defense to trespass to lands, it does not relieve the bus driver of liability for damage done to the property.

An eight-year-old child went to the grocery store with her mother. The child pushed the grocery cart while her mother put items into it. The child's mother remained near the child at all times. Another customer in the store noticed the child pushing the cart in a manner that caused the customer no concern. A short time later, the cart the child was pushing struck the customer in the knee, inflicting serious injury. Assume that the child was negligent and *the child's mother did not adequately supervise the child*. If the customer brings an action, based on negligence, against the child's mother, will the customer prevail?

Yes, because the child's mother did not adequately supervise the child's actions. Parents have a number of affirmative duties based on their special relationship to their minor children. This includes the duty to exercise reasonable care in the control of the parent's minor children. *A parent who is physically present and fails to exercise control of her child is generally NOT vicariously liable for the child's tortious behavior; rather, the parent may be liable for her OWN negligence in failing to control the child.* Because the child's mother was not adequately supervising her daughter, and it was foreseeable that the child could potentially injure someone, the customer is likely to prevail.

A petroleum company operated refineries in several states and was also engaged in the manufacture of a variety of petrochemical products. The company hired an industrial cleaning service to thoroughly clean one of its refineries. While one of the cleaning service's employees was engaged in routine cleaning activities at the refinery, one of the support legs on a crane suddenly gave way, causing part of the crane to fall onto a pipe carrying hot oil, cracking it open. The employee had his back to the pipe at the time and hot oil squirted over his back and legs, causing severe burns. The employee filed suit against the petroleum company for his injuries. The parties stipulated for trial that the crane had been designed and constructed by a crane construction specialist and was serviced at regular intervals by a reputable crane maintenance company selected by the crane construction company. The employee testified at the trial that he was injured when the pipe cracked open and submitted his medical bills and other evidence of damages. The employee introduced no further evidence. At the conclusion of the employee's case, the petroleum company moved for a directed verdict in its favor. Should the directed verdict be granted?

Yes, because the employee has done nothing to connect the petroleum company to any negligent activity that might have caused the accident. The court should grant the petroleum company's motion for a directed verdict in its favor because the employee has not established a prima facie case against the petroleum company. The question does not indicate the theory of liability for the employee's lawsuit; however, because strict liability is not applicable against the petroleum company for operation of the refinery (as discussed below) and because there is no evidence to establish that the petroleum company is vicariously liable for another party's negligence here, the employee's only feasible theory of liability is that the petroleum company itself was negligent. While the employee has established the negligence elements of duty, causation, and damages, he has not established the element of breach of duty. While breach of duty is ordinarily a question for the trier of fact, plaintiff's failure to offer any evidence on that element of the prima facie case will permit a directed verdict for defendant. Under certain circumstances, the fact that a particular injury occurred may itself establish or tend to establish a breach of duty owed, permitting the trier of fact to infer defendant's liability. This is the doctrine of res ipsa loquitur. However, for the doctrine to apply, plaintiff must show that: (i) the accident causing his injury is the type that would not normally occur unless someone was negligent; (ii) the negligence was attributable to defendant; and (iii) the injury was not attributable to plaintiff. For the second requirement, plaintiff must establish that this type of accident ordinarily happens because of the negligence of someone in defendant's position. This can often be done by showing that the instrumentality causing the injury was in the exclusive control of the defendant. Here, however, the crane that caused the injury was designed and constructed by a company other than the petroleum company and was serviced and maintained by still another company not selected by the petroleum company. Even assuming that the collapse of the crane was the type of accident that does not normally occur unless someone was negligent, there is no evidence that the petroleum company was the source of that negligence. The accident could well be attributable to negligence on the part of the manufacturer or the company hired by the manufacturer to service the crane, or simply to a defect in the materials used to construct the crane, and there is no basis for the petroleum company's being vicariously liable for the actions of either company, since the exceptions that impose vicarious liability for the conduct of an independent contractor do not apply. Since no other evidence of breach of duty was established, the petroleum company's motion for a directed verdict should be granted

A strawberry farmer held his farm open to the public to pick strawberries for a fee. The farmer knew that many patrons would eat as many strawberries out in the field as they would bring home with them, so he advertised that no chemical pesticides or fertilizers were used on his strawberries. The owner of the land adjacent to the farm began operating a soap factory, a use allowed by the zoning code. Flakes of an unavoidable chemical byproduct of the soap-making process would drift over onto the farm whenever the wind was blowing in that direction and settle onto the strawberry plants. The flakes caused no harm to the plants themselves but detracted from the appearance of the strawberries as well as their taste if eaten right off the plant; consequently, the farmer's business declined. On several occasions, the farmer complained to the factory owner, but the owner did nothing, in part because a visit to the county recorder of deeds office had convinced him that he was the true owner of a large part of the strawberry farm, although in fact it was just a recording error. Can the farmer recover damages for the harm caused to his business from the factory owner?

Yes, because the factory owner intended to conduct the activities that caused the particles to fall on the farmer's land. The farmer can recover damages for *trespass to land* because the factory owner intended to conduct the activities that caused the trespass. To establish a prima facie case for trespass to land, plaintiff must prove: (i) an act of physical invasion of plaintiff's real property by defendant; (ii) intent on defendant's part to bring about the physical invasion; and (iii) causation. Here, flakes of the chemical byproduct of the factory owner's soap factory physically invaded the farmer's property when the wind blew. The factory owner intended to bring about the trespass because, after the farmer had complained, the factory owner knew with substantial certainty that the flakes would continue to fall on the farm whenever the wind was right. Finally, the factory owner's operation of the soap factory was the cause of the flakes settling on the farmer's strawberries, completing the prima facie case of trespass to land.

In the course of repainting an apartment, the landlord of a small apartment building used a professional strength, stain-killing primer manufactured by a paint company for professional painters. The building's common ventilation system was running as the landlord applied the primer, and some fumes from the primer went through the ventilation system into the apartment of the upstairs tenant, who suffered injuries to her eyes as a result. The warning label on the can, which the landlord read, stated: "Danger. This material is extremely hazardous and volatile. Do not use near open flame. Use only with adequate ventilation." The product contained a chemical known to be harmful to people's eyes, but in the 15 years that the product has been on the market, there were no reported cases of anyone suffering an eye injury from the product. However, professional painters routinely close off or shut down any common ventilation systems in buildings before using the product. If the tenant brings an action against the paint company on a theory of strict liability, will she recover?

Yes, because the label on the product did not warn of the risk of the fumes causing eye injury. The tenant will likely prevail because the lack of a warning about eye injuries made the product unreasonably dangerous. A products liability action based on strict liability requires the following: (i) the defendant is a commercial supplier; (ii) the defendant produced or sold a product that was defective when it left the defendant's control; (iii) the defective product was the actual and proximate cause of the plaintiff's injury; and (iv) the plaintiff suffered damage to person or property. Here, the paint company is a commercial supplier of a "defective" product. Although the primer was not actually defective in that it apparently performed as it was meant to do, it is legally defective if it was unreasonably dangerous and could be made safer by adequate warnings. Here, the paint company knew of the danger and could easily have placed a specific warning on the label. Even though professional users may have known of the danger, it was not obvious, and it could have been avoided at minimal cost by including a specific warning. That would have alerted the landlord to the danger, making it more likely that he would take precautions that would have prevented the tenant from being injured. To prove actual cause where the plaintiff's claim is that the product is defective because of lack of an adequate warning, the plaintiff is entitled to a presumption that an adequate warning would have been read and heeded. Thus, the tenant can likely establish liability on her cause of action.

A teenager and his father were practicing baseball in a sandlot when the teenager hit the ball over his father's head and onto a landowner's adjacent property. The landowner had several "beware of dog" signs posted along his fence, but the father did not notice them in his haste to retrieve the ball. The father climbed over the fence into the landowner's yard and was attacked by the landowner's vicious guard dog, which was trained to maim intruders. The dog bit the father, causing him to suffer severe lacerations that required numerous stitches. If the father brings an action against the landowner to recover damages for his injuries, will he likely prevail?

Yes, because the landowner may not use a vicious dog to protect only his property. The father will prevail because the landowner may not intentionally use a vicious dog to protect only his property. One may use only reasonable force to defend property. A landowner may not use force that will cause death or serious bodily harm. Furthermore, one may not use indirect deadly force such as a trap, spring gun, or vicious dog when such force could not lawfully be directly used, e.g., against a mere trespasser.

A patient troubled by an irritating skin rash consulted a dermatologist for treatment. The dermatologist diagnosed the rash as a genetic condition that had no cure and would ultimately spread and lead to disfigurement. The patient was shocked and distressed by the diagnosis. On the advice of her family, a week later the patient consulted another doctor. That doctor immediately diagnosed the skin rash as a common bacterial infection and prescribed an ointment that cleared up the condition in a few days. Because the doctor was a friend of the family, the patient was not charged for that visit. Can the patient recover from the dermatologist for the emotional distress caused by his erroneous diagnosis?

Yes, because the misdiagnosis by the dermatologist caused the patient actual harm. The patient's distress is a recoverable element of damages caused by the dermatologist's breach of duty to her. A doctor owes a duty to possess and exercise the degree of knowledge and skill exercised by other doctors in good standing. The dermatologist also owes a duty to exercise the superior knowledge and skill that he possessed in his area of specialty. He breached his duty by misdiagnosing a common skin infection that another doctor was able to diagnose immediately. His failure to properly diagnose the condition was the actual and proximate cause of injury to the patient; *but for the misdiagnosis, she would not have had to continue suffering from the rash until the other doctor properly treated it.* The continuation of the rash and any pain and suffering from it are compensable damages that she can recover from the dermatologist. Also compensable is the emotional distress that she suffered because of the misdiagnosis. While recovery for emotional distress is restricted when there is no other injury caused by the breach, these restrictions do not apply when plaintiff is the victim of another tort that causes physical injury. Plaintiff can recover damages for emotional distress that arise from the tortious conduct.

A motorist was driving along a narrow, winding road when his car ran out of gas. Because the road had no shoulders, the motorist pushed his car onto the driveway of a landowner. Finding no one home at the house, the motorist started walking toward a gas station he had passed a mile back. While he was gone, the landowner returned and found the car in his driveway, with two of its wheels partially on his flower garden. Although the landowner had not posted any "no trespassing" signs, he believed he had the right to remove the car from his property. The car was unlocked, so he released the parking brake and pushed the car back onto the edge of the road, and then reset the brake. Before the motorist could return, a truck had sideswiped the car, damaging it. Can the motorist recover against the landowner for the damage to his car?

Yes, because the motorist was privileged to leave his car there. The motorist can recover against the landowner because the privilege of *private necessity* applied. A person may interfere with the real or personal property of another *when the interference is reasonably and apparently necessary to avoid threatened injury from a natural or other force AND the threatened injury is substantially more serious than the invasion that is undertaken to avert it.* Here, it was necessary for the motorist to push the car into the landowner's driveway to avoid the threat of other vehicles colliding with it on the narrow road, which would be a substantially more serious harm than any damage to the landowner's property. Hence, the landowner was not entitled to move the car back into the road, and the motorist can recover from the landowner the damages that resulted from doing so.

A valet parking attendant at a restaurant negligently left the keys of a car in the ignition when she parked it on a side street some distance from the restaurant, which was located in a high crime area. While dining, the car's owner received a text message from the security company that operated his car's anti-theft system that his key was in his ignition for over 30 minutes without the car running. The owner started to get up to check with the valet service but then his meal arrived and he promptly forgot about the car. About 20 minutes later, a teen saw the key in the ignition of the unlocked car and drove off with the car. By the time it was discovered that the car had been stolen, the car had been wrecked and the teen had fled. The owner sued the parking company that employed the attendant for the loss of his car. Is the owner likely to recover?

Yes, because the negligence of the parking attendant created the opportunity for the theft. The car owner is likely to recover from the parking company. The general rule of proximate cause is that the defendant is liable for all harmful results that are the normal incidents of and within the increased risk caused by his acts. In indirect cause cases, an independent intervening force may be foreseeable where the defendant's negligence increased the risk that these forces would cause harm to the plaintiff. Even a criminal act by a third party will not cut off the defendant's liability if the defendant's negligence created a foreseeable risk that a third person would commit the crime. Here, the parking company's employee negligently left the key in the ignition when she parked the owner's car on a side street away from the restaurant, creating a foreseeable risk that the car would be stolen. But for that negligence, the car would not have been stolen. A jury is likely to find that the teen's intervening criminal act was sufficiently foreseeable that the parking company will be held liable for at least some of the damage suffered by the owner of the car. *Superseding cause analysis does not apply to the plaintiff's negligence. The car owner's failure to follow up on the alert about his car is an issue of contributory negligence, which under pure comparative negligence rules is not a complete defense.*

A pedestrian was injured when hit by a chair that was thrown from an upper-story hotel window. The pedestrian sued the occupants of all the rooms from which the chair might have been thrown. At trial, the pedestrian has been unable to offer any evidence as to the exact room from which the chair was thrown. The defendants have filed a motion for a directed verdict. Should the court grant the motion?

Yes, because the pedestrian has failed to offer evidence that the defendants jointly engaged in tortious conduct. When two or more tortious acts combine to cause an indivisible injury to a plaintiff, each tortfeasor is jointly and severally liable for that injury. Defendants in a civil suit can be held jointly and severally liable only if their concurrent acts brought about the harm to the plaintiff. These acts do not have to be simultaneous, they only must contribute to the same event. Here, only one occupant threw a chair out of the upper-story hotel window. There is no evidence that the defendants all contributed to the harm that the pedestrian suffered. Therefore, the motion for directed verdict should be granted.

A college student owned a very popular video game system that was out of stock in most stores. He agreed to let his friend use the system for a few days, on condition that he return the system by the weekend because the student was hosting a small party. On the morning of the party, the friend still had not returned the game system, so the student went to the friend's apartment and demanded it back. The friend refused, so the student grabbed the system and wrestled it out of the friend's hands. If the friend sues the student for battery, will he recover on this claim?

Yes, because the student had originally agreed to lend the game system to the friend. The friend will recover for battery because the student did not have the right to use force. The defense of recapture of chattels is limited by the circumstances of the original dispossession. When another's possession of the owner's chattel began lawfully, the owner may use only peaceful means to recover the chattel. Force may be used to recapture a chattel only when in "hot pursuit" of one who has obtained possession wrongfully (e.g., by theft). Here, the friend's initial possession of the game system was a bailment, because the student consented to his borrowing it. Thus, the student is not entitled to use force to recover it, and his wrestling it away from the friend constituted the requisite harmful or offensive contact to make the student liable for battery.

A law student rented a furnished apartment. His landlord began to solicit his advice about her legal affairs, but he refused to provide it. The landlord then demanded that he vacate the apartment immediately. The landlord also began engaging in a pattern of harassment, calling the student at home every evening and entering his apartment without his consent during times when he was at school. During these unauthorized visits, she removed the handles from the bathroom and kitchen sinks but did not touch anything belonging to the student. The lease has a year to run, and the student is still living in the apartment. The student has sued the landlord for trespass. Is he likely to prevail? If so, what kind of damages are recoverable?

Yes, for injunctive relief, compensatory damages, and punitive damages. There is evidence supporting compensatory damages (for emotional distress and the removal of the faucet handles) and punitive damages (based on the landlord's malicious intent and ill will). Because the lease is still in effect and the trespasses are repeated and ongoing, injunctive relief should also be available.

Can P recover for damage to property?

Yes, the costs of repair or replacement (FMV when tort was committed). BUT NOT FOR EMOTIONAL DISTRESS AS A RESULT. EX: Dog is hit by negligent driver. Can't get emotional damages.

DUTY OF CARE Rescuers Is D liable if if he negligently puts himself or a third person in peril and plaintiff is injured in attempting a rescue?

Yes. A rescuer is a foreseeable plaintiff as long as the rescue is not reckless; hence, defendant is liable if he negligently puts himself or a third person in peril and plaintiff is injured in attempting a rescue. Note, however, that the *"firefighter's rule"* (infra, 3.d.2)c)(2)(c)) may bar firefighters and police officers, on public policy or assumption of risk grounds, from recovering for injuries caused by the risks of a rescue

A court will not likely permit a D to argue an assumption or risk defense when

a common carrier or public utility. Public policy grounds. Cannot have a disclaimer on a ticket or a bill, etc.

The *Economic Loss Rule* is a doctrine of law that prohibits a product liability claim being brought against a manufacturer for

a defective product that only destroys itself, without harm to other property or to a person. In those instances where the product fails but only damages itself and nothing else, the plaintiff's only remedy is to sue for breach of contract against the manufacturer of the product. The plaintiff cannot seek recovery from the manufacturer under product liability causes of action.

c) Other Situations: The plaintiff may be able to recover without proving the two requirements for this tort in other special situations where the defendant's negligence creates a great likelihood of emotional distress. These include:

a defendant providing an erroneous report that a relative of the plaintiff has died or a defendant mishandling a relative's corpse.

Unlike in recapturing chattels, force may not be used by __________.

a landowner to regain real property after being tortiously dispossessed Most states today do not allow resort to "self-help"; one who has been wrongfully excluded from possession of real property may bring an ejectment action or other summary procedure to recover possession. Hence, the owner who uses force to retake possession is liable for whatever injury she inflicts. (In former years, under the common law, a landowner tortiously dispossessed of real property could use reasonable force to regain possession, if she acted promptly upon discovery of the dispossession.) An owner of chattel may use force to recapture the chattel. An owner may use reasonable force to recapture a chattel when in "hot pursuit" of the tortfeasor. A demand for return of the chattel must be made before force is used, unless the demand would be futile or dangerous. However, force can be used only against the tortfeasor or a third party who knows that the chattel was tortiously obtained. If an innocent third party has obtained the chattel, the owner is no longer privileged to use force to effect a recapture of the chattel. A citizen may use force to effect a misdemeanor arrest. However, the citizen is allowed to use only the amount of force necessary to effect the arrest and never deadly force. A property owner may use force to defend the property from tortious interference. Although a property owner may use reasonable force to defend property, she may not use force that will cause death or serious bodily harm. Furthermore, one may not use indirect deadly force such as a trap, spring gun, or vicious dog when such force could not lawfully be directly used, e.g., against a mere trespasser.

Vicarious Liability 5. *Parent-child*

a parent is NOT vicariously liable for a child's torts at common law (but statutes in many states impose limited liability for the child's intentional torts) However, many states by statute make parents liable for the willful and intentional torts of their minor children up to a certain dollar amount ($10k) *Child acting as agent for parents* Courts may impose V.L. if child committed a tort while acting as parents' agent Can be DIRECTLY LIABLE for NEGLIGENTLY entrusting child to do something or if parent knows of a child's conduct on past occasions showing tendency to injure another's person or property; may be liable for not using due care in exercising control to mitigate such conduct.

Tortious Interference with Family Relationships Parent-Child

a parent may maintain an action for loss of a child's services and consortium as a result of D's tortious conduct; whether intentional or negligent. ---a child does NOT have a action in most states for injury to parent Derivative action, thus any defense that would reduce or bar recovery by the injured family member also reduces/bars recovery for interference with the family relationship

Public figure

a person who has achieved pervasive fame or notoriety by voluntarily assuming a central role in a public public controversy

Information Defects

a product may be defective as a result of the manufacturer's *failure to give adequate instructions or warnings as to the risks involved in using the product that may not be apparent to users*. For *prescription drugs and medical devices, warnings given to "learned intermediaries" (doctor)* will usually suffice in lieu of warnings to the patient.

A duty of care is generally not owed to __________.

a rescuer, unless the defendant negligently put herself or a third person in peril

Multiple Defendants 2. If joint and several liability applies, *contribution* allows

a tortfeasor who paid more than his share of the damages to recover the excess from other tortfeasors in proportion to their fault ---NOT applicable to intentional torts EX: Jury decides that Defendants A was 10% at fault, B - 30%, C - 60%. P gets 100% from A. Defendant A can now sue B for 30% and C for 60%. *Indemnity* - shifts entire loss. Available: 1) by contract 2) vicarious liability situations 3) under strict products liability *Comparative Contribution* - most comparative negligence states have adopted; contribution is in proportion to fault

Excuse for Violation of Statutes Violation of some statutes may be excused:

a) Where compliance would cause more danger than violation; e.g., defendant drives onto wrong side of road to avoid hitting children who dart into his path; or b) Where compliance would be beyond defendant's control; e.g., blind pedestrian crosses against light.

Negligent misrepresentation

a. Defined: (1) Misrepresentation by the defendant in a business or professional capacity, (2) breach of duty to the plaintiff (3) actual reliance on misrepresentation (causation) (4) justifiable reliance, and (5) damages DIFFERENCE WITH INTENTIONAL MISREPRESENTATION ---Scienter & Intent to induce (intentional) vs. breach of duty (negligent) b. The defendant owes a duty only to those to whom the misrepresentation was directed or those who the defendant knew would rely on it

Intentional misrepresentation (fraud)

a. Defined: *MS. JARID* (1) Misrepresentation by the defendant (2) Scienter (knowledge of falsity or reckless disregard as to truth/falsity) and (3) justifiable reliance (4) actual reliance (causation) (5) intent to induce reliance, ,and (6) damages b. The defendant generally has no duty to disclose material facts but may be liable for active concealment

When strict liability applies, each supplier of a defective product is liable to an injured person, but each supplier has a right of indemnification against ...

all previous suppliers of the defective product in the distribution chain, with the manufacturer of the defective product ultimately liable.

*design defect*

all products of the line have dangerous characteristics P must usually show that a less dangerous modification or alternative was economically feasible; without serious impact of price or utility

Unlike with products liability cases based on negligence, those based on strict liability do not require that suppliers have

an opportunity to inspect. Thus, for a case based on the sale of a defective product, a retailer in a strict liability action may be liable for a manufacturing or design defect simply for being a commercial supplier of that defective product, even if it had no opportunity to inspect the manufacturer's product before selling it. In a negligence action, the supplier's negligence must be proved. Products liability cases based on negligence and those based on strict liability both require that an injured bystander be foreseeable. While privity is not required in these cases, and bystanders are protected and may bring a claim under either theory, they must be foreseeable. Liability under these theories applies to foreseeable plaintiffs. Products liability cases based on negligence and those based on strict liability both prohibit recovery of solely economic losses (the product does not work as well as expected or requires repairs). The types of damages recoverable under both theories are the same: personal injury and property damages. Economic loss cannot be the sole damage claim. As under claims based on negligence, those based on strict liability will impose liability even though an intermediary negligently failed to discover the defect. The same concepts of proximate cause govern negligence and strict liability actions. The negligent failure of an intermediary to discover a defect is not a superseding cause and does not cut off the supplier's strict liability. However, if the intermediary's conduct becomes something more than ordinary foreseeable negligence, then it does become a superseding cause.

Products Liability: Express Warranty

any affirmation of fact or promise concerning goods that becomes part of the basis of the bargain creates an express warranty *Who can sue*: any consumer, user, or bystander. ---If a buyer sues, the warranty must have been "part of the basis of the bargain" ---If a P is NOT in privity (EX: bystander), she need not have relied on the representation as long as someone did. An express warranty may also be made in a lease of goods *Breach* - fault need not be shown. P only needs to show that product didn't live up to warranty Causation - same as Negligence Damages - personal injury and property; AND pure economic loss recoverable Defenses (assumption of risk, contributory negligence, failure to give notice of breach) Disclaimers: ineffective unless event is consistent with warranty (UNLIKELY)

Implied Warranty of Fitness for a Particular Purpose

arises when the seller knows or has reason to know the particular purpose for which the goods are required and that the buyer is relying on the seller's skill and judgment in selecting the goods Who can be sued: any seller (NOT just merchants)

The failure of a retailer to take action after discovering a dangerous defect may prevent establishing _____ against a manufacturer in a strict products liability action.

causation The same concepts of proximate cause that govern negligence and strict liability actions are applicable to strict liability actions for defective products. As with products liability cases based on negligence, the negligent failure of a retailer to discover a dangerous defect does not cut off the supplier's strict liability. On the other hand, when the intermediary's conduct becomes something more than ordinary foreseeable negligence, it becomes a superseding cause. Hence, the conduct of a retailer who discovered a dangerous defect and then took no action (such as alerting the manufacturer, warning the consumer, or removing the product from sale) constitutes more than ordinary foreseeable negligence and may cut off the manufacturer's liability.

Invitee

come onto your land for a purpose related to your business

Licensee

comes onto your land for their own purpose or business (social guest)

Publication

communication to a 3P who understands it. It is the intent to publish, NOT the intent to defame, that is the requisite intent. Each repetition is a separate publication However, for magazines, newspapers, etc. most states have adopted a *"single publication" rule* under which all copies are treated as one publication *Primary Publisher* - newspapers, TV stations, are liable to the same extent as the author or speaker. ---one who repeats a defamation is liable on the same general basis (even if she states the source and says she doesn't believe the defamation) *Secondary Publisher* - one selling papers or playing audio files; only liable if he knows or should know of the defamatory content Internet service provider is NOT a publisher

*Libel* -

defamation in relatively permanent form. DAMAGES ARE PRESUMED. No need to prove. But, the more you prove the more you get. But no directed verdict will be entered against you.

*Slander* -

defamatory statement that is spoken or oral in nature.

Vicarious Liability *Partners and Joint Venturers*

each member of a partnership or joint venture is vicariously liable for the tortious conduct of another member committed in the *scope and course* of the affairs of the business

Multiple Defendants 1. Under the rule of *joint and several liability*, when two or more tortious acts combine to proximately cause an indivisible injury to the plaintiff,

each tortfeasor is *jointly and severally liable* for the injury. THUS: a. A wronged party may sue any or all of them, and collect the total damages awarded by a court from any or all of them. b. Many states have abolished the rule for (i) those tortfeasors less at fault than the plaintiff, or (ii) all tortfeasors for noneconomic damages ---Thus, liability with be proportional to fault

SLANDER-

spoken- special damages required

At the defendant's trial for stealing an automobile, the defendant called a character witness who testified that the defendant had an excellent reputation for honesty. In rebuttal, the prosecutor calls another witness to testify that he recently saw the defendant cheat on a college examination. The prosecution witness's testimony should be

excluded, because the defendant's cheating can be inquired into only on cross-examination of the defendant's witness. The prosecutor cannot use specific incidents of prior misconduct to rebut the character evidence testimony given by the defendant's witness, except on cross-examination.

breach can be shown when a D's conduct

falls short of a reasonable standard of care

Egg-shell skull plaintiff principle

for purposes of damages, you take your P as you find P. Any sensitive, pre-existing injury does not cut off liability. Don't have to foresee extent of P's harm. D is liable for all resulting injuries, even if unforeseeable. EX: P has hemophilia, bleeds to death from relatively minor injury from D. D is liable for lifetime of lost earnings, wrongful death damages, etc.

Comparative Negligence/Fault

gets rid of all defenses to Negligence except Express Assumption of Risk

Any time the answer choice lists new facts

good sign that the answer is wrong unless the facts match up with the original fact pattern.

A person commits battery when

he causes a harmful or offensive touching of the plaintiff's person with an intent to do so

MISREPRESENTATION: Third Party Reliance

if a 3P relies on the D's representation, the D will be liable if she could reasonably foresee that the 3P would so rely Usually only applies to FACTS NOT OPINIONS, unless the D offering the opinion has a superior knowledge of the subject

A plaintiff sues a department store for personal injuries, alleging that while shopping she was knocked to the floor by a merchandise cart being pushed by a stock clerk and her back was injured as a result. The stock clerk testified that the plaintiff fell near the cart but was not struck by it. Thirty minutes after the plaintiff's fall, the stock clerk, in accordance with regular practice at the department store, had filled out a printed form, "Employee's Report of Accident," in which he stated that the plaintiff had been leaning over to spank her young child and in so doing had fallen near his cart. Counsel for the department store offers in evidence the report, which had been given to him by the stock clerk's supervisor. The judge should rule the report offered by the department store

inadmissible, because it is hearsay, not within any exception. The stock clerk's written report is hearsay because it is an out-of-court statement being offered for the truth of the matter asserted - that the plaintiff fell and was not hit by the cart. And, because the stock clerk's report does not fall within any of the exceptions to the hearsay rule, it is therefore inadmissible. The business records exception to the hearsay rule requires a record to be made in the regular course of business, related to the nature of the business, at or near the time of the event being recorded. *This is not a business record here because it was made with an eye towards potential litigation*, not during the regular course of business or related to the business's typical activities.

Proximate Cause

is nothing more than a mechanism we use to limit the scope of liability. D is only liable for those harms that are *within the risk* that made D's conduct negligent in the first place. All about foreseeability.

The owner is strictly liable for damage caused by the trespass of her animal only if

it was reasonably foreseeable.

Common examples of negligent misrepresentation

lawyers, accountants, other professionals can be liable if they don't use due care to see that the info on which they base their advice is reliable typically COMMERCIAL settings Only will attach if the PARTICULAR plaintiff is known. (Not a 3P)

Which of the following is correct as to lessors of realty?

liability for conditions on the leased premises hinges on who occupies and controls the premises

Res Ipsa - used by desperate Ps that don't know what happened. What to look for in MC question? Elements?

look for a "motion" question. How will the court rule on a motion? Elements *1) Event which would not normally occur absent negligence* - in the real world usually means expert testimony *2) Any negligence is attributable to D* = inference of negligence -No one else could have done it; -Usually: D had *exclusive control of the injury-causing instrumentality* *3) P (or any 3P) is not responsible for the event* *What does it mean if both elements are met?* - *P gets to jury and spared a directed verdict.* - does NOT mean that the P automatically wins. -Res Ipsa is all about what a jury could infer. -Typically the call of the question relates to a motion for directed verdict, MSJ, etc. -On the other hand CL negligence will be about "liability, damages, will Bob prevail, recover" -You make a directed verdict/SMJ because you want the case to end. Taking it away from the jury. -When the judge agree and dispose of the case, and when will he allow it to continue? -If there's enough evidence where the jury could infer negligence, the denied and we keep going -If there's no evidence such that a jury could infer negligence, Motion granted, let's go home -How will you know if there's any possibility negligence occurred? --The question will tell you

Defense of Property

may only use reasonable force. A request to desist or leave must first be made unless it clearly would be futile or dangerous Doesn't apply after tort is committed; however one may use force in HOT PURSUIT of another who has tortiously disposed the owner of chattels; but not innocent party Reasonable mistake allowed as to whether an intrusion has occurred or whether a request to desist is denied NOT available against one who has a superseding privilege because of necessity, recapture of chattels, etc. ---mistake NOT allowed here, unless entrant conducts the entry so as to lead the D to reasonably believe it is not privileged (such as by refusing to say what the necessity is) Reasonable force permitted, but NEVER deadly force unless property invasion also entails serious threat of bodily harm Self-help no longer allowed to forcefully reenter property; must use ejectment action ---same with chattels; must use peaceful means unless in hot pursuit

DEFAMATION mitigating factors (NOT defenses, just might reduce damages)

mitigating factors: - no malice, - retraction, - anger of the speaker provoked by plaintiff, etc. NOT defense; but may be considered by the jury on the damages issue

difference between res ipsa and negligence per se?

negligence per se conclusively establishes duty and breach (statutory violation - class of person, class of risk) res ipsa establishes an inference of breach sufficient to survive MSJ or directed verdict, but the jury can decide otherwise looking at the facts. Also, in negligence per se applies, and the D is not contesting causation and damages, the PLAINTIFF can move for a directed verdict. or MSJ.

As with products liability cases based on negligence, the negligent failure of an intermediary to discover the defect does NOT cut off the supplier's strict liability. But when the intermediary's conduct becomes something more than ordinary foreseeable negligence, it becomes a

superseding cause EX: The manufacturer can argue that the retailer's failure to take action after discovering a defect was not foreseeable and therefore cuts off the manufacturer's liability for the defect.

If an animal commits an unforeseeable trespass, the owner of the animal will

not be held strictly liable for damage caused by the animal.

Negligent Misrepresentation -

one who negligently gives false info to another is subject to liability for physical harm caused by action taken by the other in reliance on that false information

Assumption of risk

originally a total bar of recovery P knew of the risk and voluntarily assumed it. EX: waiver of liability before joining BJJ class. Harder cases are the implied AOR. P must have an alternative; if there's no other way to cross the street, etc., the P didn't AOR

Duty to Prevent Harm from Third Persons Generally, there is no duty to prevent a third person from injuring another. In some situations, however, such an affirmative duty might be imposed. In such cases, it must appear that

the defendant had the actual ability and authority to control the third person's action. Thus, for example, bailors may be liable for the acts of their bailees, parents may be liable for the acts of their children, employers may be liable for the acts of their employees, etc. It is generally required for imposition of such a duty that the defendant knows or should know that the third person is likely to commit such acts as would require the exercise of control by the defendant.

Defenses to Intentional Torts: The shopkeeper's privilege

permits the reasonable detention of someone the shopkeeper reasonably believes has shoplifted goods REQUIREMENTS: 1) reasonable belief as to theft 2) detention conducted in reasonable manner, only non-deadly force 3) only for a reasonable period of time and only for purpose of making an investigation

A "reasonable person" has which of the following of the defendant's personal characteristics?

physical (duty of a reasonable blind, deaf, etc. person)

1) Plaintiff Must Be Within the "Zone of Danger" The plaintiff usually must show that her distress has been caused by a threat of

physical impact; i.e., she was within the "zone of danger." Example: Driver negligently ran a red light and skidded to a stop inches away from Pedestrian, who was properly crossing the street in a crosswalk. Pedestrian's shock from nearly being run over caused her to suffer a heart attack. Pedestrian can recover for negligent infliction of emotional distress because she was in the zone of danger.

2) Plaintiff Must Suffer Physical Symptoms from the Distress For the plaintiff to recover damages, most courts usually require that the defendant's conduct cause the plaintiff emotional distress that manifests itself in

physical symptoms (e.g., a nervous breakdown, miscarriage, or heart attack, but note that severe shock to the nervous system that causes physical symptoms will satisfy this requirement). A growing minority of states have dropped the requirement of physical symptoms.

3) Effect of Establishing Violation of Statute Most courts still adhere to the rule that violation of a statute is "negligence per se." This means that

plaintiff will have established a *conclusive presumption of duty and breach of duty.* (Plaintiff still must establish causation and damages to complete the prima facie case for negligence.) A significant minority of courts, however, are unwilling to go this far. They hold either that (i) a rebuttable presumption as to duty and breach thereof arises, or (ii) the statutory violation is only prima facie evidence of negligence.

Under the "*firefighter's rule*,"

police officers and firefighters are generally *treated like licensees* rather than invitees, based on public policy or assumption of risk grounds. They cannot recover for a landowner's failure to inspect or repair dangerous conditions that are an inherent risk of their law enforcement or firefighting activity.

*manufacturing defect*

product varies from the other products in the manufacturing process and is dangerous beyond the expectation of the ordinary consumer; more dangerous than the products that were made properly. *D liable if P can show that product failed to perform as safely as an ordinary consumer would expect* ---D must anticipate reasonable misuse (standing on a chair) Test applies to food products

b) Special Relationship Between Plaintiff and Defendant The defendant may be liable for directly causing the plaintiff severe emotional distress when a duty arises from the relationship between the plaintiff and the defendant, such that

the defendant's negligence has great potential to cause emotional distress. Many states drop the requirement of physical symptoms in this situation as well. Example: Doctor negligently confused Patient's file with another and told Patient he had a terminal illness. Patient, who in fact did not have the illness, was shocked and became nauseated as a result. Patient can recover for negligent infliction of emotional distress. Although there was no threat of physical impact from Doctor's negligence, negligently providing a false diagnosis of a terminal illness creates a foreseeable risk of severe emotional distress to the patient. *Other examples* a defendant providing an erroneous report that a relative of the plaintiff has died or a defendant mishandling a relative's corpse.

A landowner usually must make a _____ before defending her property.

request to desist A request is not required if the circumstances make it clear that the request would be futile or dangerous. A landowner's right to defend her property does NOT supersede other privileges. If another is privileged to enter upon a landowner's property, due to necessity, a right of reentry, right to recapture chattels, etc., that privilege supersedes the landowner's right to defend her property. A landowner CAN make a mistake as to whether an intrusion of her property has occurred. A reasonable mistake is allowed as to the landowner's right to use force in defense of property, if the mistake involves whether an intrusion has occurred or whether a request to desist is required. A mistake is not allowed, however, if the entrant has a privilege to enter the property that supersedes the landowner's right to defend her property. Then the landowner is liable for her mistake, unless the entrant intentionally or negligently caused the mistake. A landowner may NOT use deadly force to defend her property. One can only use reasonable force to defend her property, not force that will cause death or serious bodily harm. Deadly force can be employed only if the landowner or another on the property is physically threatened, such that she may act in self-defense or defense of others

Violation of Statute

the existence of a duty owed to plaintiff and breach thereof may be established by proof that defendant violated an applicable statute.

Superseding cause -

separate act so unforeseeable it DOES cut off liability. Only liable for first injury, but not unforeseeable

Intervening cause -

separate act which does NOT cut off liability = foreseeable If it's foreseeable, the P will still pay. All negligence is foreseeable

When a person intentionally or negligently places themselves in a dangerous situation, it is foreseeable that

someone might try to help. Danger invites rescue! If P rescuer is injured, D rescuee is liable.

Proximate cause

sufficient nexus between the D's negligent conduct and the injury. Proximate cause limits liability to negligent conduct that foreseeably would cause the P's harm. RISK RULE: D is liable for the type of harm, the risk of which made D's conduct negligent in the first place EX: speeding leads to a crash. A crash is the type of harm, the risk of which made D's speeding negligent in the first place. Because a crash is a foreseeable outcome of speeding, speeding is the proximate cause of the P's harm

unforeseeable intervening forces is called

superseding

Users of Recreational Land In almost all states, a different standard applies by statute to users of recreational land. If an owner or occupier of open land permits the public to use the land for recreational purposes without charging a fee, the landowner is NOT liable for injuries suffered by a recreational user unless

the landowner willfully and maliciously failed to guard against or warn of a dangerous condition or activity. Example: The owner of a large tract of undeveloped rural land who permits the general public to use a pond on the land for swimming and fishing would be covered by this type of statute, whereas the owner of a swimming pool who permits his house guests to swim whenever they visit would not be covered by the statute (he would owe his guests the usual duties owed to licensees).

The general rule of proximate cause is that the defendant is liable for all harmful results that are

the normal incidents of and within the increased risk caused by his acts. In other words, if one of the reasons that make defendant's act negligent is a greater risk of a particular harmful result occurring, and that harmful result does occur, defendant generally is liable. This test is based on foreseeability.

A driver drove his car into an intersection and collided with a fire engine that had entered the intersection from the driver's right. The accident was caused by negligence on the driver's part. As a result of the accident, the fire engine was delayed in reaching the plaintiff's house, which was entirely consumed by fire. The plaintiff's house was located about 10 blocks from the scene of the accident. If the plaintiff asserts a claim against the driver, the plaintiff will recover

the part of his loss that would have been prevented if the collision had not occurred. This is the appropriate test for the applicable causation issue. The facts indicate aid had been undertaken and, but for the driver's negligent driving, the damage to the plaintiff's home would have been reduced because the fire engine would have arrived sooner.

The concept of imputed contributory negligence allows a defendant to use the contributory negligence of a third party as a defense against the plaintiff, where

the plaintiff and the third party stand in a particular relationship.

Assumption of Risk The plaintiff may be denied recovery if he assumed the risk of any damage caused by the defendant's acts. This assumption may be expressed or implied. To have assumed risk, either expressly or impliedly,

the plaintiff must have known of the risk and voluntarily assumed it. It is irrelevant that plaintiff's choice is unreasonable. Assumption of risk is not a defense to intentional torts. It is, however, a defense to wanton or reckless conduct.

Public concern

to determine, courts will look at content, form, and context of the publication

Direct Cause, proximate cause cases

uninterrupted chain of events defendant almost always liable (unless crazy unforeseeable)

What Is a Dangerous Condition in the Attractive Nuisance Doctrine?

where something on the land is likely to cause injury to children because of their inability to appreciate the risk. This *usually is an artificial condition*, but in some circumstances a natural condition might suffice.

Depending on the circumstances, strict liability may be imposed on the owners of what 3 types of animals?

wild animals, domestic animals, and trespassing animals. Unless an owner of wild animals can rely on a public duty exception (e.g., a zookeeper), the owner is strictly liable for injuries caused by the wild animals, even those kept as pets. An owner is strictly liable for the damage done by the trespass of his animals (other than household pets) as long as the damage was reasonably foreseeable. It does not matter that the owner acted with reasonable care to keep them from trespassing. Normally, the owner of a domestic animal is not strictly liable for injuries it causes. Strict liability will attach, however, if the owner knows of the domestic animal's dangerous propensities (i.e., propensities more dangerous than normal for that species). This rule applies even if the animal has never injured anyone.

LIBEL-

written or other permanent form, NO special damages required (money damages)

Eggshell Skull Rule

you take your victim as you find them. normal people would've had minor injuries, but this P gets seriously injured. Even though not foreseeable, D is liable for all damages.


Related study sets

8th Grade my Perspectives ELA End-of-Year Test (Online)

View Set

Saunders Integumentary Problems (Ch 43) Practice Questions

View Set

eegoo Part 19 부정사 형용사용법

View Set

Babylonian and Assyrian Civilization

View Set